Växthuseffekten, enligt (en av) 2021 års Nobelpristagare i fysik

Gästinlägg nr 26 av Gösta Pettersson

2021-års Nobelpris i fysik tilldelades bland andra Syokuro Manabe, detta med hänvisning till hans modellbeskrivningar av den växthuseffekt (GreenHouse Effect, GHE) som det stora flertalet nutida klimatologer anser koldioxid och andra så kallade växthusgaser framkalla. Det tar jag som en klar indikation på att GHE-proponenter inom nutida klimatologisk forskning anser växthuseffekten uppkomma enligt de mekanistiska principer som Manabe angett i sina modellbeskrivningar av effekten. Principer som redan 1967 angavs i ett arbete av Manabe och Wetherald (M&W).
Untitled
 I detta inlägg ska jag behandla M&W-modellens beskrivning av temperaturförhållandena i troposfären. Jag ska ställa växthuseffektens betydelse i relation till den gravito-termala effektens betydelse, så som den senare effekten på 1870-talet beskrevs och kvantifierades av Loschmidt enligt mitt KU-inlägg 2022-05-26. En effekt vars empiriskt befästa betydelse fick Maxwell att vid samma tid föreslå att de troposfäriska temperaturerna återspeglar uppkomsten av en ”konvektiv jämvikt” karakteriserad av de av Loschmidt härledda sambanden. Tankar som vidareutvecklats av forskare av vad jag kommer att kalla den Maxwellska skolan. Och som enligt mitt KU-inlägg 2022-05-27 fått dem att dra slutsatsen att den 32-gradiga atmosfärseffekt som FN:s klimatpanel (IPCC) åberopar som ett förstahandsbelägg för existensen av en signifikant ”naturlig” växthuseffekt i själva verket fullt tillfredsställande kan förklaras som en gravito-termal effekt.

För säkerhets skull vill jag förtydliga att jag med växthuseffekt fortfarande avser begreppet så som det definierats av IPCC, dvs. som den radiativt betingade förhöjning av jordytans temperatur som förmodas åstadkommas av atmosfärens växthusgaser på grund av deras specifika förmåga att absorbera och återutsända infraröd värmestrålning.

Jag inser att många läsare hyser ringa intresse för den detaljerade vetenskapliga argumentationen och främst är intresserade av vilka slutsatser jag drar från det jag anför. Sådana läsare kan hänvisas till inläggets avslutande två avsnitt.

Pehr Björnbom redogjorde i ett KU-inlägg daterat 2012-06-01 för grundtankarna bakom M&W-modellen. Den utgår från att det basalt existerar en troposfärisk temperaturgradient på grund av den gravito-termala effekten och accepterar Maxwell-skolans förmodan att denna basala gradient återspeglar uppkomsten av en konvektiv jämvikt. M&W kombinerade detta med beräkningar av vilken temperaturgradient som borde uppstå om man enbart tar hänsyn till strålningseffekter, dvs. vid en tänkt ”radiativ jämvikt”. Om den senare gradienten i något luftskikt har högre ”lapse rate” än vad den konvektiva jämvikten föreskriver, så tenderar strålningen enligt M&W att åstadkomma en instabil situation med varma luftlager under kallare. Då bör det enligt deras tänkande uppstå en konvektion (uppvärmd luft stiger normalt uppåt) som resulterar i att den konvektiva jämviktens temperaturvärden kommer att antas. I det motsatta fallet tenderar strålningen enligt M&W att åstadkomma ett stabilt tillstånd med varmare luft ovanför kallare, och då kommer den radiativa jämvikten att kunna bevaras och dess temperaturvärden att antas.

lapse

Fig. 2 ovan återger trunkerat Fig. 16 i M&W:s ovan angivna arbete från 1967, vilken visar vilka troposfäriska temperaturgradienter som enligt deras modell bör uppkomma för olika lufthalter av koldioxid inom det för vår tid intressantaste intervallet 300–600 ppm.

Pehr Björnboms kommentar till M&W-modellens beräkningsresultat var att de stämmer riktigt bra med vad som observerats (vilket torde betyda att han syftar på modellbeskrivningen för 300 ppm koldioxid).

Så tycks onekligen vara fallet. Men för troposfärens vidkommande har detta sin förklaring i att modellresultaten huvudsakligen återger den temperaturgradient som uppkommer genom den gravito-termala effekten. Variationer av lufthalten av växthusgasen koldioxid från 300 ppm till 600 ppm påverkar enligt M&W-modellen endast i ringa mån den gravito-termalt uppkomna troposfäriska temperaturgradient som meteorologer sedan länge befunnit föreligga och tillfredsställande kunnat förklara i ljuset av Loschmidts ekvationer.

Men den lilla påverkan som föreligger enligt M&W-modellen är förvisso av sådan art att den ger upphov till en växthuseffekt. Enligt Fig. 2 är effekten av sådan styrka att jordytans temperatur stiger cirka 2 °C när lufthalten av koldioxid ökar från 300 ppm till 600 ppm. Denna växthuseffekt orsakas enligt Fig. 2 inte av någon förändring av tropopausens belägenhet eller temperatur, vilket hade kunnat vara en teoretiskt möjlig mekanism för dess uppkomst. I stället framställer M&W:s modell växthuseffekten som resultatet av en i översta troposfären radiativt uppkommen icke-linjär modifiering av den enligt Maxwell-Loschmidts gravito-termala teori förväntade konstanta troposfäriska temperaturgradienten med temperaturer som avtar linjärt med ökande höjd över jordytan ända upp till tropopausen.

Därmed kan den vetenskapliga kontroversen mellan Maxwell-skolan och GHE-skolan i M&W:s Nobelprisade tappning preciseras till att gälla den troposfäriska temperaturgradientens ursprung och observerade karakteristika. Är det troposfäriska temperaturavtagandet med ökande höjd över jordytan strikt linjärt som meteorologer normalt framställer det med sina på den gravito-termala effekten baserade deskriptiva samband (Loschmidts ekvationer)? Eller uppvisar temperaturavtagandet i troposfärens översta skikt en signifikant krökning som återspeglar hur mycket växthusgaser luften innehåller? Ett slutgiltigt svar på dessa frågor kan endast fastställas genom adekvat analys av relevanta empiriska observationer och några sådana analyser har jag ännu inte sett till.

Däremot kan jag peka på två för mig uppenbara teoretiska svagheter med M&W-modellen. För det första ställer jag mig helt avvisande till resonemanget om att en radiativ jämvikt bör kunna uppstå om radiativa effekter främjar ett förmodat stabilt tillstånd med varmare luft ovanför kallare. Varmare luft ovanför kallare luft utgör inte alls något stabilt tillstånd för den gravitationellt påverkade troposfären. Tvärtom har troposfären generellt sett befunnits vara varmast nära marken och ligga under kallare luft hela vägen upp till tropopausen på grund av den gravito-termala effekten, helt i enlighet med Loschmidts ekvationer och den av Maxwell föreslagna ”konvektiva jämvikten”.

Solen värmer jordytan som värmer den marknära luften och får den att stiga uppåt på grund av den gravito-termalt framkallade konvektionen. Konvektion i vertikal riktning uppstår nämligen när ett luftpakets temperatur avviker från den som svarar mot det ”konvektiva jämviktsläge” som bestäms av den gravito-termala effekten och beskrivs av Loschmidts ekvationer. Vad vi har god erfarenhet av är den tendens mot det konvektiva jämviktsläget som på grund av gravitationen får marknära uppvärmd luft att stiga uppåt, eftersom den blivit hetare och lättare än vad jämviktsläget föreskriver. Men tendensen mot jämviktsläget måste kvarstå oavsett om avvikelsen är åt det varma eller kalla hållet. Därför betvivlar jag starkt hållbarheten av M&W:s förmodande att strålningseffekters tendens att främja avvikelser åt det ena av hållen kan sätta konvektionen ur spel och leda till uppkomsten och bevarandet av en radiativ jämvikt med temperaturtillstånd som avviker från den konvektiva jämviktens.

För det andra leder M&W-modellens angivna förutsättningar och beräkningsutfall till en helt diskvalificerande självmotsägelse. Modellen föreskriver att en ökning av luftens koldioxidhalt från 300 ppm till 600 ppm enligt IPCC:s terminologi leder till en ”förstärkt växthuseffekt” som ökar jordytans temperatur med 2 °C. Samtidigt som modellen för lufthalten 300 ppm koldioxid de facto accepterar Maxwellskolans slutsats att den enligt IPCC i förindustriell tid föreliggande ”naturliga växthuseffekten” är av insignifikant storlek. GHE-proponenter har anammat modellutsagan rörande den förstärkta växthuseffekten och negligerat Maxwellskolans invändningar rörande den den naturliga växthuseffekten. Gissningsvis eftersom de inte förstått invändningarna eller betraktat dem som obefogade.

Men för mig framstår Maxwellskolans invändningar som fullt befogade och därmed högst värda att rekapitulera och förtydliga.  Utstrålningen av värme från Jorden sker enligt SMHI från luftskikt som befinner sig på 4–8 km:s höjd, vanligen medelvärdesutjämnat till ett luftskikt på cirka 5 km:s höjd med den gentemot solinstrålningen balanserande temperaturen -18 °C. Dessa eller detta skikt återfinns enligt Fig. 2 inom en region där troposfärens temperaturer enligt M&W-modellen helt och hållet bestäms av den gravito-termalt betingade ”konvektiva jämvikten”. Den 32-gradiga temperaturskillnaden mellan utstrålningsskiktet och jordytan framställs därför som en renodlat gravitationell effekt av såväl Maxwellskolan som M&W-modellen.

När jag påpekade detta i mitt KU-inlägg 2022-05-27 reste några läsare i sina kommentarer invändningen att den 32-gradiga temperaturskillnaden även måste återspegla en växthuseffekt hänförbar till radiativa processers inverkan på utstrålningsskiktens höjdmässiga belägenhet. Så kan inte vara fallet enligt Maxwells och Maxwell-skolans tänkande, vilket är empiriskt baserat och stöder sig på observationen att troposfärens temperatur avtar linjärt med ökande höjd över jordytan i enlighet med Loschsmidts ekvationer och gravito-termala teori. Skulle radiativa effekter ha någon signifikant inverkan på de uppkomna temperaturskillnaderna i troposfären, så skulle det leda till avvikelser från det observerade linjära temperaturavtagandet.

Denna centrala tanke i Maxwellskolans resonemang har enligt min erfarenhet av den vetenskapligt orienterade klimatdebatten varit bristfälligt förstådd av GHE-proponenter i gemen. Därför ska jag försöka förtydliga resonemanget. Radiativa processer bidrar förvisso till värmetransporten från jordens yta direkt eller via atmosfären till rymden. De tenderar förvisso att i det senare fallet åstadkomma troposfäriska temperaturförhållanden svarande mot en i princip beräkningsbar radiativ jämvikt, vars karakteristika starkt avviker från de som anges av Loschmidts ekvationer och t. ex. föreskriver att troposfärens temperaturer bör avta logaritmiskt i stället för linjärt med ökande höjd över jordytan.

Men vad Maxwellskolan framhåller är att de temperaturförhållanden som radiativa processer tenderar att åstadkomma aldrig kommer att uppnås, eftersom de ständigt ”suddas ut” av konvektionen. Den gravito-termala effekten sätter luftpaket i konvektiv rörelse så snart deras temperatur avviker från det av Loschmidts ekvationer beskrivna konvektiva jämviktsläget. Den gravito-termala effekten åstadkommer med andra ord en ständig omfördelning av de troposfäriska temperaturerna i riktning mot det konvektiva jämviktsläget. Någon radiativ jämvikt hinner aldrig uppstå eftersom värmetransport genom konvektion är snabbare än radiativ värmetransport. Så varför hålla på och presentera beräkningar av vad en radiativ jämvikt skulle medföra, om någon sådan aldrig uppnås?

När empiriska data säger att troposfärens temperaturer avtar linjärt med ökande höjd över jordytan i enlighet med Loschmidts ekvationer, så ger det belägg för att den gravito-termala effekten helt har överskuggat radiativa effekter. Det har uppstått en konvektiv dynamisk jämvikt med troposfäriska temperaturskillnader som tillfredsställande beskrivs av Maxwell-Loschmidt gravito-termala teori utan några som helst signifikanta inslag av radiativa effekter. Linjärt avtagande troposfäriska temperaturer är liktydigt med att temperaturgradienten i huvudsak uppkommit på grund av gravitationen och inte signifikant återspeglar några radiativa effekter. Empiriskt styrkt av att temperaturavtagandets ”lapse rate” för såväl ”torradiabaten” som ”våtadiabaten” befunnits ansluta sig väl till Loschmidts och andra ekvationer som framställer den som proportionell mot gravitationskonstanten och oberoende av troposfärens halt av växthusgaser och deras radiativa egenskaper.

M&W accepterar i 1967-års arbete klart utsagt denna innebörd av det av Maxwell introducerade begreppet ”konvektiv jämvikt”. Deras modell föreskriver enligt Fig. 2 att troposfäriska temperaturskillnader vid en koldioxidhalt av 300 ppm bestäms av en konvektiv jämvikt upp till 8 km:s höjd och framställer den vid högre koldioxidhalter uppkomna växthuseffekten som ett resultat av en på högre höjder föreliggande radiativ jämvikt.

Förtydligat är därför självmotsägelsen i M&W-modellen att den accepterar att den 32-gradiga temperaturskillnaden mellan utstrålningskiktet och jordytan för koldioxidhalten 300 ppm återspeglar en konvektiv jämvikt och därmed representerar en gravito-termal effekt utan radiativa inslag. Vilket innebär att den ”naturliga växthuseffekt” som enligt IPCC orsakas av troposfärens förindustriella halt av växthusgaser (cirka 10 000 ppm vattenånga och 300 ppm koldioxid) enligt modellen är av insignifikant storlek. Samtidigt som modellen föreskriver att en fördubbling av koldioxidhalten (till de troposfäriska växthusgashalterna 10 000 ppm vattenånga och 600 ppm koldioxid) ger upphov till en förstärkning av den insignifikanta ”naturliga växthuseffekten” svarande mot 2 °C.

Det är en fysikalisk och logisk orimlighet. Bidrar inte den förindustriella koldioxidhalten 300 ppm till uppkomsten av en signifikant ”naturlig växthuseffekt”, så lär inte heller en förhöjning av koldioxidhalten med ytterligare 300 ppm kunna ge upphov till någon signifikant växthuseffekt. Förenklat uttryckt av det simpla skälet att 2 x 0 = 0. Finns det ingen signifikant ”naturlig växthuseffekt”, så lär det inte heller finnas någon signifikant ”förstärkt växthuseffekt” inom det av M&W beaktade intervallet för luftens koldioxidhalt.

Då kan det vara dags att jämföra M&W-modellens beskrivning av troposfärens temperaturer med Maxwell-skolans. Den senare framställer de troposfäriska temperaturerna som linjärt avtagande med ökande höjd över jordytan ända upp till tropopausen. Den hänför detta till uppkomsten av en konvektiv jämvikt karakteriserad av Loschmidts ekvationer, vilka kvantitativt förmår beskriva det linjära temperaturavtagandets storlek (lapse rate) i gravito-termala termer som inte inkluderar några som helst bidrag från radiativa växthuseffekter. Den är konsistent med och ger en sedan länge accepterad förklaring till meteorologers empiriskt grundade beskrivning av situationen.

När det föreligger en sådan acceptabel beskrivning av vad som observerats sitter man vetenskapligt sett normalt nöjd. Man är medveten om och öppen för att beskrivningen kan komma att visa sig vara teoretiskt ohållbar eller stå strid med nytillkomna observationer. Vad det teoretiska beträffar har mig veterligt ännu ingen opponerat mot Loschmidts och Maxwellskolans beskrivning av den gravito-termala effekten och dess helt dominerande bidrag till uppkomsten av den troposfäriska temperaturgradienten.  Ej heller har någon mig veterligt presenterat empiriska data som befäster att det i övre troposfären föreligger avvikelser från det av Loschmidts ekvationer föreskrivna linjära temperaturavtagandet i den riktning och av den storlek som M&W-modellen föreskriver.

Så på vad sätt bidrar M&W-modellen till vår kunskap om temperaturförhållandena i troposfären? Modellen invaliderar på intet sätt Maxwellskolans beskrivning av situationen. I stället accepterar den och utgår från att det råder konvektiv jämvikt i troposfären upp till 8 km:s höjd. Därmed konfirmerar den Maxwellskolans slutsats att det inte finns någon ”naturlig växthuseffekt” av signifikant storlek.

I direkt motsägelse till detta föreskriver M&W-modellen att en ökning av av luftens koldioxidhalt från 300 ppm till 600 ppm ger upphov till en signifikant ”förstärkt växthuseffekt”. Detta på grund av det teoretiskt dubiösa och empiriskt obefästa antagandet att radiativa effekter på troposfäriska höjder över 8 km överflyglar den gravito-termala effekten och leder till uppkomsten av en radiativ jämvikt. Tar man bort detta dubiösa antagande, så hamnar man i Maxwellskolans empiriskt stödda beskrivning av situationen.

För att ytterligare belysa hur dubiöst och självmotsägande M&W-antagandet är ska jag återvända till beräkningsresultaten i Fig. 2. Enligt dessa anser M&W att det för 300 ppm koldioxid råder en konvektiv jämvikt upp till 8 km över jordytan. För 600 ppm koldioxid anses konvektiv jämvikt råda upp till 9 km över jordytan. Ökande halt av koldioxid anses alltså utöka de troposfäriska regioner där den gravito-termala effekten dominerar och minska de regioner där radiativa effekter antas dominera och leda till en ”förstärkt växthuseffekt”.

Den enligt M&W-modellen föreslagna förstärkta växthuseffekten är med andra ord självutplånande. Ju högre halt man har av växthusgasen koldioxid, desto säkrare blir det enligt modellen att det inte kan existera någon ”naturlig växthuseffekt” av signifikant styrka. Samt att utrymmet för föreslagna radiativa ”förstärkningar” av den insignifikanta naturliga växthuseffekten ständigt minskar. Ju mer koldioxidhalten överstiger 600 ppm, desto mer tenderar M&W-modellen att konfirmera Maxwellskolans syn att det råder konvektiv jämvikt ända från jordytan upp till tropopausen och därmed vare sig kan föreligga någon ”naturlig” eller ”förstärkt” växthuseffekt av signifikant storlek.

Varav man i politiskt avseende kan dra slutsatsen att alla alarmistiska varningar om växthuseffekter och den på grund av sådana påstått uppkomna risken för att vår användning av fossila bränslen kan leda till en potentiellt katastrofal global uppvärmning representerar obefogad räddhåga förorsakad av politisk skrämselpropaganda utan vetenskapligt fog.

”Much ado about nothing”, som Shakespeare uttryckte det. Mycket väsen för ingenting. Enligt min mening rent otroligt mycket vetenskapligt obefogat politiskt väsen.

Jag är övertygad om att det existerar radiativa växthuseffekter i IPCC:s mening, men tillåter mig att starkt betvivla att de kan vara av signifikant storlek. De av 2021-års Nobelpristagare Manabe presenterade modellstudierna befäster mitt tvivel. Jag är fullt beredd att acceptera tanken att växthuseffekter är av signifikant storlek så snart jag påträffar hållbara teoretiska belägg för att så skulle kunna vara fallet och observationsmässiga belägg för att så faktiskt även tycks vara fallet. Några sådana belägg har jag ännu inte påträffat. Må de som tycker annorlunda anföra vilka belägg de stöder sin avvikande syn på. Jag är beredd att lyssna, begrunda och anamma eller kritisera.

Kommentarer

Kommentera längst ner på sidan.

  1. Staffan Granström

    Detta påminner mig om mitt inlägg i KU 2021-06-03 ”Ny forskning ifrågasätter om koldioxidpåverkar temperaturen. ”Ny forskning visar att halten av koldioxid i atmosfären inte påverkar temperaturen”.

    De irländska forskarna Michael och Ronan Connolly har under perioden 2010 – 2011 analyserat mätdata från milliontals väderballonger som sänts upp i atmosfären till 35 km höjd (Stratosfären) från norra Kanada (Norman Wells) till Södra USA (Lake Charles i Arizona). En väderballong når en höjd av ca 35 km innan de förstörs (sprängs) av det låga omgivningstrycket. Ballongen hinner passera de 2 luftlagren troposfären och stratosfären med gränsskiktet tropopausen mellan dem.

    Uppmätta data i atmosfären såsom, lufttryck, lufttemperatur, luftfuktighet, höjd har analyserats med hjälp av de allmänna gaslagarna som beskriver atmosfärens fysikaliska egenskaper, dvs D = n/V = p/(RT) där D är molekyltätheten, antal gasmolekyler per volymenhet.

    De har analyserat molekyltätheten i atmosfärens olika luftlager och funnit att atmosfären är i fullständig termodynamisk energibalans och att inverkan av växthusgasen koldioxid på atmosfärens temperatur är noll.

    De har också visat hur ozonlagret, bestående av O3 molekyler bildas och förnyas genom multimerisation av syrgas och/eller kvävgas molekyler i atmosfären ovanför troposfären.

    Den uppmätta temperaturen i atmosfären visar ingen ”temperaturfälla” med ökad medeltemperatur i troposfären utan en jämnt avtagande temperatur upptill ca 16 km höjd där temperaturen stiger igen efter en fasförändring.

    Mer information om Michael and Ronan Connollys forskning kan se i deras presentation i Balloons
    och deras artikel i GlobalWarmingSolved.

  2. Mycket bra sakligt inlägg, som visar att Nobelkommittén i Fysik har liten förståelse för även elementär klasssisk fysik. Tilläggas kan att koncentrationen av CO2 är mycket liten och att någon ”återstrålning” eller ”back radiation” inte kan finnas, då det i likhet med ”baklänges värmeledning” är ett instabilt fenomen och som sådant inte kan bestå över tid. Mer om detta finns på TNT Radio enligt https://claesjohnson.blogspot.com/2022/11/interview-on-tnt-radio-corruption-of.html där också länkar till diskussion i ärendet med Will Happer finns.

    Det finns mycket om klimatet på TNT Radio och på Principia Scientific https://principia-scientific.com väl värt att ta del av!

  3. Lars Cornell

    Tack för intressanta synpunkter. Som amatör måste jag verkligen anstränga mig för att förstå. Det är några saker som jag hänger upp mig på – och kanske inte förstår.
    • 90% av solens strålar (=energi) går ned i haven. Uttrycket ”med ökande höjd över jordytan” borde då vara ”med ökande höjd över vattenytan” eftersom den dominerar.
    • Den största mängden energi kommer således i form av vattenånga från haven. Vattenånga är mycket lättare än luft, vid samma temperatur, vilket förstärker konvektionen. Hur mycket kan det påverka? Försumbart?
    • Utstrålning sker även från stratosfären med omvänd laps rate. Den sammanlagda utstrålningen är således en summering av de två. När vi har så mycket koldioxid att medelpunkten flyttats upp i tropopausen kommer vi att få omvänd växthuseffekt från stratosfären (som i Antarktis). Jag är därför mycket missnöjd med att man bara utgår från en sammanvägd punkt på ca 8 km höjd. För att det skall bli rätt måste vi utgå från två sådana punkter, en i troposfären och en i stratosfären och väga samman de två.

  4. Fredrik V

    #3 Lars Cornell

    Jag är ännu mer amatör än du, men vad gäller din första punkt har jag nog svaret. ”Jordytan” avser jordens=jordklotets totala yta, så uttrycket i texten är korrekt. Det du tänker på, och blandar ihop det med är kanske ”landytan”?

  5. Tack för detta Gösta. Enligft dig så har växthusgaserna noll, eller ”insignifikant” effekt.

    Men då bör du även kunna förklara

    1. Varför strålningsfysiken, inklusive kvantmekaniken, är fel. Strålningen borde i annat fall åtminstone ge NÅGON effekt.

    2. Ge en förklaring till varför vi haft en global värmeökning – och klimatförändringar i allmänhet? Det är ju trots allt detta som hela klimatfrågan handlar om.

  6. Peter+Stilbs

    Jag menar att nyckeln till förståelsen här är att M&L & lapse rate beskriver hur energin (värme & potentiell) fördelar sig genomsnittligt i vertikalled i atmosfären – medan lapse rate etc inte säger något som helst om hur energin ursprungligen tillförts atmosfären.

  7. tty

    Det finns ingen motsättning mellan den ”gravitotermala” och växthuseffekten. Med mera växthusgaser ökar den höjd där utstrålningen av den konvekterade värmen sker.
    Resultatet blir (allt annat lika) en ”parallellförflyttning” av temperaturkurvan mot högre temperaturer och en höjning av tropopausen.

    Men p g a att det finns vattenånga i atmosfären är INTE allt annat lika. Högre temperatur innebär mera avdunstning och mera vattenånga (den berömda vattenångeåterkopplingen). Men det är mycket mera komplicerat än så. Mera vattenånga ökar luftens specifika värme. Lapse-raten minskar och temperaturen sjunker långsammare med höjden, temperaturkurvan blir ”brantare” och temperaturhöjningen blir proprtionellt större på högre höjd (den berömda troposfäriska ”hot-spoten” som är så svår att hitta). Konvektionen blir effektivare genom att luftens värmetransportförmåga ökar.

    Men komplikationerna har bara börjat. Vattenånga är en KONDENSERANDE växthusgas. När den konvekteras uppåt och temperaturen sjunker kondenseras den till vatten och/eller is. Och vatten och is är inte växthusgaser utan absorberar och emitterar i ett kontinuumspektrum (tidigare skulle jag ha sagt att de är svartkroppsstrålare, för skillnaden är försumbar, men vissa här tar tydligen illa upp om jag använder det ordet).

    Dessutom bildar vattenångan moln vars effekt på temperaturen varierar starkt beroende på molntyp, molnhöjd, temperatur, geografi och tid på dygnet.
    För att krångla till det ytterligare är vattenångan i motsats till koldioxid inte alls jämt fördelad. För det första finns det mycket litet vattenånga på hög höjd eftersom den mesta kondenseras på lägre höjd. Dessutom varierar avdunstningen enormt geografiskt. Den mesta sker över tropiska hav och vattnet transporteras sedan omkring i atmosfären. Varvid det kan falla ner som regn eller snö, och växla mellan gas/vätska/fast form upprepade gånger.

    Vad Manabe fick nobelpriset för var i stort sett första stycket ovan. Han gjorde också beräkningar på två förenklade typfall med varierande fukthalt i luften beroende på temperaturen (konstant relativ fuktighet och konstant absolut fuktighet), men utan moln. Längre än så kunde han inte komma med den beräkningskapacitet som fanns då, och det gäller i praktiken fortfarande, även om man inte medger det.

  8. Ingemar #5: Den vetenskapliga frågan är om det finns någon evidens (observation + teori) som visar att en liten mängd koldioxid i atmosfären kan märkbart påverka jordens medeltemperatur? Nuvarande svar på denna fråga är att det inte finns några observationer som tyder på något sådant. Inga! Vad gäller teori så finns en överslagsberäkning av enklaste heuristiska slag som ger +1 C vid fördubbling av CO2 från förindustriell tid, och det blir inte mer vetenskaplig tyngd av att dra in Schwarzschild’s ofysikaliska bakåfram ekvationer för värmeöverföring med strålning. Det finns alltså ingen vetenskap som ger stöd åt att CO2 märkbart styr jordens klimat. Ingen. Detta är vad klimatfrågan handlar om. Inte Dina frågor 1 och 2.

  9. Gösta Pettersson

    #3 Lars Cornell

    Vattenånga från haven har en påvisad stark effekt på troposfärens temperaturer. Det är på grund av vattenångan som man skiljer mellan en ”torradiabat” och en ”våtadiabat” för temperaturavtagandets lapse rate.

    Som jag angav i mitt KU-inlägg 2022-05-27 anser SMHI att Jordens värmeutstrålning huvudsakligen sker från luftskikt på 4–8 km:s höjd, vanligen medelvärdesutjämnat till ett luftskikt på 5 km:s höjd med den balansskapande temperaturen -18 °C. Jag utgår från att de har fog för vad de säger, dvs att utstrålningen från stratosfären är av ringa betydelse i detta sammanhang.

  10. Lasse

    #9 Gösta P
    Läser och försöker förstå.
    5 km höjd har ett halverat lufttryck.
    Såhär ser det ut där uppe :
    https://earth.nullschool.net/#current/wind/isobaric/500hPa/overlay=temp/orthographic=-17.48,80.56,721/loc=-126.618,58.982
    Temperaturen varierar enl denna en hel del.
    Går man över till relativ fuktighet (RH)så blir det ännu mer variationer.
    Det finns en del väder där uppe!

  11. Gösta Pettersson

    #5 Ingemar Nordin

    1. Jag hittar ingenting i mina inlägg som berättigar slutsatsen att strålningsfysiken, inklusive kvantmekaniken, är fel. Därför att jag ingenstädes påstår att strålningen inte kan ha NÅGON effekt. Insignifikant är inte samma sak som noll.

    2. Vetenskapliga framsteg görs genom att förkasta hypoteser. Vad jag uppmärksammat och gett mitt stöd åt är Maxwellskolans förkastande av IPCC:s hypotes att den 32-gradiga atmosfärseffekten utgör en ”naturlig” växthuseffekt, alldenstund atmosfärseffekten kvantitativt tillfredsställande kunnat förklaras utgöra en gravito-termal effekt.

    Vilket i förlängningen innebär att man bör söka andra förklaringar till global uppvärmning än växthuseffekter. Vad jag beskriver är de belägg som berättigar ett starkt ifrågasättande av den för närvarande mest omhuldade förklaringen till global uppvärmning under industriell tid. Det kan vara nyttigt att göra, även om man inte kan hänvisa till något annat än astronomiska faktorer som allmän förklaring till de temperaturvariationer som förelegat i förindustriell tid och rimligen även bidragit till temperaturvariationerna under industriell tid.

  12. Lars Cornell

    #11 Gösta Pettersson, ”astronomiska faktorer”
    Hur ser du på jordens klimat som ett internt självsvängande system? Vi har LaNina på ca 5 år, NAO ca 60 år och djuphavens tidsfördröjning med ca 1000 år som exempel?
    Vi måste även lyssna på jordens egen musik.

  13. tty

    #11

    Den ”gravito-termala” effekten säger bara hur mycket temperaturen sjunker med höjden vid jämvikt, men ingenting om hur hög den är. Den är alltså ca 30 grader högre än den jämviktstemperatur man får enligt Stefan-Boltzmanns lag. Sedan kan ju just den där siffran 33 grader diskuteras eftersom den innebär att man förutsätter att Jordens albedo vore detsamma utan växthusgaser.

  14. tty

    #12

    Klimatet är förvisso självsvängande, men det verkar inte finnas någon tydlig cyklicitet med ett intervall större än ett år och mindre än den kortaste Milankovich-cykeln om 21 000 år.

    Det finns många periodiska fenomen, ja, ENSO, IOD, NAO, PDO, Dansgaard-Oeschger events, Heinrich events, Agulhas leakage, Grand solar minima m fl, men inga med fast förutsägbar cyklicitet.
    Solfläckcykeln kommer väl närmast, men även den varierar mellan 9 och 14 år.

    Det är bara att konstatera att enligt vårt nuvarande vetande är klimatförändringar inte förutsägbara.
    Vi kan säga att efter sommar kommer vinter, och efter mellanistid kommer istid, men det är allt.

  15. #5 Ingemar
    ”Ge en förklaring till varför vi haft en global värmeökning”

    Menar du just denna vi befinner oss i nu eller alla andra tillbaka i tiden?

  16. Björn

    Lars Cornell [12]; Nej, jorden är inte ett självsvängande system, eller med andra ord, en perpetuum mobile. Utan tillförsel av energi från solen svalnar jorden tveklöst. La Nina och NAO etc finns där av strukturella skäl, men drivkraften till dessa företeelser beror av solenergi. Men trögheten i vädersystemen beror på ackumulerad värme i haven. Men vad CO2 i det hela står för är tveksamt, annat än att den avsvalning som nu är kommande på grund av en avtagande solaktivitet, kanske fördröjs något på grund av atmosfärens absorption.

  17. Gösta Pettersson

    #7 tty
    ”Det finns ingen motsättning mellan den ”gravitotermala” och växthuseffekten.”

    Jo, det gör det enligt Maxwellskolan. Skulle radiativa processer ha någon signifikant inverkan på troposfärens temperaturer, så skulle det inte kunna uppstå någon temperaturgradient à la Loschmidt med linjärt avtagande temperaturer i vertikal riktning och med en lapse rate som är proportionell mot gravitationskonstanten. Så länge temperaturavtagandet är linjärt anses ”konvektiv jämvikt” råda och den innebär att observerade temperaturskillnader är av gravitationellt ursprung utan signifikanta inslag av radiativa processer. (Speciellt även den 32-gradiga skillnaden mellan utstrålningskiktets och jordytans temperatur).

    Manabe ansluter sig till den uppfattningen av begreppet ”konvektiv jämvikt”. Hans parallellförflyttning av temperaturkurvorna hänför sig inte till någon CO2-beroende förändring av utstrålningsskiktets belägenhet, utan till antagandet att det uppstår en radiativ jämvikt på 8–9 km:s höjd. Vlket leder till att temperaturkurvorna i den regionen kröks och åstadkommer den CO2-beroende parallellförskjutningen.

    Du har alltså en annan syn på växthuseffektens uppkomst än Nobelpristagaren Manabe. Enligt dig leder ökande CO2 till en förhöjning av utstrålningsskiktets läge och därav föranledd parallellförskjutning av temperaturkurvorna. Vad har du för skäl att tro det? Finns det empiriska belägg för att utstrålningsskiktets belägenhet förhöjts under industriell tid på grund av ökningen av luftens CO2-halt? Enligt vilka teoretiska mekanimer skulle den tänkta förhöjningen åstadkommas? Och om det ska medföra en parallellförskjutning av kurvorna, så lär det även innebära att tropopausens höjd eller temperatur förändrats under industriell tid. Finns det några belägg för det och att det kan hänföras till ökningen av CO2-halten?

    Utan empirska eller teoretiska belägg framstår dina tankar än så länge som en ren spekulation. Återkommer eventuellt senare beträffande detta, för det finns ännu en för mig intressant aspekt att beakta i sammanhanget.

  18. Mikael Andersson

    #14 tty

    Jag finner detta från illustrerad vetenskap:

    ”Beräkningar visar att vinkeln mellan jordens axel och den axel som står vinkelrätt mot jordens bana runt solen kan variera mellan ytterpunkterna 21,8 och 24,4 grader. Ändringen i jordens lutning är periodisk, och det tar cirka 21500 år från det att lutningen är som minst tills den är som störst. Den nuvarande lutningen är ungefär 23,5 grader och avtagande. Solinstrålningen på olika platser på jorden ändras, när jordens lutning i förhållande till banan runt solen förändras, och det leder till klimatförändringar globalt och lokalt. En liten lutning betyder mindre skillnad mellan årstiderna, medan en stor lutning ger större skillnader mellan årstiderna och därmed också mer dramatiska väderförhållanden.

    Jordens bana runt solen förändras från att vara oval till nästan rund i cykler på 100 000 år. För närvarande är den nästan rund vilket innebär att skillnaden i solinstrålning är liten under året. Dessutom varierar jordaxelns lutning vilket påverkar klimatet. När effekterna av dessa faktorer sammanfaller blir det ett kyligare klimat.”

    Är detta Milankovich-cykeln som du nämner?

  19. Gösta Pettersson

    #12 Lars Cornell

    Jo, astronomiska faktorer var kanske onödigt begränsande. Det finns förvisso även jordisk musik med mer eller mindre långvariga effekter på temperaturer.

    Men jag tror Ingemar förstod andemeningen i mitt svar: Det kan vara värdefullt att rensa bort ohållbara förklaringar till global uppvärmning, även om man inte är beredd att förorda några alternativa (av ett otal föreliggande) förklaringar till genom tiderna observerade temperaturvariationer.

  20. leg

    Inlägget och kommentarerna visar med önskvärd tydlighet hur komplext jorden klimat är.

  21. Jag har nyss vaknat upp från en höftledsoperation så jag är kanske lite virrig, men för mig handlar den vetenskapliga fråga om både och. Dvs både de historiska klimatförändringarna (alltifrån istider till interglacialer) till variationen inom interglacialerna, dvs vad som orsakar värmeperioder och Lilla Istiden

    Man får väl anta att atmosfärens volym är ganska konstant sedan många miljoner år. Så blir Göstas och Claes svar lite vid sidan om huvudfrågan.

  22. JonasW

    #Claes Johnson

    ”Schwarzschild’s ofysikaliska bakåfram ekvationer för värmeöverföring med strålning”

    Blev lite nyfiken. Jag kan hålla med om att det finns något konstigt med Schwartzschild´s ekvation. Å andra sidan så lyckas Happer&Wiijngaarden beskriva jordens utstrålning (nästan) perfekt med denna ekvation.

    Skulle gärna vilja förstå vad som är bristen (felet?) med Schwartzschild´s ekvation? Vad är din uppfattning?

  23. JonasW

    #18 Mikael Andersson

    Man kan räkna ut hur den instrålade effekten beror av jordens bana. Lite tvärsemot vad som sägs så är den instrålade effekten (Energi/varvtid) störst när eccentriciteten är störst.

    En cirkulär bana ger alltså lägre instrålning.

    Det är ganska rakt på beräkningar bakom detta.

  24. JonasW #22
    Hur vet man/Du att Happer lyckas beskriva jordens utstrålning nästan perfekt? Felet med Schwarzschild’s ekv (S) är att de bygger på tvåvägs värmeöverföring mellan kroppar av olika temperatur förmedlad av s k ”fotoner” som i massiva strömmar av små energipaket antas fara åt båda håll och då innebär att inte bara en varm kropp värmer en kall, utan även att en kall kropp värmer en varm (med s k ”återstrålning”), medan det bara är nettot som de facto överförs. Men en sådan dubbelvägs massiv värmeöverföring fram och tillbaka är instabil likaväl som baklänges värmeledning, och kan då inte bestå över tid. Spec innehåller (S) absorption även för återstrålning, vilken alltså inte kan finnas. En modell för envägs radiativ värmeöverföring från varm till kall finns beskriven på https://computationalblackbody.wordpress.com
    Fotoner som små energipaket farande framåtillbaka finns bara i fantasin. Radiativ värmeöverföring är ett elektromagnetiskt resonansfenomen med cut-off av höga frekvenser som ökar med temperaturen och som därmed ger en riktning från varm till kall, enligt min mening i alla fall. Vad tror Du?

  25. JonasW

    #Claes Johnson

    ” Spec innehåller (S) absorption även för återstrålning”

    ja, jag har problem med detta också. Effekten blir att strålningen är självgenererande, och att källan inte spelar så stor roll.

    Jag gillar inte Schwarzschild´s ekvationer, men jag kan inte i sak bemöta dessa.

    I övrigt håller jag helt med dig. Återstrålning finns inte. Däremot finns minskad utstrålning. I mina ögon är återstrålning ett helt ofysikaliskt begrepp. Som du skriver – ”baklänges värmeledning” – det blir nonsens. Å andra sidan verkar många tycka att det är ett naturligt begrepp, och jag orkar inte argumentera med alla dessa människor.

    Beträffande Happer&Wiijngaarden så redovisar de ett emissionsspektrum från jorden. Det stämmer mycket bra med det som registrerats från Voyager satelliter. Är lite tveksam till mätningarna från Voyager, men ändå. Tyder på att man får rätt resultat när man använder Schwarzschild’s ekvationer.

  26. Mikael Andersson

    #JonasW,23

    Jag är varken fysiker eller matematiker men föreställer mig att just den differensen i solinstrålning i W/m2 mellan excentrisk och cirkulär bana inklusive jordaxelns lutning påverkar huruvida vi har istid eller inte?
    Den klassiska vad som är ”hönan eller ägget”? Växthusgaserna finns ju här och vi med dem Co2 likaså, huruvida dessa ökar eller minskar är ju en konsekvens av mängden energi i form av solinstrålning som planeten utsätts för, jag menar, Venus och Mars visar väl detta tydligt. (En tanke)

  27. tty

    #18

    ”Är detta Milankovich-cykeln som du nämner?”

    Två av dem. Det finns egentligen fyra (eller fem):

    1. Excentricitetscykeln, alltså hur avlång (elliptisk) jordbanan är. Den varierar med en cykel om ca 400 000 år, men det finns även cykler om 95 000 och 125 000 år. Dessa samverkar på ett komplicerat sätt,, som man förenklat brukar beskriva som en cykel om 100 000 år. Detta är den enda av Milankovich-cyklerna som påverkar den totala solinstrålningen, om också inte så mycket. De övriga påverkar bara solstrålningens fördelning geografiskt och under året.

    2. Oblikvitetscykeln, d v s hur mycket jordaxeln lutar i förhållande till banplanet. Den är 41 000 år lång, och när den varierar så förflyttas polcirklarna och vändkretsarna. Vid stor lutning ligger vändkretsarna längre från ekvatorn och polcirklarna längre från polerna.

    3. Precessionscykeln. Det är inte bara jordaxelns lutning som ändras utan ockå dess riktning rör sig runt i en cirkel. Just nu ligger polstjärnan ungefär i axelriktningen. Men om 13 000 år kommer den att ligga nästan 45 grader från axelriktningen, men om 25700 år är axelriktningen tillbaka på samma plats (fast polstjärnan kommer inte att vara kvar där, men det är en annan historia)

    4. Absidernas precession. Jorden rör sig i en ellips runt solen, men den ellipsens längdriktning rör sig runt solen med en period av 112 000 år.

    5. Jordbanans lutning i förhållande till solsystemets plan (i stort sett = jupiters banplan) varierar med en cykel av ca 100 000 år. Denna hörde inte till de ”ursprungliga” Milankovich-cyklerna och det är oklart om den verkligen har någon effekt på klimatet.

    Nedan en bild som illustrerar de olika cyklerna.
    Blå kurva = oblikvitet
    Grön kurva = excentricitet
    Violett kurva = precession
    Röd kurva = excentricitet + precession
    Svart kurva = den samlade effekten på solinstrålningen på 65 breddgraden av alla kurvorna

    De båda undre kurvorna återger istidernas förlopp. Som synes inträffar mellanistider bara vid maximal instrålning på höga breddgrader.

    https://en.wikipedia.org/wiki/Milankovitch_cycles#/media/File:MilankovitchCyclesOrbitandCores.png

    Det är synd att kurvorna bara går 600 000 år tillbaka, för tidigare (mer än en million år sedan) var effekterna mycket tydligare. Då var det oblikviteten som styrde helt och istidscyklerna var 41 000 år.

    Bilden nedan visar förändringen för ca 1 miljon år sedan (Mid Pleistocene Transition, MPT). Den undre kurvan visar förresten koldioxidhalten. Korrelationen är som synes inte direkt uppenbar.

    https://www.researchgate.net/profile/Clara-Bolton/publication/235631156/figure/fig1/AS:340489103527938@1458190336483/Plio-Pleistocene-climate-over-the-past-5-my-a-LR04-benthic-d-18-O-stack-Lisiecki-and.png

    Varför ändrades då effekten av de olika Milankovich-kurvorna på klimatet? Ingen vet, det finns många teorier, men ingen helt övertygande.

  28. JonasW

    Hej Mikael,

    Ska inte påstå att jag vet heller.

    Det jag vet är att jorden får mer energi från solen när banan är elliptisk. Det är inte det som brukar påstås.

    Energiinflödet vid en elliptisk bana blir mer oregelbundet, och det kan säkert vara så att vintern blir kallare – mer is- mer reflektion.

    Jag är helt övertygad om att jordens bana påverkar vårt klimat. Däremot vet jag inte åt vilket håll. Blir det istid när jordens bana är mer elliptisk? Inte helt uppenbart för mig.

    Totala värmeinstrålningen ökar när banan blir mer elliptisk. Motsatsen påstås ofta.

  29. Mikael Andersson

    tty #27
    Tack för ett extremt uttömmande svar! Nu får jag fundera några dagar på det, kul!

  30. tty

    #25

    ”I övrigt håller jag helt med dig. Återstrålning finns inte. Däremot finns minskad utstrålning. I mina ögon är återstrålning ett helt ofysikaliskt begrepp. Som du skriver – ”baklänges värmeledning” – det blir nonsens. Å andra sidan verkar många tycka att det är ett naturligt begrepp, och jag orkar inte argumentera med alla dessa människor.”

    Det ÄR ett naturligt begrepp. Om återstrålning inte funnes skulle det kräva någon mekanism som hindrar gaser i atmosfären (och moln) att utsända strålning nedåt. Att sedan denna nedåtriktade strålning nästan alltid är mindre än den uppåtriktade från marken är en annan sak.

  31. Mikael Andersson

    JonasW #28

    Tack för svar Jonas! Om man nu har en ”klimatpanel” med de vassaste kunskaperna om planetens klimat, hur stor del i AR1-6 behandlar Milankovich-cyklerna i procent?
    Jag bara undrar, verkar ju lite ynkligt att koncentrera sig på några ppm antropogent Co2 som orsak för att plöja ner hundratals miljarder i ”klimatåtgärder” och straffskatter…men men, det ger väl arbetstillfällen!

  32. JonasW

    #30

    Undrar igen vad du egentligen har för några meriter inom fysik. Du vet en massa saker som inte stämmer med något jag lärt mig.

    Ja gskulle vilja att du svarar på din bakgrund inom fysik. Jag kan absolut förstå det som Claes Johnson skriver. Du påstår saker som man ska acceptera eftersom det är du som skriver det.

    Var i fysiken hittar du att återstrålning ÄR ett naturligt begrepp? Vilka fysikaliska teorier stödjer du dig på?
    Svara med teorier/modeller, och inte ditt personliga tyckande.

    Det du anför som argument är blaj. Jag skrev att utstrålning kan minska, men att återstrålning inte kan öka.

    En gissning är att dina fotonteorier kommer in igen,. Fotoner som studsar fram och tillbaka, och några fler studsar tillbaka.

  33. Mikael Andersson

    Jag vill be Gösta om ursäkt, jag kan givetvis hålla mig i den allmänna tråden med mina bryderier. Förlåt Gösta!

  34. JonasW

    #Gösta A

    Håller med Mikael. En mycket välskriven och intressant artikel.

    Det jag undrar lite över är det atmosfäriska fönstret. Där sker inte utstrålningen från 5 km höjd. Hur stor betydelse har det atmosfäriska fönstret för jordens energibalans?

    Försummbart?

  35. tty

    #34

    ”Försummbart?”

    Ingalunda! Utstrålningen genom ”fönstret” är ca 40 Wm^-2, att jämföra med ca 100 Wm^2 via konvektion.

  36. tty

    ”Var i fysiken hittar du att återstrålning ÄR ett naturligt begrepp? Vilka fysikaliska teorier stödjer du dig på?
    Svara med teorier/modeller, och inte ditt personliga tyckande.”

    Om återstrålning inte finns förutsätter det t ex att alla moln har en temperatur av 0 K. Eller har du någon teori/modell om något annat som skulle hindra dem från att avge elektromagnetisk strålning nedåt?

  37. Gösta Pettersson

    #21 Ingemar Nordin

    ”Så blir Göstas och Claes svar lite vid sidan om huvudfrågan.”

    Du tycker att jag borde ge en förklaring till vad som orsakat temperaturförändringar i största allmänhet, vilket du betraktar som den klimatologiska huvudfrågan. Förvisso intressanta problem. Men mitt inlägg handlar, som rubriken anger, om växthuseffekten. Det är väl snarare dina kommentarer som faller lite vid sidan av den huvudfråga som behandlas i inlägget.

    Hoppas din höftledsoperation leder till önskat resultat och komplikationsfri återhämtning.

  38. Matsa

    #24 Claes

    Ånej, inte detta svammel igen: ”Fotoner som små energipaket farande framåtillbaka finns bara i fantasin. Radiativ värmeöverföring är ett elektromagnetiskt resonansfenomen med cut-off av höga frekvenser som ökar med temperaturen och som därmed ger en riktning från varm till kall, enligt min mening i alla fall.”

    Har du hört talas om universums bakgrundsstrålning? Den strålar från alla håll i universum och stämmer mycket bra med Plancks strålningslag för en svartkropp med temperaturen ca. 3 K. Vi kan alltså mäta strålningen från en 3 K källa trots att temperaturen här på jorden är MYCKET högre.

    https://sv.wikipedia.org/wiki/Kosmisk_bakgrundsstrålning

    Hemmasnickrade fysikteorier i all ära, men det här är inte rätt forum att torgföra dem.

  39. Lars-Eric Bjerke

    I en kommentar till ett tidigare inlägg av Gösta Pettersson modellerade Johan M temperaturfördelningen i en hög gasfylld termos stående på jordytan. Han fann att temperaturen i termosen med tiden blev konstant i enlighet med termodynamikens lagar d.v.s. att värme alltid går från de varmare molekylerna till de kallare. Gösta Pettersson påstod att så inte är fallet för en ideal gas.

  40. Gösta Pettersson

    #33 Mikael Andersson

    Jag kan inte hitta något i dina kommentarer som du har anledning att be om ursäkt för. Dina frågor rör temperaturpåverkande faktorer, och jag är glad att det finns läsare som försöker ge dig svar.

  41. Claes Johnson

    Matsa #38

    Man kan mäta temperaturen av ett objekt med ett instrument som är varmare, men inte återstrålning eftersom den inte finns. Temperatur är lätt att mäta, strålning svårare, återstrålning omöjligt.

  42. Gösta Pettersson

    #39 Lars-Eric Bjerke

    ”Gösta Pettersson påstod att så inte är fallet för en ideal gas.”

    Du måste gravt ha missförstått meningsutbytet mellan Johan M och mig.

    För det första har jag aldrig någonsin i klimatdebatten funnit anledning att nämna begreppet ideal gas.

    För det andra har jag alltid betonat att värme enligt termodynamikens andra huvudsats endast kan överföras från varmare materia till kallare (bl a för att kritisera intuitiva tankar som att det uppstår en växthuseffekt genom att atmosfären uppfångar från jordytan utsänd värme, som sedan återsänds till och värmer jordytan; det är en termodynamisk omöjlighet att värme skulle kunna överföras från en kallare atmosfär till en varmare jordyta).

    Vad Johan M sa var att han betvivlade att det fanns någon gravito-termal effekt eftersom han inte tyckte sig ha kunnat upptäcka någon sådan vid enkla modellstudier. Vad jag bl a genmälde var att hans modellresultat snarare tycktes mig tyda på att det förelåg en stark gravito-termal effekt.

    Vad det vetenskapliga kunskapläget beträffar, så befästes det redan på 1800-talet att under förhållanden där den gravito-termala effekten är försumbart liten (t. ex. för gaser inneslutna i en termos) så karakteriseras den termisk jämvikten av isotermi (gasen antar samma temperatur överalllt i flaskan).

    Och under tidigt 1900-tal genomförde man med statistisk-mekanisk metodik betydligt mera avancerade studier än de Johan M ägnade sig åt. Studier som bekräftade att atmosfären på grund av den gravito-termala effekten uppnår ett jämviktstillstånd karakteriserat av en temperaturgradient förenlig med Loschmidts ekvationer.

    Så jag har lite svårt att förstå vad du vill ha sagt i din kommentar.

  43. Gösta Pettersson

    #32 JonasW (apropå #30 tty)

    Jag upptäckte så här på kvällskvisten att du haft ett meningsutbyte med tty rörande begreppet ”återstrålning” och hans idéer om ”fotoner som studsar fram och tillbaka”.

    Nu är det så att jag som avdankad bio(fysikal)kemist gärna betraktar strålning som ett partikulärt fenomen återspeglande utsändningen av fotoner. Om Jorden skickar ut värmestrålning i form av en foton och den uppfångas av och exiterar en växthusgasmolekyl, så är den i min föreställningsvärld frekventaste följden att den exiterade molekylen efter cirka 50 ns relaxeras genom oriktad återutsändning av fotonen. Varvid det av geometriska skäl är vid pass 50% sannolikhet att fotonen återförpassas till jordytan.

    Jag har inga problem med att betrakta detta som en återstrålning av värmeenergin. Men vad jordytans temperatur beträffar representerar sådan återstrålning basalt en energetisk nolleffekt. Jorden försökte avsvalka sig genom att skicka ut en värmefoton. I den mån fotonen återvände på grund av återstrålning, så misslyckades försöket. Det blidde ingen avkylning. Ej heller blidde det någon uppvärmning. Det blidde ingenting alls vad jordytans temperatur beträffar. Den utskickade fotonen (-1 energikvantum) återvände (+1 kvantum) och återställde det energetiska utgångstillståndet (-1 +1 = 0). Jorden kan inte värma sig själv genom utstrålning av värme som återstrålas. Inte ens om atmosfären innehölle 95% CO2, som den gissningsvis gjorde en gång i tidernas begynnelse.

    Likväl kan återstrålning förväntas indirekt ge upphov till en kinetiskt betingad uppvärmning av jordytan. Antag att de värmefotoner som utsänds från jordytan genomsnittligt absorberas på 5 km:s höjd och till 50% återstrålas till jordytan. Då tar det 33 mikrosekunder för en utsänd och återstrålad foton att återvända till jordytan. Ett tidmått som jag med insikt om att summan av den oändliga serien 1/2+1/4+1/8… är 1 tillåter mig att betrakta som karakteristisk för den fördröjning av Jordens avsvalning som växthusgasers återstrålning av upptagen energi genomsnittligt kan åstadkomma.

    Under dessa 33 mikrosekunder hinner jordytan absorbera cirka 4 TJ solenergi. Varav jag drar slutsatsen att återstrålning förvisso kan ge upphov till en växthuseffekt enligt principen att jordytans avsvalning försinkas.

    Men vid en konservativ kalkyl av vilken temperaturförhöjning som tillförseln av 4 TJ kan åstadkomma finner jag att den måste vara bra mycket mindre än 2 mikrograder C. Jag är övertygad om att växthuseffekter existerar, men betvivlar starkt att de är av signifikant storlek.

  44. #43
    Jag köper det.

  45. Ulf

    Ja nu är ju diskussionen kanske i praktiken inte så intressant eftersom problemet i skymundan håller på att lösa sig ändå. Om det nu fanns ett problem. Här något som aldrig hörs i debatten.
    ”Det mesta av utvecklingen beror på mer energisnål produktion, medan förnyelsen av energisystemen har spelat en mindre roll, så där finns mycket att hämta. Siffrorna hade förstås sett ännu bättre ut om inte länder som Sverige och Tyskland med berått mod avvecklat fossilfri elproduktion.”

    https://www.tn.se/kronikor/losningen-pa-klimatkrisen-fanns-visst-i-sagoberattelserna/

  46. Gösta P #37,

    ”inte signifikant”

    Där har du fel menar jag. Igen: ”klimatförändringarna (alltifrån istider till interglacialer) till variationen inom interglacialerna, dvs vad som orsakar värmeperioder och Lilla Istiden” Detta är det centrala klimatologiska problemet som undersöks av klimatvetenskapen.

    Skillnaden i temperatur mellan inom interglacialer, och än mer mellan glacial istid och icke- istid är mycket stora och kan knappas kallas icke-signifikant.

    Jag menar att M&W-modellen är alldeles för grov för att avfärda växthusgasernas relevans. Denna modell klarar inte av att förklara de stora och ibland mycket långvariga klimatförändringarna. Då MÅSTE andra faktorer än gravito-terminala faktorer komma in. Jag är skeptisk till alla enfaktorsförklaringar till klimatförändringar. Jag menar inte att just växthusgaserna ensamt kan förklara detta (det finns som bekant en lång rad andra delförklaringar). Men det finns också annat som påverkar.

    Min skepsis bottnar i att du utifrån existensen av en gravito-terminal effekt menar dig kunna utesluta en växthusgas effekt. Hur skulle du kunna göra det? Det saknas länkar i en sådan tankekedja.

  47. tty

    ”För det andra har jag alltid betonat att värme enligt termodynamikens andra huvudsats endast kan överföras från varmare materia till kallare (bl a för att kritisera intuitiva tankar som att det uppstår en växthuseffekt genom att atmosfären uppfångar från jordytan utsänd värme, som sedan återsänds till och värmer jordytan; det är en termodynamisk omöjlighet att värme skulle kunna överföras från en kallare atmosfär till en varmare jordyta).”

    Ett tankeexperiment: Du placerar en 1000 grader varm järnplåt ensam ute i rymden. På en annan plats placerar du en likadan plåt, men med en 900 grader varm plåt på vardera sidan. Enligt dig kommer alltså båda 1000 graders plåtarna att svalna lika fort?

  48. tty

    För de som vägrar att betrakta fotoner som små energipaket som far omkring och emitteras och absorberas kan det vara värt att komma ihåg att Einstein fick sitt Nobelpris 1921 just för upptäckten att det är så.

  49. Ingemar # 46

    Det verkar inte som mitt tidigare inlägg gått fram. Den vetenskapliga frågan är om det finns evidens (observation + teori ) som visar att fördubbling av CO2 kan orsaka märkbar global uppvärmning. Anser Du att sådan evidens finns? Vilken i så fall? Om det inte finns någon sådan, så faller väl CO2alarmismen?

    Det verkar som Du tror att frågan istället är att begära evidens för att CO2 INTE kan påverka klimatet på något enda litet vis. Är det så Du tänker? I så fall har Du ju vunnit diskussionen, eftersom något sådant ju är omöjligt. Det är omöjligt att bevisa att vissa INTE har någon insikt alls. Vad som är meningsfullt är att fråga efter tecken på att sådan finns.

  50. Claes J #49,

    Du verkar inte ha förstått. För det första har du själv – i olika sammanhang – påstått att du BEVISAT att CO2 har noll effekt. För det andra att du inte med ett ord berört hur du förklarar de stora diskrepanser i temperatur som jorden upplevt historiskt – med ungefär samma konstanta massa i atmosfären som nu. Det måste du göra om du vill göra anspråk på att bevisa noll effekt av koldioxiden. Dvs du måste visa vilka andra faktorer, vid sidan om Maxwells lilla teori om konvektiv jämvikt, som är stora eller små.

    Därmed utdefinierar du dig själv från den vetenskapliga frågan om klimatets dynamik. Och något BEVIS för CO2s nullitet har du inte.

  51. Matsa

    #41 Claes

    Jag pratar inte om att mäta temperatur utan om att mäta strålning. (Som i sin tur ger temperaturen hos exempelvis en svartkropp).

    Din hypotes om att fotoner är ett olämpligt sätt att beskriva elektromagnetisk strålning är ju helt hårresande. Hur förklarar du fotoelektrisk effekt, Comptoneffekten, excitation och de-excitation av atomer och molekyler, annihilation, parbildning etc. ?

  52. Peter Stilbs

    MYCKET märkligt uttalande om fotoner i #24 (och #41) – overkliga påståenden

  53. Björn

    tty [47]; Som du beskriver det så är den totala värmeenergin för de två plattorna med vardera temperaturen 900 gr, 1800 gr, vilket skall jämföras med den mellersta plattan som håller 1000 gr. Med detta förhållande mottar denna senare platta värmeenergi från de två andra. Naturligtvis kommer den att svalna långsammare än den fria plattan med 1000 gr. Men Gösta menar knappast detta som du vill få fram genom ditt exempel med plattorna, för tidsfaktorn har överhuvudtaget inte nämnts.

    Den här meningen i Göstas kommentar [43] säger mycket om vad han menar. ”Jorden kan inte värma sig själv genom utstrålning av värme som återstrålas”. Vi måste ju betänka att det är högst 50% av den utstrålade värmeenergin från jordytan som kan återstrålas och det säger ju sig självt att denna approximativt återstrålade energi knappast kan värma jordytan. Jordytan har ju i detta exempel förlorat 50% av den ursprungliga strålningsenergin. Däremot så tar det längre tid för den ackumulerade värmen i jordytan att stråla ut på grund av återstrålningen.

  54. Peter Stilbs

    För att anknyta till frågan om ”gaser strålar värme” så finns ju en mångfald litteratur inom främst ingenjörsvetenskapen, där sakfrågan blir högst reell i exempelvis ugnar. Jag har skrivit om detta tidigare här på KU – men hittar inte inlägget, som kanske var för 10 år sedan. Här är dock en modern länk, som ger samma info https://link.springer.com/content/pdf/10.1007/978-3-540-77877-6_66.pdf

    Man finner mer genom att söka på thermal radiation gases – men en del är bakom betalvägg.

  55. Claes Johnson

    Ingemar #50

    Det var konstigt att vad jag säger inte går fram. Ännu en gång: Det handlar inte om att visa CO2 INTE har någon effekt, utan att visa att CO2 har NÅGON effekt och i så fall HUR MYCKET? Kan Du inte se skillnaden? VILKEN effekt har då CO2 enligt Din mening? Den frågan bör Du väl besvara!

  56. GoranA

    Angående om ett kallare föremål kan värma ett varmare har jag gjort några försök som klart visar att det är möjligt.
    Försöket använder en motståndstråd med konstant effekt vilket ger en viss temperatur. Isolerar man tråden kommer trådens temperatur att stiga.
    Temperaturökningen avtar med isoleringens tjocklek så att man går mot en maximal temperatur, ett gränsvärde.
    Man kan läsa om försöket på min blogg

    https://begand.blogspot.com/2020/06/kan-atmosfaren-varma-jorden-till-en.html

  57. Claes J #55,

    Jo, jag ser att du vill slingra dig och lasta över bevisbördan på mig. Från ditt påstående om att Maxvell-spåret visar att CO2 inte har någon effekt till att JAG skall bevisa att den har det. Visst kan vi diskutera det – i själva verket är det bl.a. detta som diskuteras inlägg efter inlägg här på KU. Det är oärligt av dig att nu diskutera något annat än vad du hävdat tidigare. och lätt att visa. Jag tror att jag avslutar debatten med dig här och nu.

  58. Håkan Bergman

    Peter Stilbs #54
    Eller ångpannor.
    http://runeberg.org/inghb/2/1037.html

  59. Claes Johnson

    Peter #52 tty #48

    Vad gäller fotoner så sa den senare Einstein att han inte visste vad det skulle kunna vara för något, och att alla (”Tom and Jerry” som E uttryckte det) som tror sig veta, inte vet.

    Vet Du vad en foton är? En ljuspartikel utan massa som färdas med ljusets hastighet? Ett nätt litet energipaket? Vilka fysiska egenskaper har en foton och vilken natur har de? Är inte ljus ett vågrörelsefenomen? Det är frågor som en fysiker bör kunna besvara, eller hur?

    I sin 1905 lilla artikel om fotoelektriska effekten talar E om ”a heuristic point of view” vad gäller ljuspartiklar/fotoner dvs om något för vilket fysiken fattas. Och det är inte klarare 2022.

  60. Lars-Eric Bjerke

    Gösta Petersson
    Du skriver följande i kommentar #50 till ett tidigare inlägg.
    ”Med statistisk-mekanisk metodik har man funnit att temperaturgradienter, snarlika den naturligt observerade, på grund av den gravito-termala effekten uppstår även i energetiskt slutna system (ingen uppvärmning underifrån, ingen energitransport), samt oberoende av om de tänkta atmosfärerna innehåller växthusgaser eller ej. Tydligare än så kan man knappast visa att temperaturgradienten representerar en gravitationseffekt.”)

    (Se #50 i https://klimatupplysningen.se/den-gravito-termala-effekten/

    Enligt min åsikt strider detta mot termodynamikens andra huvudsats.

  61. Gösta Pettersson

    #46 Ingemar Nordin
    ”Min skepsis bottnar i att du utifrån existensen av en gravito-terminal effekt menar dig kunna utesluta en växthusgas effekt. Hur skulle du kunna göra det? Det saknas länkar i en sådan tankekedja.”

    Nej, alla länkar i den tankekedjan finns på plats, även om jag tydligen misslyckats med att klargöra det för alla. Så här kommer ett nytt försök att kondenserat beskriva tankekedjan:

    1. IPCC har som förstahandsbelägg för växthuseffektens existens och signifikans anfört att den 32-gradiga atmosfärseffekten (temperaturskillanden mellan jordytan och utstrålningsskiktets balansskapande -18 °C) utgör en naturlig växthuseffekt åstadkommen av luftens förindustriella halt av vattenånga och koldioxid.
    2. Maxwellskolan (och Manabe) har funnit att den 32-gradiga atmosfärseffekten tillfredsställande kan förklaras som en gravito-termal effekt. Jordytan är 32 grader varmare än utstrålningsskiktet på grund av den gravito-termalt uppkomna troposfäriska temperaturgradienten.
    3. Därav följer att den naturliga växthuseffektens bidrag till jordytans temperatur befunnits vara av insignifikant storlek. (Jordytans temperatur är ”precis” den som en planet på Jordens avstånd från solen kan förväntas uppvisa. Det har uppstått ett troposfäriskt utstrålningskikt med den balansskapande temperaturen -18 °C. Jordytan är 32 grader varmare på grund av gravitationen. Ingenting tyder på att jordytan är varmare än vad man skäl att tro när hänsyn tas till den gravito-termala effekten).
    4. Och är den naturliga växthuseffekten (orsakad av ca 10 000 vattenånga + 300 ppm koldioxid) av insignifikant storlek så lär även den förstärkta växthuseffekten (orsakad av upp till ytterligare 300 ppm koldioxid) vara det.
    5. IPCC:s förstahandsbelägg för växthuseffektens signifikans utgör för mig ett förstahandsbelägg för växthuseffektens insignifikans.

  62. Matsa

    #59 Claes

    Elektromagnetisk strålning har både våg- och partikelegenskaper. Detta kallas inom fysiken våg-partikeldualismen, vilken dessutom inkluderar att samma sak gäller för ”partiklar” (elektroner, protoner, …).

    Fotonen har enligt Einstein egenskaperna att den, trots att vilomassan är noll, har en rörelsemängd ( = Plancks konstant dividerat med våglängden) och kinetisk energi (= Plancks konstant gånger frekvensen). Där har du några fysiska egenskaper.

    Vågegenskaperna beskrivs av Maxwell. De är fluktuationer hos elektromagnetiska fält. Ofta beskrivs fotonen som ett ”vågpaket” bestående av en superposition av vågor med olika våglängder i ett våglängdsintervall.

    I partikelfysikens standardmodell är fotonen fältpartikeln (bosonen) för det elektromagnetiska fältet.

  63. Fredrik S

    Fotonen brukar beskrivas som ett energipaket, ett energikvantum, som kan ha olika utbredning i olika riktningar.

    Fråga vilken professor i experimentell fysik som helst.

    Detta är väl inget konstigt?

  64. Gösta P #61,

    Du argumenterar kring den 32-gradiga uppvärmningen. Jag argumenterar om det som tillhör mainstream debatten i den vetenskapliga klimatdebatten, nämligen hur vi kan förklara istider, variationer inom glacialer och icke-istider. Där har du ingalunda BEVISAT att koldioxiden är en nullitet.

    Och hur skulle det Maxwellska spåret kunna göra det när atmosfärens volym är konstant över årmiljonerna?

    Eller vill du som Claes J förneka all känd strålningsfysik inklusive kvantmekaniken och fotoner?

  65. Gösta Pettersson

    #60 Lars-Eric Bjerke
    ”Enligt min åsikt strider detta mot termodynamikens andra huvudsats.”

    Jag är lite osäker på vad ditt ”detta” syftar på. Men med kännedom om vilka argument som brukar anföras på alarmistiska bloggar förmodar jag att det är Loschmidts gravito-termala teori för uppkomsten av den troposfäriska temperaturgradienten som du anser strida mot andra huvudsastsen. Maxwell invände ju på 1800-talet mot Loschmidts tankar med hänvisning till att enligt andra huvudsatsen bör en gasmassa vara isotermisk vid termisk jämvikt.

    Men planetariska kroppars atmosfärer är inte isotermiska utan uppvisar konstanta temperaturgradienter så snart trycket överskrider ca 0,1 bar. 1900-talets forskning har gett Loschmidt rätt. Isotermi-jömvikten gäller endast under förhållanden där den gravito-termala effekten är försumbar. Jämför mitt svar i #42.

  66. Jag frågar Ingemar om vilken evidens det finns för att CO2 märkbart påverkar jordens medeltemperatur? Då avslutar Ingemar debatten. Hur skall jag tolka detta?

    Jag avfärdar inte all kvantmekanik eller strålningsfysik, vad det nu är. Hur skulle jag kunna göra det? Men jag frågar i likhet med Einstein vad en foton är för något, och vilka fysikaliska egenskaper den har? Hur rör den sig i rummet? Flyger?

    Är den ett energipaket utan massa och rumslig utsträckning? Hur interagerar den med andra fotoner eller materia? Vilken matematisk modell/ekvation beskriver en foton? Men jag får inga svar. Bara meddelandet att det inte alls är något konstigt med fotoner, även om den mogne Einstein så ansåg när han tog avstånd från de ljuspartiklar/fotoner den unge Einstein hittat på som heuristisk förklaring av fotoelektricitet, en förklaring utan fysik.

    När Nobelkommittén ansåg sig pressad efter slutet av första världskriget med försoning med Tyskland på agendan, att ge priset till Einstein, så uppstod frågan för vilken vetenskaplig bedrift det skulle vara. Inte för relativitetsteori som ingen begrep och efter mycket letande hittade ledamoten Oseen Einsteins blygsamma artikel ”a heuristic point of view on photolectricity” från 1905 som prismotivering, och det fick duga (under protester från Arrhenius o Gullstrand, se bok av Ant Elzinga). I prismotiveringen stod att priset speciellt INTE hade givits till Einsteins relativitetsteori!

  67. #43 Gösta Pettersson

    Din föreställningsvärld saknar helt överensstämmelse med verkligheten: ”Om Jorden skickar ut värmestrålning i form av en foton och den uppfångas av och exiterar en växthusgasmolekyl, så är den i min föreställningsvärld frekventaste följden att den exiterade molekylen efter cirka 50 ns relaxeras genom oriktad återutsändning av fotonen. Varvid det av geometriska skäl är vid pass 50% sannolikhet att fotonen återförpassas till jordytan.”

    Detta hänger väl ihop med den olyckliga formuleringen i ditt inlägg: ”För säkerhets skull vill jag förtydliga att jag med växthuseffekt fortfarande avser begreppet så som det definierats av IPCC, dvs. som den radiativt betingade förhöjning av jordytans temperatur som förmodas åstadkommas av atmosfärens växthusgaser på grund av deras specifika förmåga att absorbera och återutsända infraröd värmestrålning.”

    Det är inte fråga om någon återutsändning!

    Några elementa:
    1) I fria rymden, långt från alla stjärnor är det mycket kallt. En koldioxidmolekyl har nästan 100% sannolikhet att befinna sig i sitt grundtillstånd.

    2) Om en koldioxidmolekyl ute i fria rymden råkar absorbera en foton (från en avlägsen stjärna) och därmed övergå till ett vibrationsexciterat tillstånd så kommer den efter mycket lång tid att sända ut en foton och därmed förlora sin vibrationsenergi.

    3) Energin hos den utsända fotonen är inte densamma som energin hos den absorberade. Beroende på om absorptionen ledde till rotationskvanttal 0 eller 2 så kommer rotationskvanttalet efter emissionen att vara 0 eller 2 alternativt 0 eller 2 eller 4.

    4) Beroende på (den slumpmässiga) förändringen i riktning kommer den utgående fotonens energi att skilja sig från den inkommandes på grund av dopplereffekten.

    Orden återutsända liksom återstrålning är olyckliga i sammanhanget därför att dom leder tankarna fel. Jag menar inte att orden egentligen är fel men att man bör akta sig noga för att använda dom i klimatsammanhang. Det bör heta utsända respektive strålning om fenomen i atmosfären.

    I atmosfären är det så här:

    1) När jorden skickar ut värmestrålning i form av en foton och den uppfångas av och exiterar en växthusgasmolekyl, så kommer energin nästan omedelbart att omvandlas till kinetisk energi, dvs värme, genom kollition med någon luftmolekyl.

    2) I atmosfären är det varmt. Där råder termodynamisk jämvikt. Det betyder att alla växthusgaser till rätt stor del befinner sig i ett vibrationsexciterat tillstånd. I vilken grad beskrivs av Boltzmannfördelningen. För koldioxid vid 667 cm-1 är energin 84 meV och i storleksordningen 2% av molekylerna befinner sig i vibrationsexciterat tillstånd.

    3) Sannolikheten för den enskilda CO2-molekylen att utsända en foton är mycket låg, men betraktar man atmosfären genom ett spektroskop finner man exakt den strålningsintensitet som man finner om man i vakuum mäter strålningen från en ideal svartkropp. Detta gäller givetvis bara inom absorptionsbandet och på höjder där temperaturen är konstant inom det område dit strålningen kan nå. Vid markytan och för CO2 667 cm-1 är det avståndet mycket mindre än 10 meter.

    4) Sträckan strålningen kan gå innan den absorberas beror på luftens täthet, dvs antalet CO2-molekyler per volymsenhet.

    5) Det finns en höjd där antalet CO2-molekyler ovanför en CO2-molekyl som råkat sända en foton uppåt är så litet att fotonen ibland inte absorberas utan fortsätter ut i rymden och därvid tar med sig energi från jorden.

    6) Fördubblar man CO2-halten från 300 till 600 ppm kommer denna höjd att öka tills antalet ovanförliggande molekyler är nästan detsamma. (Spektrallinjens form beror av tryck och temperatur.) Därvid kommer strålningen från jorden att utgå från ett atmosfärsskikt med annan temperatur vilket betyder att utsänd strålning blir annorlunda.

    7) Utstrålningshöjden beror av respektive spektrallinjes intensitet (som varierar över linjeprofilen beroende på höjd) Vid 667 cm-1 sker strålningen från stratosfären och då stiger utstrålningen från jorden vid höjd CO2-halt och därmed förlorar stratosfären mer energi. Huvuddelen av spektrallinjerna är svagare och strålar från troposfären. Där sjunker temperaturen med höjden så mer CO2 leder till att mindre energi sänds ut på respektive frekvens. Det leder till svagare avkylning av troposfären och därmed stigande temperatur i övre troposfären. I stratosfären vinner strålningen från de starkaste linjerna så där stiger temperaturen med ökande CO2-halt.

    Man mäter spektrum från jorden och ur den observerade intensiteten kan man deducera vilken höjd strålningen kommer ifrån och däemed hur mycket olika skikt i atmosfären kyls på grund av IR-strålning. Man ser givetvis även strålning från marken i det fall att luftfuktigheten är mycket låg (Antarktis)

    Oavset på vilken höjd troposfären kyls genom strålning, så kommer vertikal energitransport genom latent värme, konvektion (och i liten utsträckning strålning) att se till att laps rate blir i enligher med gravito-termala effekten, helt i enlighet med Loschmidts ekvationer och den av Maxwell föreslagna ”konvektiva jämvikten”.

    Vid ökad CO2-halt kommer den ”radiativa, konvektiva jämvikten” att nå lite högre upp enligt fig 16 i Manabes skrift. Det beskrivs så här: ”Free and forced convection, and mixing by the large-scale eddies, prevent the fapse rate from exceeding a critical lapse rate equal to 6.5C/km. Whenever the lapse rate is subcritical, the condition of local radiative equilibrium is satisfied.” Fig 2 i din presentation tycker jag är en överförenklad bild av Manabes figur 16. Att approximera med räta linjer tycker jag inte är OK.

    Nyckeln till din bristande förståelse finns här: ”Förtydligat är därför självmotsägelsen i M&W-modellen att den accepterar att den 32-gradiga temperaturskillnaden mellan utstrålningskiktet och jordytan för koldioxidhalten 300 ppm återspeglar en konvektiv jämvikt och därmed representerar en gravito-termal effekt utan radiativa inslag. Vilket innebär att den ”naturliga växthuseffekt” som enligt IPCC orsakas av troposfärens förindustriella halt av växthusgaser (cirka 10 000 ppm vattenånga och 300 ppm koldioxid) enligt modellen är av insignifikant storlek.” Detta förutsätter att utstrålningskiktets höjd inte påverkas av CO2-halten. MEN det är just utstrålningskiktets höjd som stiger med ökande CO2-halt.

    ”Utstrålningskiktet” är ett begrepp från en förenklad modell. Det är ett medelvärde av den höjd man utifrån rymden kan observera. Medelvärde över tid, plats, observationsvinkel och våglängd. När höjden ökar avsevärt för CO2-strålningen så ökar medelvärdet över hela spektrum litegrann och det kan beräknas. Lite högre höjd för utstrålningskiktet och oförändrat laps rate ger 1,4 grader högre marktemperatur vid fördubbling av CO2 enligt Manabe och Wetherald såväl som Wijngaarden och Happer.

    Figure 1 här: https://skepticalscience.com/print.php?r=35 visar skillnaden i spektrum från jorden från 1970 till 1996. Man ser tydligt effekten av CO2 och metan. Det borde finnas modernare data, men svårt att hitta.

    Detta är ett bevis för att ”utstrålningsskiktets” höjd ökat och därmed måste jordytans temperatur ha ökat på grund av CO2.

  68. Munin

    # 67
    Finns det uppgifter om eller går det att beräkna hur utstrålningsskiktets höjd förändras vid olika nivåer på koldioxid i atmosfären t.ex. vid 300, 420, 600, 800 ppm? Hur stor är omkretsen på atmosfären vid dessa olika nivåer? Ska det uttryckas i måttenheten mil eller km för att kunna greppa storleken på ändringarna? Går det också att räkna ut det som vilken volym (kubikmil, kubikmeter) atmosfären har vid dessa olika koldioxidnivåer?

  69. Björn

    Det kanske är så, att när man börjar skärskåda eller närmre studera något, så upptäcker man att den intuitiva bild av något man har, inte längre håller, som här med fotonen. Det är just rubbandet av invanda föreställningar som ger upphov till lite tvister, vilket kanske är bra som här, där fotonen är i allra högsta grad en viktig överförare av energi. Utan fotonen fanns ju inget ljus, eller elektromagnetisk strålning, eller liv. Därför tål fotonen att diskuteras på alla ledder utan att peka finger, för den är trotts allt ännu inte helt förstådd. Till historien hör också att fotonen faktiskt inte är masslös, för den massa som emitterar fotoner, förlorar något av sin vikt, om än mycket lite. För att Einsteins berömda formel skall hålla, E=mc^2, måste varje minskning av massan ge en ökning i E och tvärtom. Tyvärr så är inte science settled i något avseende. Nya fakta kan faktiskt rubba det mesta.

  70. Håkan Bergman

    Munin #68
    Du kan själv se hur utstrålningen förändras med höjden i modtran.
    http://climatemodels.uchicago.edu/modtran/
    Välj 1976 U.S. Standard Atmoshere och looking down, låt allt annat vara tills du varierar höjd och CO₂-halt. Om du börjar från 0 och går upp 100 meter, 0.1 km obs punkten, åt gången ser du skillnaden i boxen längst ner för Upward IR Heat Flux, den ökar för 0.1 km men sen minskar den med höjden.

  71. tty

    #66

    ”Men jag frågar i likhet med Einstein vad en foton är för något, och vilka fysikaliska egenskaper den har? ”

    Den är en stabil, oladdad boson utan vilomassa med spin +-1 och paritet -1 som är växelverkanspartikel (gauge boson) för elektromagnetisk växelverkan. Baryontal. leptontal och färgkvanttal är alltså 0. Den påverkas utöver den elektromagnetiska växelverkan även av svag växelverkan och gravitationsväxelverkan, men inte stark växelverkan. Den rör sig med ljusets hastighet (vilket kanske är dumt att påpeka, eftersom ljus just är fotoner). Och, ja, den är både en vågrörelse och en partikel vilket lätt kan påvisas experimentellt.

  72. tty

    #69

    ”Till historien hör också att fotonen faktiskt inte är masslös, för den massa som emitterar fotoner, förlorar något av sin vikt, om än mycket lite.”

    Den har ingen vilomassa enligt gängse teorier, vilket experimentellt har visats vara korrekt med en osäkerhet av mindre än 10^-18 eVc^-2. Däremot har den en relativistisk massa, eftersom den har energi.

  73. tty

    ”…de ljuspartiklar/fotoner den unge Einstein hittat på som heuristisk förklaring av fotoelektricitet, en förklaring utan fysik.”

    Du tycks ha missat att fysik är en empirisk vetenskap. Det är inte alltför ovanligt med resultat som inte har någon bra teoretisk förklaring. Supraledning t ex, den upptäcktes av Onnes 1911 men den teoretiska förklaringen kom först 1957.

  74. tty

    #67

    Instämmer helt!

  75. tty

    #68

    ”Finns det uppgifter om eller går det att beräkna hur utstrålningsskiktets höjd förändras vid olika nivåer på koldioxid i atmosfären t.ex. vid 300, 420, 600, 800 ppm? ”

    ”Utstrålningsskikt” är som sagt en förenkling. Om vi hade en syn som fungerade inom IR-området och tittade uppifrån ner i molnfri luft skulle det se ut ungefär som när det är dimma. Dimman skulle vara olika tät vid olika våglängder och vi skulle kunna se olika djupt ner i ”dimman”, men det skulle inte finnas någon skarp gräns (utom där det finns täta moln).

    Om du vill se effekten av olika CO2-halter så gå till följande site:

    http://climatemodels.uchicago.edu/modtran/

    Välj höjd 70 km och ”Looking down” (=grundinställning). Till höger ser du en kurva som anger utstrålningstemperaturen och den temperaturprofil som beräkningen gäller (man kan välja ett antal olika profiler).

    I utstrålningskurvan kan du ungefärligen läsa av utstrålningstemperaturen vid olika våglängder och genom att klicka på temperaturprofilen kan du läsa av den ungefärliga höjd som den temperaturen motsvarar.

    Du kan sedan ändra siffran för koldioxidmängden och kontrollera hur resultatet förändras. Men med de nivåer du anger kommer du knappast att se någon mätbar förändring.
    En god idé kan vara att spara den första kurvan som ”save this run to background”. Den ligger då kvar och du kan direkt jämföra med de senare kurvorna.

  76. #66 Claes Johnson

    Vad är en foton? Många har frågat sig detta. Jag skulle vilja säga att fotoner inte finns i verkligheten. Fotonen hör hemma i en viss (mycket framgångsrik) modell av verkligheten. Där kan man se fotonen som både partikel och elektromagnetisk vågrörelse. Samtidigt.

    Man kan konstruera paradoxer av olika slag. Vi vet att det finns gravitationslinser, galaxer ute i universum. När vi observerar en foton från en avlägsen kvasar genom en sådan lins har fotonen, när den passerade linsen, varit utspridd över ett område på många hundra tusen ljusår. Vad skulle ha hänt om någon astronom på galaxens ena sida observerat fotonen? Vad skulle hända med den halva som fanns på andra sidan gravitationslinsen?

    Ett sätt att se saken är att fotonen inte finns varken som partikel eller våg utan är en beskrivning av en händelse. Det som ”finns” är en vågfunktion som beskriver sannolikheten att detektera en foton på en viss plats. Dvs en händelse, ett energiutbyte mellan det elektromagnetiska fältet och en laddning av något slag. Om/när en foton detekteras försvinner den till fotonen associreade vågfunktionen momentant över hela universum.

    Hur vågfunktionen ser ut beror på hur vi arrangerat vårt detektorsystem. Mäter vi frekvens oerhört noga är vågfunktionen mycket utsträckt i rummet men mäter vi tid så är utsträckningen liten.

    Det elektromagnetiska fältet kan inte vara noll. Det förbjuder osäkerhetsrelationen. Det elektromagnetiska fältet har en nollpunktsenergi som ökar med frekvensen upphöjt till tre. I varje punkt i rummet finns således ett elektromagnetiskt fält som är summan av bidragen från alla fotoner plus nollpunktsenergins fält. Sannolikheten för en viss händelse, växelverkan mellan fältet och något annat (en elektrisk laddning eller ett magnetiskt moment) beskrivs av kvantelekrodynamikens formler. Den allmänna åsikten är att man inte kan förstå dessa formler men man kan lösa de ekvationer de ger upphov till.

    Två gånger (minst) har lösningarna varit så absurda att de inte ansetts rimliga av den etablerade vetenskapen.

    Jag minns ett seminarium där Josephssons teori, sedemera Josephssoneffekten presenterades. Föreläsaren visade josephssons artikel (första sidan av den.) Jag tror det var i Physical Review. Ett eller att par nummer senare kom en artikel där en av tidens ledande kvantteoritiker förklarade vad Josephsson hade gjort för fel och att effekten inte fanns. Artikeln efter var skriven av ett par experimentalister som hade gjort experimentet och verifierat Josephssons teoretiska resultat! Det dröjde inte länge tills den unge Josephsson fick Nobelpris. Josephssoneffekten har stor praktisk betydelse i dag.

    Det andra är Einstein–Podolsky–Rosen-paradoxen. Nobelpris i år. https://www.nobelprize.org/uploads/2022/10/popular-physicsprize2022-swedish.pdf Här handlar det om en tvåfotonvågfunktion som kollapsar samtidigt på två platser långt från varandra. Det är rätt komplicerat, tvingar man en av fotonerna att ha en viss polarisation genom att låta den gå genom en polarisator före detektionen blir den andra fotonens polarisation momentant bestämd trots att information om vad som hänt omöjligen kan ha skickats från den ena detektorn till den andra därför att information eller energi i annan form inte kan skickas fortare än med ljusets hastighet (för allt vad man vet.)

    När det gäller kvantmekanik är det klokt att hålla sig till en relativt enkel modell där man tänker vågor eller partiklar och växlar mellan dessa ”bilder” beroende på vad man vill veta. Är man någotsånär bekant med experimentell fysik vet man hur ett brett sortiment av relevanta experiment utfaller – och då är det inte konstigt att man tycker sig förstå. Men det gör man nog egentligen inte.

    När det gäller koldioxiden och dess växelverkan med det elektromagnetiska fältet så är det experimentell erfarenhet som talar, förklädd till teori.

  77. Munin

    # 70, 75
    Tack för dessa uppgifter. Det gör det betydligt mer konkret. Ett annat sätt att se på koldioxidhalter och när de ändras måste vara att ju högre halt det finns i atmosfären desto mer koldioxidmolekyler svävar runt i atmosfären. Det innebär att deras sammanlagda massa ökar med stigande halt i atmosfären. Finns det uppgifter om eller går det att räkna ut hur stor denna massa är vid olika halter och hur stora tal handlar det om?

  78. Håkan Bergman

    Munin #77
    Det är nog försumbart. först ska man komma ihåg att 32/44 av massan för CO₂-molekylen är syret som redan innan fanns i atmosfären. Blandningen 80% kväve och 20% syre har en medelvikt av 28,8 så 400 ppm CO₂ ökar massan med 400*12/28,8 = 167 miljondelar.

  79. Munin

    # 78
    Ja, då blir det kolatomerna (C) som det kan vara intressant att räkna ut hur deras totala massa är i atmosfären och hur den ändras med halten koldioxid i atmosfären. Det blir denna massa och hur den ändras, som gravitationen kommer att styra över (allt annat lika). Vad gör gravitationen med denna massa? Var vill gravitationen placera denna massa?

  80. Gösta Pettersson

    #56 GoranA

    Verbet ”värma” kan i svenskan användas fär två olika begrepp:
    1. Överföra värme till (så som brasan värmer oss)
    2. Försinka avkylning av (såsom en ylletröja värmer oss)

    I ditt försök värms inte tråden genom överföring av värme från den kallare isoleringen till den varmare tråden. Tråden värms genom att du tillför värmeenergi när du skickar ström genom den. Isoleringen försinkar avkylningen av tråden, med påföljd att den uppnår en högre jämviktstemperatur när den isoleras.

    Att bevara värme genom isolering är en välkänd princip. Att överföra värme från en kallare kropp till en varmare är omöjligt enligt termodynamikens andra huvudsats.

  81. Leif Å #76,

    Du berör en debatt om tolkningar av kvantmekaniken som pågått i över 100 år. Det finns mycket att säga om den.

    Jag har bara en liten kommentar: Årets nobelpris har sitt ursprung i Bells teorem från 1964 (vill jag minnas). Och ännu längre tillbaka till diskussionen om Einstein-Podolski-Rosens paradox (som de slog i skallen på Niels Bohs tolkning.

    Jag hade faktiskt förmånen att träffa och brevväxla lite med John Bell på 70-talet. Han hade förstås en åsikt om hur man skulle tolka kvantmekaniken och det var David Bohms tolkning (50-talet) som bl.a. innebar existensen av överföring av information i överljushastighet. Bohm skrev en lärobok i ämnet som i sin tur gick tillbaka till De Broglies teori från 1910-talet och den kom att kallas för ”pilotvågsteorin”. Den antog att det fanns både vågor (piloten som styrde partikelns väg genom rummet) och partiklar.

    En av nobelpristagarna var Aspect som genomförde ett berömt experiment som skulle visa att s.k. sammanflätade tillstånd (en superposition) omöjliggjorde en ren partikelteori med s.k. dolda variabler. Men det gjorde experimentet inte eftersom filtren inte uteslöt att det kunde ske en selektion av mätresultat …

  82. Gösta Pettersson

    #64 Ingemar Nordin

    Jag blir mer och mer förbryllad av dina kommentarer:

    ”vill du som Claes J förneka all känd strålningsfysik inklusive kvantmekaniken och fotoner”

    Det jag refererat till och stött mig på i årets inlägg är allmänt accepterade fysikaliska samband och beräkningar, anammade även av GHE-proponenter. Vad jag tycker mig kunna utläsa av din fråga är att du anser att strålningsfysiker redan har lagt fram klara belägg för växthuseffektens signifikans. Betraktar du möjligen fakta som att växthusgaser absorberar värmestrålning, att en stor del av den absorberade värmeenergin återstrålas till jordytan, att luften avger nedåtriktad värmestrålning, Tyndalls experiment, Arrhenius utredning av temperaturer i en gravitationsfri atmosfär etc. som sådana klara belägg? Det gör nu inte jag.
    Och vad fotoner (samt återstrålning) beträffar, se mitt svar #43.

    ”Du argumenterar kring den 32-gradiga uppvärmningen”

    Ja, eftersom det är den inlägget handlar om. Ett välvalt ämne, alldenstund om man accepterar slutsatsen att den nutida naturliga växthuseffekten är av insignifikant storlek, så lär den (i motsats till vad du hävdar) även ha varit det under närliggande forntida istider och icke-istider.

    ”Och hur skulle det Maxwellska spåret kunna göra det när atmosfärens volym är konstant över årmiljonerna?”

    Jag har ingen aning om vad det är för något argument du syftar på. Jag har aldrig träffat på något resonemang där atmosfärens volymkonstans åberopats som stöd för någon tanke. Så var snäll och förtydliga. Jag är uppriktigt nyfiken.

  83. Gösta Pettersson

    #67 Leif Åsbrink (och inbakat #7 tty)

    Jag har läst din kommentar. Den påminner tankemässigt om vad tty anförde i #7 och som jag i svaret #17 föreskickade att jag eventuellt skulle återkomma till. Jag tänker ge mitt svar på dina och tty:s synpunkter i morgon.

    Datorn säger mig att idag är det torsdag. I min tidsförvrängda syn på tillvaron är det därför fredag i morgon. Varför Bellman i Fredmans sång nr 49 ger en för mig adekvat framställning av sakernas tillstånd:

    Som nu och emedan rätten funnit allaredan
    att Bacchi parter tagit in så många kvarter
    att ej mer någon ser
    varken himmel, ny ell’ nedan.
    alltså må saken då
    vila till och med till fredan

  84. Lennart Bengtsson

    Växthuseffekten som Manade klargjorde i sin artikel från 1967 är sedan dess övertygande påvisad i såväl empiriska som teoretiska arbeten. Sådana artiklar har de fakto ingående diskuterats i denna blogg.

    Att ständigt ifrågasätta fundamental naturvetenskaplig kunskap utan några som helst rimliga belägg är lite tröttsam, to say the least. Om man är oförmögen eller ovillig att lära sig elementa i fysik borde man i ändå lyssna på sådana som råkar ha nödvändig kunskap. Varför inte lyssna på W. Happer och R Lindzen som många läsare på denna blogg känner till-

  85. Hej Leif #76

    Det var intressant att läsa vad en riktig fysiker tänker om fotonen, nämligen att den inte finns i verkligheten. Men det ställer väl frågor om varför fysiker laborerar med detta begrepp?

    Svaret framställs ofta som Einsteins försök ”the miraculous year 1905” att förklara fotoeelektricitet och speciellt då att endast ljus av tillräckligt hög frekvens måste till för att generera en ström av elektroner, dvs en tröskeleffekt. Men Einstein introducerade ljus som partiklar endast som en ”heuristic point of view”, dvs utan fysik och det anknyter till vad Leif säger.

    Men kan man förklara tröskeleffekten på ett mer fysikaliskt vis?

    Jag gör ett sådant försök på https://computationalblackbody.wordpress.com som beskriver ett drivet system av oscillatorer av viss temperatur T med frekvenser upp till en viss cut-off som skalar med T. Systemet har liten dämpning som motsvarar utgående kraft och drivningen består av en oscillerande kraft av viss frekvens som inkommande.

    Tröskeleffekten innebär att frekvenser under cut-off balanseras som utgående = inkommande, medan frekvenser över cut-off inte ger något utgående utan lagras som värme i systemet. Det är tröskeleffekten som ger riktningen av värmeöverföringen från varm till kall. Ensidig värmeöverföring från varm till kall. På samma sätt som ensidig kunskapsöverföring från mer vetande till mindre. Inte tvåsidig framåtillbaka.

    En sådan modell har tydlig fysik som vågmodell och är begriplig. Det var en sådan modell Planck betraktade och sökte tröskeleffekten från statistik, utan att bli nöjd. Men tröskeln kan kanske alternativt förklaras deterministiskt som ovan. Man kan tex tänka sig att en kallare kropp är “stelare” och därmed ha lägre cut-off än en varmare.

    Däremot saknar en foton/partikel modell tydlig fysik och någon tröskeleffekt vad gäller värmeöverföring finns inte heller. Många verkar tro att varje kropp spottar ur sig fotoner som värme oberoende av omgivningens temperatur, men varför tror man på något sådant?

    Leif: Är detta något vi kan diskutera? Vänligen Claes

  86. Lennart #84

    Vilka experiment finns det som visar/indikerar att mänskliga utsläpp av CO2 höjer jordens medeltemperatur mätbart? Påverkades temperaturen mätbart under av nedgången i utsläpp under Covid 20-21? Om inte, varför? Vilka andra experiment med jordens klimatsystem finns att titta på?

    Det finns olika modeller som visar allt från katastrofal uppvärmning till katastrofal nedkylning över noll påverkan. Vilken modell skall man tro i frånvaro av experiment? Skall Sverige satsa på att bli världens första fossilfria s k välfärdssamhälle med utgångspunkt från dessa modeller, som inte säger något?

  87. När denna tråd presenterades på KU för två dagar sedan skrev jag ett inlägg på min fb;
    ”Just nu pågår en intressant diskussion om hur stor ”växthuseffekten” de facto är på KU. En verbal brottning mellan ett antal kunniga naturvetare med professorskompetens inom fysik, kemi och geovetenskap (och om jag får gissa kommer även meteorologiprofessorn Lennart B snart att blanda sig i). Vad som styr klimatet är en mycket komplicerad historia det är det som gör det så intressant. Välkommen att ta del av ”brottningsmatchen” ”
    och det blev precis som jag förväntade mig, en riktigt intressant verbal ”sumobrottning”. Intressant att se att dualismen inom den fysikaliska grundvetenskapen finns kvar (som tog sin början för drygt hundra år sedan) och att den kokar ner även i den kaotiska klimatvetenskapen.
    Stor tack för till alla inblandade för att ni gjort mig och andra än mer förvirrade fast på ett högre plan

  88. tty

    #77

    Mängden koldioxid i atmosfären är ca 3 000 miljarder ton.

  89. Matz Hedman

    #88 tty
    Jag är lite lat men eftersom du redan räknat.
    Hur många kubikmeter luft måste CCS tekniken processa för varje önskad global ppm om vi förutsätter att all CO2 tas bort i processen.

  90. Munin

    # 88
    Tack för den uppgiften. Då får jag det till att 1 ppm koldioxid motsvarar ca 7 miljarder ton. Det skulle alltså vara den mängden och hur stora ppm ändringarna är som påverkar atmosfären. Dock bör den räknas om till mängden kol det motsvarar, eftersom syret redan finns i atmosfären. Det blir således den mängden kol och ändringarna av denna massa, som jordens gravitationskraft påverkar och som kan vara en del i förklaringarna av hur klimatet blir påverkat.

    Hur förhåller sig ppm-ändringarna och ändringarna av massan av kol i atmosfären till de kvantiteter av fossila bränslen vi använder på jorden?

  91. Lennart Bengtsson

    Nettoproduktionen av koldioxid uppgår till ca 38 GTon/år.

    45-50 % ackumuleras i atmosfären ( på tidsskalor > 50-100år) eller cirka 5 Ton C/år

    1ppm motsvarar 2.12 Gton C.

  92. Lennart Bengtsson

    Matz

    En pelare luft väger 10 ton och jordens yta är 510×10^12 m2 = 5 Pton = 5 x 10^15 ton = 5Exa kg ( i standardenheter)

    Det blir en hel del luft att processa då en kubikmeter luft väger litet mer än 1 kg.

  93. Håkan Bergman

    Om luften väger 1300 g per m³ och man räknar med att CO₂-molekylen väger cirka 1,5 gånger övriga molekylers medelvikt får man följande för 400 ppm.
    1300×600÷10⁶ = 0,8 g CO₂ per m³ luft, när man börjar dräneringen!
    Nån som är villig att satsa egna pengar på en sån där makapär?

  94. Munin

    # 91
    Tack för uppgifterna. Då finns det ca 900 miljarder ton kol (C) i atmosfären vid 420 ppm och ändringstakten är 5 miljarder ton per år.
    Ändringstakten kan uttryckas som 5/900=0,0056 ökning av kolets massa i atmosfären. Det är på denna låga takt klimathotet i så fall är baserat.
    En fråga, är de 5 miljarderna ton kol från enbart fossila bränslen eller finns det fler källor?

  95. Gösta P #82,

    ”Vad jag tycker mig kunna utläsa av din fråga är att du anser att strålningsfysiker redan har lagt fram klara belägg för växthuseffektens signifikans.”

    Nej, jag har inte ens antagit det, även om jag tror att växthuseffekten är signifikant (om än ganska liten i nuläget) .

    Det jag undrar över är hur du menar att du kommer fram till att växthusgasen INTE är signifikant? Jag kan för debatten skull köpa att Maxwellspåret förklarar de 32 graderna. OK.

    Men sedan kommer vi till frågan om klimatFÖRÄNDRINGAR. Och dessa är verkligen ingen nullitet, särskilt om vi ser på skillnaden i global temperatur mellan istider och icke-istider. Eftersom atmosfärens volym och massa har ganska konstant under årmiljoner så måste vi hitta andra faktorer som orsakar dessa förändringar. Vi kan kalla dem för X. Eftersom X kan orsaka stora temperaturförändringar på jorden så är de signifikanta och lätt mätbara.

    I mängden faktorer, X, så ingår allt möjligt som vi har stött och blött här på KU i massor av inlägg. En sådan faktor är växthusgaser. Hur vet du att just dessa gaser är insignifikanta? – Ja, ett sätt vore att gå igenom de troligaste kandidaterna, såsom solen, havsströmmar, molnen, vulkaner och då även koldioxid. Och så kanske du kan komma fram till att, jo, just växthusgaserna har en väldigt liten uppvärmande effekt. Men det gör du inte, utan istället hänvisar du till att Maxwell minsann har förklarat de 32 graderna – jaha, men det var ju inget svar på frågan om vad som orsakar de stora och signifikanta temperaturFÖRÄNDRINGARNA.

    Så min kritik rör inte Maxwell eller att den effekten kan hålla värmen vid jordytan eller att den är upphovet till temperaturgradienten. Utan min kritik rör den slutsats som du kommer fram till, nämligen att allt annat i X är insignifikant – och då särskilt koldioxiden och andra växthusgaser. För gör du det så saknar du förklaring till de stora temperaturFÖRÄNDRINGAR som vi kan observera.

    Det saknas alltså länkar i din argumentationskedja. Hur ser du exempelvis på strålningsfysiken, som Happer, Lindzen och många andra pekat på? Hur ser du på de stora havsströmmarna? Hur ser du på molnens betydelse? Osv.

  96. Mikael Andersson

    #94
    Människan CO2-utsläpp är mycket mindre än naturens utsläpp. Djurs och mikroorganismers konsumtion av växter står för ca 220 gigaton per år. Utandning från växter släpper ut ytterligare omkring 220 gigaton. Haven släpper ifrån sig 332 gigaton. I motsats till detta, när man kombinerar effekterna från fossila bränslen och ändringar av landanvändning släpper människan ut endast 29 gigaton per år.

  97. Mikael Andersson

    #94

    (Källa: Figure 7.3, IPCC AR4).

  98. Mikael Andersson

    Ingemar Nordin

    Vad anser du om denna artikeln om klimatförändringarna historiskt och om förekomsten av eller avsaknaden av ”feedback systems”?

    https://www.science.org/doi/10.1126/sciadv.adc9241

  99. Munin

    # 91, 96, 97
    Tack för denna komplettering. Då bör det handla om 5 miljarder ton kol, som årligen kommer genom människans aktiviteter.

    Den låga ändringstakten som klimatpåverkande faktor kommer också minska undan för undan om vi också tar hänsyn till kunskaperna inom strålningsfysiken. Det ska vara en logaritmiskt avtagande funktion som gäller för koldioxid. Runt 600 ppm är det knappast någon effekt kvar, som kan ge klimatpåverkan.

    Mänskligheten bör inte se koldioxiden som ett hot utan ta till sig att det är ”livets gas” och glädjas åt att den ökar skördarna och ger oss en tryggare matförsörjning.

  100. Adepten

    #79 Munin

    ”Var vill gravitationen placera denna massa?”
    Kul fråga 🙂 Om vi antar att vi har 400 ppm CO2 molekyler och inte har någon konvektion eller vindflöde av luft åt något håll i atmosfären skulle CO2 sakta sjunka till marken och bilda ett ca 3 meter högt skikt. Verkar inte vara så nyttigt för en människa i de läget 😉

    Förövrigt tycker jag att detta inlägg och dess kommentarer om växthuseffekten var intressant och gav upphov till många funderingar.
    En fråga från dessa funderingar är hur många procent av växthuseffekten som CO2 svarar för? Jag har sett siffror på 2,75% eller 3,3 W/m2 ?

  101. Björn

    Nu handlar ju det här inlägget om växthuseffekten, men ytterst har vi ingen växthuseffekt utan det kortvågiga inflödet av solenergi. Om den avtar, avtar också utstrålad IR i intensitet. Solenergi och växthuseffekt går helt enkelt hand i hand. Problemet här är att IPCC har gjort det lätt för sig och behandlar solen som en konstant, men dess effekt, TSI, varierar både på grund av planetära avståndsvariationer och variation i dess egen utstrålning. Vi har två variabler som är solen och växthusgaserna. Hur kan man med vetenskaplig ärlighet påstå att man förstår klimatsystemet, när man utesluter en så viktig variabel som solen? Det är först med satelliterna som man har kunnat studera solens spektrala innehåll, där variationen i UV-området är betydande. Huvuddelen av UV filtreras ut av atmosfärslager ovanför troposfären, där stratosfären är en stor mottagare av energi från UV. Utöver 11-årscyklerna, har vi att ta hänsyn till, som nu, även ett reducerat inflöde av energi under hela solfläckscykler. Vi har också att ta hänsyn till att den globala temperaturen varierar med dessa två variabler, solen och växthusgaserna, men IPCC och dess klimatforskare, utesluter solen som en drivande faktor.

  102. TorbjörnR

    #95 Ingemar Nordin

    Jag tycker att Gösta, GP, framförallt argumenterar för att de 32 graderna mellan -18 och +14 grader Celcius är en gravitationseffekt och inte som bla IPCC verkar tro en effekt av växthusgaser.

    Är de en effekt av växthusgaser så borde också en ökning av tex co2 ge ytterligare uppvärmning enligt logaritmisk samband.
    Tex ökning från 300 till 600 ppm ger 1-2 grader C i ökning beroende på klimatkänsligheten.

    Är det som GP tror att de 32 graderna har lite med växthusgaser att göra så är det rimligt att tro att de är av ”insignifikant” betydelse om co2 ökar till 500 ppm.

    Vore ju bra om flera kan enas på den punkten att GPs teori så långt kan stämma! För det är här debatten står idag och de senaste 50 åren.

    Vad som sedan på 1000-tals år förändrar temperaturen är nästa fråga.

    Så, håller du med GP om de 32 graderna?

  103. I added a post on quality of heat energy and knowledge
    https://claesjohnson.blogspot.com/2022/11/quality-of-heat-energy-vs-radiative.html
    Hope it can help increasing knowledge.

  104. tty

    #102

    ”Jag tycker att Gösta, GP, framförallt argumenterar för att de 32 graderna mellan -18 och +14 grader Celcius är en gravitationseffekt och inte som bla IPCC verkar tro en effekt av växthusgaser.”

    Och genom vilken mekanism skulle gravitationen höja temperaturen med 32 grader?

    Gravitationseffekten förklarar varför temperaturen i atmosfären sjunker med höjden, men inte varför det inte är -18 vid markytan.

  105. Adepten

    #102 TorbjörnR

    ”Är de en effekt av växthusgaser så borde också en ökning av tex co2 ge ytterligare uppvärmning enligt logaritmisk samband.
    Tex ökning från 300 till 600 ppm ger 1-2 grader C i ökning beroende på klimatkänsligheten.”

    Eftersom nuvarande CO2 endast utgör 2,75 procent av växthuseffekten på 32 grader kommer en ökning av CO2 från 300 till 600 ppm att ge 0,11 graders uppvärmning.
    Således en liten ökning vid en fördubbling av CO2 🙂

  106. Torbjörn 102,

    Javisst. Det är självklart att den effekt som Maxwell pekar på har en stor betydelse för temperaturgradienten. Men, som sagt, jag förstår inte hur Gösta kommer fram till slutsatsen att växthusgaser (och annat) inte skulle vara ”signifikant”. Klimatdebatten handlar ju huvudsakligen om Climate Change, så därför är det konstigt att utelämna dessa centrala frågor i klimatdiskussionen.

    Sedan kan man ju diskutera X-faktorernas betydelse för de grundläggande 32 graderna och utseendet på temperaturgradienten. Men det är inte det som jag vill kritisera Gösta P för här utan för hans obefogade slutsats att växthusgaserna inte är signifikanta variabler för att förstå Climate Change. Det följer inte av hans resonemang.

    Vi tycks emellertid vara helt överens om att gravitationen inte kan tillföra värme till systemet, ty annars hade vi haft en perfekt perpetuum mobile där planeter kan värma sig själva utan energitillskott utifrån.

  107. tty

    #100

    ”En fråga från dessa funderingar är hur många procent av växthuseffekten som CO2 svarar för? Jag har sett siffror på 2,75% eller 3,3 W/m2 ?”

    Det är betydligt mera. Det är svårt att säga en exakt siffra, eftersom det helt hänger på hur mycket vattenånga det finns i atmosfären. I tropikerna är det storleksordning 20-25 %, i Arktis upp mot hälften.

  108. Gösta Pettersson

    #67 Leif Åsbrink

    Det var intressant att få ta del av din föreställningsvärld och syn på verkligheten. Jag går direkt till femte stycket nerifrån i ditt inlägg:

    ”Fig 2 i din presentation tycker jag är en överförenklad bild av Manabes figur 16. Att approximera med räta linjer tycker jag inte är OK.”

    Fig 2 är ritad efter en scannad bild av Manabes figur 16 med hjälp av Adobe Illustrator som har en funktion som ritar av scannade förlagor. Fig 2 är förenklad (för inläggets vidkommande ointressanta delar har raderats). Den innehåller kurvavsnitt (rött och blått) vilka inte approximerats som linjära utan är linjära, alldenstund konvektiv jämvikt anses råda så att temperaturen avtar linjärt med ökande höjd. Den innehåller kurvavsnitt (grönt) där radiativ jämvikt anses råda, så att temperaturavtagandet inte längre är linjärt. Vilket jag explicit påpekar i min text.

    Det är möjligt att att Manabe i sin figur approximerat svagt krökta gröna kurvavsnitt med räta linjer. Det är möjligt att Illustrators algoritmer genomfört en sådan approximation. Det tycker jag är OK. Men att obefogat söka skapa misstro till en inläggsförfattare tycker jag inte är OK.

    ”Nyckeln till din bristande förståelse finns här: + citat. Detta förutsätter att utstrålningskiktets höjd inte påverkas av CO2-halten. ”

    Jag tycker snarare att det är du som visar brist på förståelse av Maxwell-skolans argument. Slutsatsen att temperaturskillnaden mellan utstrålningsskikt och jordytan utgör en gravitationseffekt utan radiativa inslag, samt följdslutsatsen rörande den naturliga växthuseffektens insignifikans, kan dras så snart man placerar utstrålningskiktet för 300 ppm CO2 inom det område där konvektiv jämvikt anses råda. Och där placerar Manbes modell utstrålningsskiktet.

    ”MEN det är just utstrålningskiktets höjd som stiger med ökande CO2-halt. …
    Lite högre höjd för utstrålningskiktet och oförändrat laps rate ger 1,4 grader högre marktemperatur vid fördubbling av CO2 enligt Manabe och Wetherald”

    Du ger en felaktig beskrivning av Manabes arbete. Han förklarar växthuseffektens uppkomst som ett resultat av att det uppstår radiativ jämvikt på höjder över 8 km. Han kalkylerar parallellförskjutningen av den konvektionella temperaturgrafen utgående från vad denna radiativa jämvikt föreskriver. I de kalkylerna ingår inga som helst antaganden om utstrålningsskiktets höjd eller variation med CO2-halten.

    Så underkänner du Manabes kalkyler, men gillar hans idé om att det kan uppstå en växthuseffekt genom parallellförskjutning av den konvektionella temperturgrafen? Då får du först dödförklara Manabemodellen.

    Sen befinner du dig i samma läge som #7 tty, enligt mitt svar till #7 i #17. Vad jag efterlyser är empiriska belägg för att utstrålningsskiktet signifikant förhöjts med den industriella tidens stigande lufthalter av CO2. Samt en teoretisk förklaring till hur ökande CO2 kan åstadkomma en ökning av utstrålningsskiktets höjd.

    Här är jag tvungen att bryta, men återkommer troligen i natt med ytterligare några synpunkter.

  109. Adepten

    #107 tty

    Jag hittade uppgifterna i boken The Inconvenient Skeptic: The Comprehensive Guide to the Earth’s Climate av John H Kehr

    Han uppskattade att vattenånga bidrog med ca 5 grader till växthuseffekten och avdunstning till 22 grader

  110. tty

    #109

    ”vattenånga bidrog med ca 5 grader till växthuseffekten och avdunstning till 22 grader”

    Avdunstning producerar ju vattenånga och all vattenånga i atmosfären kommer ju från avdunstning så ovanstående är svårbegripligt.

  111. tty

    Att växthuseffekten i allmänhet och koldioxiden i synnerhet spelar en mycket underordnad roll i de långsiktiga klimatförändringarna håller jag helt med om.

    Det framgår ju mycket tydligt inte minst av att skillnaden i koldioxidhalt mellan ett istidsmaximum och maximal värme under en mellanistid är betydligt mindre än en fördubbling.

    Samtidigt är den globala temperaturförändringen minst 6-7 grader. Vilket skulle förutsätta en ECR i trakten av 10 grader/fördubbling.

    Dessutom styrs ju istidscykliciteten tydligt av Milankovichkurvorna, och koldioxiden kan ju inte påverka jordbanan, medan motsatsen är fullt möjlig. Och koldioxidförändringarna släpar ju också regelmässigt efter temperaturen.

    Man kan alltså lugnt konstatera att det primärt är förändringarna i solstrålningen som är huvuddrivkraften i glacialcyklerna. Dessa förstärks sedan av albedoåterkopplingar och kanske i någon mån också av växthusgaser.

    Men det finns mycket som tyder på att ytterligare andra faktorer också är viktigare än växthusgaser. De mycket abrupta klimatomslag som händer under istider kan t ex inte förklaras av någon av de tidigare nämnda faktorerna, och inte hellar förklarar de varför det numera bara är varannan eller var tredje oblikvitetscykel som leder till en mellanistid.

    Det finns många hypoteser, men ingen som är helt övertygande.

  112. TorbjörnR

    #104 tty

    Förstår inte din fråga men GP har väl förklarat det flera ggr bla i #61

  113. TorbjörnR

    #105 adepten

    Vart hittar du dina siffror? Verkar ju som någon räknat väldigt noggrant 😄

  114. Munin

    # 88
    Finns det uppgifter om mängden vatten, som i genomsnitt finns i jordens atmosfär? Om så finns det också hypoteser om hur den mängden varierar över tid? Både vad gäller kortare cykler som dygn, år och längre cykler inkl. istider.

  115. Bubo

    Intressant. Denna gravitationseffekt, förklarar den också lika väl hur det är på andra planeter. Tänker kanske främst på Venus som ju är hyfsat lika stor men har en helt annan sammansättning på sin atmosfär?

    Den frågan blir väl då främst riktad till Gösta P.

  116. GoranA

    Med anledning av att Milankovitch kommit upp ett par gånger

    Läser nu en gratisbok på i pdf-format av Javier Vinos med förord av Judith Curry. Finns att hämta på
    https://wattsupwiththat.com/wp-content/uploads/2022/11/Vinos-CPPF2022.pdf

    Boken börjar med att behandla Milankovitch cyklerna och deras kopplingar till glacialer och interglacialer.

    Enligt boken om jag förstår det rätt så är en ökande jordaxellutning och med en vinkel överstigande 23° en förutsättning för en interglacial vilket förekommer en gång per 41000 år,

    Till detta krävs också att det finns en utbredning av inlandsisar över ett tröskelvärde på norra halvklotet och att instrålningen är mycket hög på norra halvklotet under sommaren, alternativt att inlandsisarna har en extremt stor utbredning och att instrålningen under sommaren når ett tröskelvärde.

    Instrålningen beror kombinerat på jordbanans excentricitet och på precisionen..

    Detta gäller för interglacialer som uppkommit under den senaste miljonen åren med en period på 80-120 tusen år.

    Som tidigare nämnts (tty) när interglacialerna kom med en period på ungefär 41000 år var jordlutningen (oblikviteten) den helt dominerande faktorn.

  117. Adepten

    #113 TorbjörnR

    Se här #109 Adepten
    https://klimatupplysningen.se/vaxthuseffekten-enligt-en-av-2021-ars-nobelpristagare-i-fysik/#comment-428477

  118. Adepten

    #110 tty

    Vattenånga som blivit moln och vatten som avdunstat eller konvektion

  119. Lars-Eric Bjerke

    #65
    Gösta Pettersson
    ”Men planetariska kroppars atmosfärer är inte isotermiska utan uppvisar konstanta temperaturgradienter så snart trycket överskrider ca 0,1 bar.”
    Det beror på att atmosfären kring dessa värms underifrån via ledning och konvektion i och med att solen värmer planeten.

  120. Mikael Andersson

    #116, #111

    Precis! Logiskt…to say the least, science is now settled about climate change…(in my mind) 😎
    Det krävs större mekanismer än kor som fiser för att påverka ett klimat större än en ladugård!

  121. Daniel Wiklund

    #119 Lars-Eric B Just nu värmer inte solen planeten mycket, i varje fall inte i Norrbotten. Det brukar vara så att när det hålls klimatmöten och global värme står på dagordningen brukar det bli kallt.

  122. tty

    #118

    Men vatten som kondenserat till moln är ingen växthusgas.

  123. tty

    #119

    ”Det beror på att atmosfären kring dessa värms underifrån via ledning och konvektion i och med att solen värmer planeten.”

    Strålning och konvektion. Ledning är nästan försumbar mekkan marken och atmosfären.

  124. Gösta Pettersson

    #105 Adepten, även #109 Adepten
    ”växthuseffekten på 32 grader”

    Du defierar begreppet ”växthuseffekt” på helt annat sätt än vad IPCC och jag definierar det som.

    Förutsätter man att den 32-gradig atmosfärseffekten är en växthuseffekt, så lär man inte kunna komma fram till något annat än att den är en växthuseffekt. Förmår man inte skilja mellan begreppen ”växthuseffekt” i radiativ bemärkelse och ”atmosfärseffekt”, så har man inte skugggan av en chans att förstå Maxwellskolans argumentering. Vilken går ut på att den 32-gradiga atmosfärseffekten helt kan förklaras som en gravitationseffekt. Jordytan är ”precis” så varm som den kan förväntas vara för en planet på Jordens avstånd från solen. Det finns inget som tyder på att den är varmare än det förväntade värdet. Dvs det finns ingenting som tyder på att den sigifikant värmts upp på grund av radiativa växthuseffekter.

  125. Gösta Pettersson

    #115 Bubo

    På klimatbloggar har jag träffat på åtskilliga diskussioner av Venus heta yttemperatur. GHE-proponenter hänför den till vad de med barnkammarspråk kallar en galopperande växthuseffekt som intuitivt förmodas uppstå på grund av att ytvärmen inte kan komma ut genom Venusatmosfären med dess höga CO2-halt. GHE-opponenter hävdar att de kan förklara Venusatmosfärens temperaturgradient som ett resultat av den gravito-termala effekten. Agumentationerna kompliceras av att endast cirka 1% av solljuset når ner till Venusytan. Samt av att CO2 vid de temperaturer och tryck som råder neråt ytan intar ett udda aggregationstillstånd som kanske inte helt beter sig som en gas.

    Tyvärr har jag inte kunnat googla mig fram till några seriösa vetenskapliga diskussioner av problemet. Man får nog leta på universitets- eller institutionsbibliotek för att hitta sådana.

  126. Gösta Pettersson

    #119 Lars-Eric Bjerke
    ”Det beror på att atmosfären kring dessa värms underifrån via ledning och konvektion i och med att solen värmer planeten.”

    Ja, men det är väl precis vad även Maxwellskolan säger. Solen värmer planetytan som värmer den marknära luften, varefter strålning och konvektion (cf #119) får värmeöverskottet att omsider strålas ut mot rymden. Varvid det (som Loschmidt visade) kan förväntas uppstå en atmosfärisk temperaturgradient med linjärt avtagande temperaturer på grund av den gravito-termala effekten.

  127. Gösta Pettersson

    #104 tty
    #106 Ingemar Nordin

    Skam den som ger sig. Ännu ett försök att förtydliga Maxwellskolans argument.

    Det tycks råda stor enighet om att den temperatur som skapar balans mellan in- och utstrålning för en atmosfärsfri Jord är -18 °C. Det finns ringa anledning att ifrågasätta IPCC:s bedömning att Jordytans globala temperatur 1989 var 14 °C. Alltså föreligger det en atmosfärseffekt som gör jordytan 32 grader varmare än den balansskapande utstrålningstemperaturen -18 °C.

    Vad bidrar till denna 32-gradiga temperaturskillnad?
    Växthusgasers uppvärmning av jordytan måste beaktas säger IPCC. Den gravito-termala effekten måste beaktas säger Maxwellskolan. Då döper vi det förra bidraget till VÄXT och det senare till GRAV, vilket ger oss ekvationen
    VÄXT + GRAV = 32

    Bidraget GRAV är lätt att beräkna, alldenstund den balansskapande temperaturen -18 °C återfinns i de troposfäriska skikt där det råder konvektiv jämvikt enligt allmän mening sedan tidigt 1900-tal. Och i sådana troposfäriska områden föreligger det en gravito-termal temperaturgradient karakteriserad av Loschmidts ekvationer. Vilket innebär att temperaturskillnader mellan olika luftskikt inom området bestäms av temperaturgradientens lapse rate och är av gravitationellt ursprung (t. o. m. proportionella mot gravitationskonstanten).

    Det gäller speciellt även den 32-gradiga temperaturskillnaden mellan utstrålningsskiktets -18 °C och den marknära luftens/jordytans 14 °C. Så GRAV = 32.

    Då sätter vi in det i ekvationen ovan och får VÄXT = 0. Vilket jag med befogad språklig försiktighet översatt till att växthuseffekten befunnits vara av insignifikant storlek.

    Därför är Maxwellskolans påpekande att den 32-gradiga atmosfärseffekten tillfredsställande kan förklaras vara en gravito-termal effekt extremt betydelsefullt. Det leder till följdslutsatsen att den naturliga växthuseffekten inte kan vara av signifikant storlek.

    Så, tty, den mekanism som höjer jordytetemperaturen med 32 °C gentemot utstrålningsskiktets -18 °C är gravitationen enligt den av Loschmidt karakteriserade gravito-termala effekten. Jordytans temperatur är inte -18 °C eftersom utstrålningsskiktet hamnat en bit upp i troposfären och jordytan därför på grund av den gravito-termala effekten måste uppvisa en högre temperatur.

    Och Ingemar, jag hoppas att beräkningsutfallet VÄXT = 0 ovan får dig att inse varför jag betraktar växthuseffekten som en ”nullitet”. Hade det förelegat någon signifikant förhöjning av jordens yttemperatur på grund av den naturliga
    växthuseffekten, så skulle den ha avslöjat sig i ovanstånde ekvation och uteslutit att GRAV = 32. Resultatet VÄXT = 0 representerar ett väsentligt vetenskapligt framsteg. Om VÄXT = 0 år 1989, så lär den även ha varit det miljontals år tidigare. Det tillåter dig att bland alla faktorerna X bakom de temperaturmässiga variationer (istider, interglacialer etc) som du betrakta som klimatologiska huvudproblem utesluta växthuseffekter som en faktor av signifikant betydelse.

  128. Gösta Pettersson

    #7 tty
    #67 Leif Åsbrink

    Min tids- och orkmässiga kapacitet är starkt begränsad. Återkommer senare idag med utlovade komplementerande synpunkter på era kommentarer.

  129. Claes Johnson

    Bra Gösta! Tilläggas kan kanske att det är vattenånga som dominerar koldioxid som aktör (tillsammans med gravitation) vad gäller totala atmosfärseffekten med en faktor minst 10, så att av de 32 C högst 3 C kan tilldelas koldioxid varav en fördubbling med logaritm kan ge högst 1 C vilket icke är märkbart/mätbart. Tricket att blanda koldioxid med vattenånga är förvillande och är grunden i CO2hysterin. Det finns ingen evidens (teori+experiment) som visar att CO2 styr jordens medeltemperatur. Ingen. Om någon har sådan vill jag gärna ha referens. Däremot har moln stor inverkan.

  130. Lennart Bengtsson

    Gösta Pettersson

    Detta klargjorde Manade tydligt i sitt arbete från 1967. Det är främst strålning och lapse rate som bestämmer marktemperaturen

    Att atmosfärens lapse rate bestäms av gravitation (g) och atmosfärens värmekapacitet (cp): g/cp) har varit allmänt känt sedan 1800-talet och finns självklart med i alla modellberäkningar för de senaste 100 åren ( dock inte hos Arrhenius)

    Att CO2 genom sin betydligt längre uppehållstid än vattenånga har en drivande effekt framgår till exempel av följande artiklar:
    Held and Soden Journal of Climate 2006

    Lacis et al Tellus 2013

  131. Claes Johnson

    Lennart 130

    Vilka experiment/observationer, som stödjer vad Du påstår, vill Du hänvisa till? Endast teori räcker inte! Please.

  132. tty

    #127

    ”Jordytans temperatur är inte -18 °C eftersom utstrålningsskiktet hamnat en bit upp i troposfären och jordytan därför på grund av den gravito-termala effekten måste uppvisa en högre temperatur.”

    Och vad är det då som gör att ”utstrålningsskiktet” hamnar en bit upp i atmosfären? Just det, växthuseffekten.

  133. Gösta P #127,

    Tack för att du är ihärdig. Men bara för att Maxwell-spåret kan bidra till en förklaring av den uppmätta lapsraten och 32-grasersdifferensen så följer inte att växthusgaserna är uteslutna i en sådan förklaring. 32-gradersgapet och lapsraten är empiriskt bestämda och därmed måste alla faktorer inom det jag kallat mängden X ovan ha bidragit till dessa mätningar – inklusive växthusgaser.

    Hursomhelst vidhåller jag att fokus för klimatdebatten är faktorerna i X snarare än gravitationseffekten. En del av dessa faktorer i X går att beräkna medan andra är mer gissningar. Hur stor del av uppvärmningen sedan 1850 beror på CO2? Här kan man få svar alltifrån 0,3 till 1,1.

  134. Munin

    # 133
    Detta att globala medeltemperaturen ökat sedan 1850 är inget axiom. För det första fanns det inte förrän på 1950-talet så pass mycket mätstationer på södra halvklotet att det gick att med någon säkerhet söka beräkna en global medeltemperatur, På norra halvklotet har vi sedan ännu inga förklaringar till en uppgång från 1850-talet fram till 1930-talets höga temperaturer, som väl främst mätserier från USA och även Sverige indikerar. Sedan sjönk temperaturen fram till 1970-talet och det larmades om en ny istid. Sedan runt 1980 började dagens klimathysteri växa fram och koldioxiden pekades ut som mänsklighetens ödesfråga. För att få något överhuvudtaget att skrämmas med började det med att nu ska vi starta med 1850-talet. Detta utnyttjades särskilt i den s.k. Parisöverenskommelsen och 1,5 gradersmålet. Hade de valt att starta på 1930-talet hade de kunde pratat om 0 gradersmålet, men det ligger så nära i tiden att ännu levande människor kan vittna om att då var det ingen klimatkris. Viktigast av allt är dock att temperturuppgångarna och nedgångarna, som kunnat mätas upp, i alla fall regionalt, stämmer så dåligt in på att det skulle vara koldioxid, som avgörande faktor för hur medeltemperaturen blir över tid. Sedan kan man också fråga sig varför vi ska söka det ideala klimatet och påstå att det fanns i mitten av 1800-talet med sina missväxtår och umbäranden.

  135. forts 131

    Det verkar inte som Lennart som en av världens främsta klimatexperter vill/kan visa vilka experiment/observationer som stödjer hypotesen att dubblad CO2 i atmosfären kommer att ge märkbar/mätbar global uppvärmning. Jag drar slutsatsen att några sådana inte finns. Vad säger KU?

    I brist på exp/obs laborerar IPCC med begreppet Radiative Forcing RF och påstår att CO2 under industriell tid bidragit med 3.3 W/m2 som om de 170 (alt 240) W/m2 från Solen fått hjälp att värma Jorden från lite CO2. Kan det vara så? Vad säger KU?

  136. Gösta Pettersson
    Jag vill tacka för Ditt tålamod och Ditt pedagogiska nit att förklara GRAV (enligt Din terminologi i #127). En viktig bit, som Du anser självklar är kanske inte självklar för alla:
    GRAVs kinetiska/termiska förankringspunkt i atmosfären är principiellt atmosfärens masscentrum, inte dess ’jämviktsstrålningspunkt’. Masscentrum finns vid pass 5 kms höjd. Gravitationens verkan förefaller också delvis oklar att döma av kommentarer. Den åstadkommer naturligtvis ingen ’värme’ utan omfördelar den kinetiska energin under iakttagande av energikonservering. Masscentrums temperatur är beräknad med den generella våt-lapse ratens värde 6,5 K per km: 288,4-5,1×6,5=255 K, samma värde som ansätts för jämviksstrålningspunkten.
    Varför är distinktionen viktig? Ett exempel är att vi kan enkelt approximativt beräkna vattnets i atmosfären temperaturpåverkan på jordytan.
    Om vi tar bort de 1% vatten från atmosfären, som ingår i den generella våt-lapse ratens värde, erhåller vi torr-lapse rate om 9,8 K/km. Tankemässigt ersätter vi vattnet med lika många mol kväve. Vi får då ringa påverkan på masscentrums läge i atmosfären. Schablonmässigt blir då vattnets temperaturverkan: 5,1(6,5-9,8)=ca -17K, dvs jordytans temperatur skulle vara 288+17=305 K utan vattnets avkylande effekt enligt GRAV.

    I anslutning till Claes Johnsons terminologi i hans utomordentligt intressanta nyparadigmatiska skrift ’Mathematical Physics of BlackBody Radiation’ angående våg/resonansfunktioner och härledda cut-off frekvenser för att förstå strålningens effekter, vill jag tillägga att en enkel applikation härav kan visa att ökande koldioxidhalt inte kan ha någon påverka på jordytans temperatur. Jag har i tidigare kommentarer på klimatupplysningen framfört detta med en liknande terminologi:
    Enligt Wien’s Displacement Law är den strålningsmaximala våglängden vid -60° C 13,595 mikrometer och vid +60°C 8,698 mikrometer. Temperaturspannet +/- 60° får anses motsvara normaltemperaturintervallet på jordytan, med uppenbara undantag (Antarktis). Dessa våglängder motsvaras av enkelt beräkningsbara frekvenser, maxfrekvenser eller cut-off frekvenser. Endast frekvenser överstigande dessa (lägre våglängd) vid resp temperatur kan åstadkomma en höjning av jordytans temperatur enligt termodynamikens andra lag. Enligt Johnsons terminologi blir frekvenser under cut-off inkommande=utgående och därför utan temperaturverkan genom konduktiv värmespridning. Det betyder att koldioxid som har lejonparten av sin absorptions/emission kapabilitet vid 14-16 mikrometers våglängd inte kan öka påverka jordytans temperatur, oberoende av halt, inom de angivna normala temperaturintervallen, eftersom dess motsvarande frekvenser ligger under cut-off i det aktuella temperaturintervallet.

  137. GoranA

    tty #123
    ”Strålning och konvektion. Ledning är nästan försumbar mellan marken och atmosfären.”

    Jag har framfört det innan och gör det igen att jag anser ledning är en underskattad mekanism att överföra energi från jordytan till atmosfären. Jag vet inte under vilka premisser man kan negligera ledning.

    I min värld där det finns en ljusbelyst planet kommer den att värmas ojämnt, finns det en atmosfär kommer den att värmas ojämnt som leder till konvektion vilket ger upphov vind som ökar kontakten mellan atmosfär och jordytan. Vi vet att vindar har en ökande kylande eller värmande effekt viket man kan ta som bevis för en icke oväsentlig effekt av ledning.

  138. Gösta Pettersson

    #7 tty
    #67 Leif Åsbrink

    Men hänvisning till mina kommentarer i #124 och #127 finner jag att Maxwellskolan (på basis av de resultat och teorier som presenterades av termodynamiker på sent 1800-tal) gett en tillfredsställande förklaring till temperaturförhållandena i troposfären. Jordytans och den marknära luftens temperatur är ”precis” den som man kan förvänta för en planet på Jordens avstånd från solen, när hänsyn tas till att utstrålningsskiktet med dess balansskapande temperatur -18 °C hamnat i mellantroposfären så att jordytans temperatur måste vara högre på grund av den gravito-termala effekten. Det finns enligt Maxwellskolan påpekanden inga empiriska data som tyder på att jordytans temperatur är förhöjd på grund av radiativa växthuseffekter.

    Så vad föranleder er två att föra fram spekulationer om att jordytans temperatur förhöjs på grund av det intuitiva förmodandet att ökad halt av växthusgaser ökar utstrålningskiktets höjd? Varför är ni missnöjda med Maxwellskolans för mig tillfredsställande förklaring av de troposfäriska temperaturförhållandena? Vad är det för empiriska observationer ni inte tycker har fått en tillfredsställande förklaring? På vad sätt anser ni kunna presentera en bättre förklaring av sådana observationer?

    Manabe har presenterat en formellt sett godtagbar modell för hur det skulle kunna uppkomma en växthuseffekt. Jag inser att övergången mellan luftskikt där det råder konvektiv jämvikt (större delen av troposfären) och luftskikt där radiativa processer dominerar som temperaturbestämmande faktor (t. ex. stratosfären) inte kan vara knivskarp. Manabe har kanske rätt i att det uppstår en radiativ jämvikt i troposfärens översta skikt. Så vad jag i första hand efterlyser i hans fall är empriska data til stöd för att det troposfäriska temperaturavtagandet uppför att vara linjärt i översta troposfären och uppvisar den av hans modell föreskrivna krökniingen.

    Samma sak gäller ert förslag att det uppstår en växthuseffekt genom att ökning av CO2-halten leder till en ökning av utstrålningsskiktets höjd. Ni har kanske rätt, men jag har inte träffat på några empiriska belägg för att så är fallet eller beräkningar som påvisar att effekten är av signifikant styrka. Leif säger att sådana empiriska data i princip kan tas fram. Då väntar jag med spänning och nyfikenhet på att dessa data tas fram och kan förvandla er spekulation till en empiriskt testbar hypotes.

  139. Gösta Pettersson

    #132 tty
    ”Just det, växthuseffekten.”

    Utstrålningsskiktet hamnar i mellantroposfären eftersom det är där växthusgaserna finns och via utstrålning förmedlar värmetransport från jordyta till rymden. Men att växthusgaser finns i troposfären och deltar i värmetransporten betyder inte att de åstadkommer en växthuseffekt i form av en radiativt betingad förhöjning av jordytans temperatur. Deras närvaro är ett nödvändigt, men ingalunda tillräckligt, villkor för att det ska uppkomma en växthuseffekt.

  140. tty

    #139

    ”Deras närvaro är ett nödvändigt, men ingalunda tillräckligt, villkor för att det ska uppkomma en växthuseffekt.”

    Nej, det behövs också en gravitationsgradient, men eftersom det är en planetatmosfär vi diskuterar så finns det alltid en sådan.

  141. Munin

    # 136
    Masscentrum: Jag utgår ifrån att det avser att gravitationen har en påverkan på molekylerna i atmosfären och skapar en del i atmosfären där densiteten blir högre. Är det vad som avses? Var placerar sig koldioxidmolekylerna och i dessa ingående kol i sammanhanget? Finns det ett masscentrum för koldioxiden, som en delmängd i masscentrum?

  142. Munin #141
    ’Masscentrum’ är helt enkelt den ’punkt’/höjd i atmosfären, där massan ovanför är lika stor som masssan under.

  143. Munin

    # 142
    Om vi avgränsar det till att finna masscentrum för koldioxiden i atmosfären. Var finns den? Var är den punkt/höjd i atmosfären där koldioxidens massa ovanför är lika stor som massan under?

  144. Munin

    # 143
    Tillägg: Logiskt sett måste det finnas ett masscentrum för koldioxiden liksom för övriga gasformiga ämnen i atmosfären. Koldioxiden är dock av extra intresse i klimatfrågan. Då mänsklig verksamhet årligen tillför atmosfären ca 38 miljarder ton koldioxid, varav ca 45-50 % ackumuleras i atmosfären ska det gå att räkna ut hur mycket det höjer koldioxidens masscentrum (se # 91). 19/2 hamnar under och 19/2 hamnar över masscentrum. Om man ritar in masscentrum som en sfär ökar dess yta undan för undan. Motsvarande sfär går också tänka sig längre ut där de sista koldioxidmolekylerna möter rymden. Denna ytan är mycket större än den som definieras av masscentrum och ökar därmed mycket mer. Vad säger strålningsfysiken om vad som händer? Antalet koldioxidmolekyler ökar, men så gör också volymen de svävar i. Vidare ökar ytan mot rymden och därmed ”avsvalningsytan”, ytan där jordens utgående energi förs ut i rymden. Vad blir det för netto av detta i jordens strålningsbalans över tid?

  145. tty

    #144

    ”Då mänsklig verksamhet årligen tillför atmosfären ca 38 miljarder ton koldioxid, varav ca 45-50 % ackumuleras i atmosfären ska det gå att räkna ut hur mycket det höjer koldioxidens masscentrum (se # 91).”

    Fast samtidigt förbrukas ju 28 miljarder ton syre för att producera koldioxiden, och syret är betydligt stabilare i atmosfären, så eftersom hälften av koldioxiden relativt snabbt försvinner så sjunker nog masscentrum snarare.

    .

  146. Munin

    # 145
    Tack för detta påpekande! Då blir det så att gravitationen sänker koldioxidens masscentrum vid ökande halt koldioxid i atmosfären. Samtidigt ska utstrålningsskiktets höjd öka när halten koldioxid ökar i atmosfären. Vad gör att vi får det omvända förhållandet mellan fysisk verklighet och den verklighet, som kan kallas fotonernas värld?

  147. Lennart Bengtsson

    Claes Johnson

    Mängden vattenånga i atmosfären följer temperaturen med cirka 5 %/°C baserat på globala analyser för temperatur och vertikalt integrerat vatteninnehåll. Beräkningen avser globala data för perioden 1979-2021 tillgängliga from EU:s Copernicus data bas och är ett medelvärde för hela perioden. Beräknat från analyserade globala fält vid ECMWF ( ERA5 re-analys) Jag använde den vertikalt integrerade vattenmängden i luften och temperaturökningen på 850 hPa vilken är en medelhöjd för vattenångepelaren. Jag fick samma resultat för det europeiska landområdet. Felintervallet är sannolikt litet då det är ett medelvärde över 43 år. Ändringen är i både fallen räknar från den linjära trenden
    Detta är något mindre än en beräkning baserad på oförändrad relativ fuktighet som ger cirka 7%/°C

  148. Claes Johnson

    Lennart 147

    Jag frågade efter exp/obs som visar att fördubblad CO2 i atmosfären kommer att leda till märkbar/mätbar global uppvärmning. Detta är den helt centrala frågan. Du ger inget svar i Din roll som världsledande expert på jordens klimat. Hur skall jag tolka detta? Finns det inte något sådant? Om inte, så faller väl hela CO2hysterin? Vad är Ditt svar?

  149. tty

    #146

    ”Masscentrum” är knappast ett relevant begrepp i sammanhanget. Koldioxiden är väl blandad i atmosfären. Koldioxidmolekylen är inte så mycket tyngre än kväve och syre att det blir någon stor skillnad i koncentrationen med höjden.

  150. Matz Hedman

    #148 Cleas
    Ibland kan det vara bra att analysera uppmätta data. Om man gapar om att fakta saknas utan att ens läsa eller studera det som inte följer ens egen tes hamnar man ofta i en uppförsbacke.

  151. bubo

    #148
    Först vill jag säga att jag har ”svarta döden” i bokhyllan (computational differential equations) men det börjar bli ett tag sedan jag läste den. Bra bok dock!

    Jag har följt er konversation men måste säga att det är litet svårt att få något grepp på vilken typ av experiment/observation som du efterfrågar. Vi har ju bara en jord/atmosfär och fullskaleexperiment är ju svåra. Man kan ju hävda att det pågår ett fullskaleexperiment nu där det ständigt kommer observationer ifrån men resultaten påverkas ju av många faktorer. Jag har jobbat en del med storskaliga försök för grundvattenflöde och den viktigaste delen i försöksplaneringen är ju att fundera på hur man kan utforma försöket, placera mätning etc för att få tolkningsbara resultat.

    Istället för att fråga Lennart så undrar jag därför: vilken typ av mätningar/observationer skulle du vilja utfördes för att vi enligt dig ska kunna besvara det du kallar den centrala frågan? Hur tydligt måste resultaten vara? Är sådana mätningar praktiskt möjliga att göra? Hur skulle en sådan mätningar ske? Det blir ju enligt min mening meningslöst att ropa efter mätningar om de mätningar man eftersöker är omöjliga att utföra och man kan då inte heller förkasta något på grund av avsaknaden av sådana mätningar (om det finns annat, ex en bra teori, andra mätningar etc som tyder på existensen av det)

  152. Claes Johnson

    Bubo 151

    Jag frågar Lennart som världsledande expert på jordens klimat inkl ev global uppvärmning från CO2, vilka exp/obs det finns som visar att mänskliga utsläpp av CO2 orsakar mätbar/märkbar global uppvärmning. Jag får inget som helst svar. Jag drar slutsatsen att några sådana inte finns. Vilken slutsats drar Du, och KU?
    Och om de inte finns, vad blir nästa slutsats? Skall vi satsa allt för att minska våra utsläpp av CO2, när vi vet att det kommer att dra undan grundförutsättningen för rimligt liv?

  153. Munin

    # 149
    Det är intressant om det finns en direkt relation mellan koldioxids masscentrums höjd och utstrålningsskiktets höjd och hur dessa höjder ändras vid ökande halter koldioxid i atmosfären. Inget av dem är distinkta, men bör kunna fångas i var sin beräkningsmodell. Jag har dock ingen bild av vilka mått, som är aktuella (km, mil?) Det är ett gravitationskopplat mått och ett mått kopplat till strålningsfysiken.

  154. Håkan Bergman

    Munin #153
    Jag tror du tänker fel i fråga om olika atmosfärskikts yta, man måste räkna med skiktens avstånd från planetens centrum, Ett enkelt exempel förhållandet mellan ytan vid 6 resp. 5 km höjd givet jorden radie = 6371 km blir
    6377²÷6376² = 1,00031370088
    Inte mycket att ligga vaken för, eller hur?

  155. bubo

    #152
    Om du efterfrågar mätningar från försök där man dubblat koldioxidhalten i atmosfären, lyckats hålla alla övriga temperaturpåverkande faktorer konstant (solinstrålning, havsströmmar, vulkanutbrott, aerosoler etc) i 50 år för att nå litet jämvikt och sedan mätt påverkan så nej, resultat av några sådana experiment har inte jag heller sett. Jag tror även de skulle vara svårt att passera etikprövning och att rent tekniskt genomföra. Slutsatsen av frånvaron av sådana observationer skulle jag nog dra att det är ingen framkomlig väg att försöka sig på det och att alla redan insett det.

    Jag skulle nog försöka mig på att mäta den temperaturutveckling som sker (vilket ju görs), teoretiskt försöka utvärdera vilka andra faktorer som kan ha påverkat resultatet (vilket görs). Jag skulle nog försöka mig på att skapa en teoretisk modell som kan ge förutsägelser på snävare områden som är betydligt enklare att verifiera med observationer/mätningar men som ändå ger ledtrådar till svaret på den stora frågan. Om modellen klarade det skulle nog min tro på att den även svarar på den stora frågan som var svår att undersöka experimentellt öka. Skulle däremot de snävare mätningarna/observationerna gå in helt annan riktning skulle jag bli skeptisk. Något i den riktningen i varje fall. Det finns väl vad jag vet en hel massa ”snävare mätningar” som gjorts som ligger i linje med en mänskligt driven uppvärmning.

    Huruvida det är tillräcklig för att göra åtgärder är ju mer en politisk bedömning. Dock blir jag nyfiken på vilka observationer/mätningar du lutar dig mot när du säger att vi vet att åtgärder för en minskning av CO2 drar undan förutsättningarna för rimligt liv? Jag har för mig att jag sett någon här på KU dra fram siffran 20 000 miljarder om året i kostnad och det är väl bara ca 2,5 % av världens BNP. Mycket pengar/resurser men inte så att det är tillbaka till stenåldern (mer tillbaka till 2020).

    Sedan tror jag vetenskapen till slut kommer sätta sig
    kring klimatfrågan då Plancks princip ju obönhörligen verkar och vi får nog krasst konstatera att vi som deltar i denna diskussion har kortare väg dit än Greta.

  156. Gösta Pettersson

    #155 bubo

    Det du säger tycks mig vara i linje med vad Claes vill framhålla: Det finns inga empiriska data som trovärdigt påvisar att ökande lufthalt av CO2 signifikant bidragit till det senaste århundradets globala uppvärmning. Och som du påpekar lär det vara synnerligen svårt att få fram sådana data.

    Vad jag själv skulle ha valt att satsa på vore att försöka förklara och modellbeskriva vad som orsakade temperaturvariationerna under förindustriell tid när CO2-halten enligt IPCC var tämligen konstant och låg. Med en sådan förklaring och modellbeskrivning skulle man enkelt kunna påvisa i vilken mån temperaturutvecklingen under industriell tid påverkats av de stigade CO2-halterna.

    De IPCC-stödda klimatmodellerna förmår inte ens tillnärmelsevis beskriva temperaturutvecklingen under årtusendet före industrialismens genombrott. Eller för att inte vara knusslig, temperaturutvecklingen under holocen. Ingen har kunat ge en trovärdig förklaring till de holocena temperaturvariationerna, alldenstund de torde återspegla ett samspel av en mångfald faktorer som vi för närvarande har otillräcklig kunskap om och i stor utsträckning inte ens vet vilka de kan vara.

    De GHE-troende klimatmodellerarnas lösning av det problemet har varit att antaga att i stort sett alla faktor som bidragit till de förindustriella temperaturvariationerna upphörde att vara av betydelse år 1750 eller 1800 eller 1850 eller vilket annat år man nu behagar utgå från i sina försök till modellskrivnngar av den industriella tidens temperaturutveckling. En erbarmerligt usel uttolkning av vad som empiriskt observerats.

    Istället har sagda klimatmodellerare utgått från att temperaturutvecklingen under industriell tid huvudsakligen bestämts av ökningen av luftens koldioxidhalt på grund av dess förmodade framkallande av en temperaturförhöjande växthuseffekt.

    För vars signifikans IPCC som huvudsakligt bevis angett att den 32-gradiga atmosfärseffekten utgör en växthuseffekt. Ett bevis som enligt vad jag anfört i mitt inlägg och mina kommentarer i själva verket utgör ett bevis för att det inte existerar någon växthuseffekt av signifiknt storlek.

    Så för att återknyta till Claes velat framhålla. Det finns inga empiriska belägg för att CO2 på grund av en förmodad växthuseffekt förhöjer jordytans temperatur. Den enda empiriskt belagda observationen är att jordytans temperatur är den som man kan förvänta på grund av Jordens avstånd från solen och existensen av den gravito-termala effekten.

  157. Gösta Pettersson

    #155 bubo

    Korrektion av slutmeningen i min kommentar #156

    Den enda i detta sammanhang intressanta och empiriskt belfästa observationen är att jordytans temperatur är den som man kan förvänta på grund av Jordens avstånd från solen och existensen av den gravito-termala effekten.

  158. Lennart Bengtsson

    Den mest robusta temperaturmätare som finns på jorden är havens temperatur. Det finns numera tillförlitliga mätningar som visar att havens värmeinnehåll har stigit systematiskt de senaste 50 åren. Varför inte föreslå en annan alternativ mekanism som är orsak till uppvärmningen som åtminstone till omväxling kunde debatteras.

  159. Claes Johnson

    Lennart 158

    Hur vet man som världsledande klimatexpert att havens värmeinnehåll (hur mäter man det?) stigit systematiskt (hur mycket?) och att det är mänskliga utsläpp av CO2 som är orsaken? Skulle detta vara hela den vetenskapliga evidensen vad gäller exp/obs för CO2hysterin som hotar hela västvärldens med undergång? Detta är huvudfrågan, eller hur? Härvid behövs ingen ”omväxling” utan full fokus, eller hur? Det gäller västvärldens överlevnad, eller hur?

  160. Claes Johnson

    Bubo 155

    Energiförsörjningen av jordens befolkning kommer till över 80% från fossil energi, och om denna elimineras gäller samma sak för befolkningen.

  161. Björn

    Gösta Pettersson [156]; Jag citerar; ”Den enda empiriskt belagda observationen är att jordytans temperatur är den som man kan förvänta på grund av Jordens avstånd från solen och existensen av den gravito-termala effekten”. Det är ju inte bara jordens avstånd till solen som har betydelse, annat än på lång sikt, men på kort sikt har vi ju solens egen variation i utstrålning och den har ju varit betydande under 1900-talet, ”Modern maximum”. Åt motsatt håll har vi Maunder minimum och Dalton minimum. Maunder minimum var ju Lilla istidens kallaste period som varade ca 70 år, då ett fåtal solfläckar kunde observeras. I nutid är vi inne på den andra solcykeln med låg amplitud, alltså 25 som ser ut att bli lika låg i amplitud som 24. Främst emitterar solen för närvarande mindre energi i UV-området, vilket främst påverkar temperaturen i stratosfären. Vilken betydelse har det för jetströmmarnas förändrade beteenden?

    Klimatforskningen vill naturligtvis undanhålla fakta kring solen när man så hårt har fört fram CO2 som den faktor som driver klimatsystemet mot en global uppvärmning. Solaktiviteten är korrelerad med TSI, vilken är måttet på den irradians som når oss. Läs på Dr. Hathaways webbsite om solens utveckling och speciellt under fliken Solar Cycles.

    http://solarcyclescience.com/

  162. Lennart Bengtsson

    80+ % kommer från fossil energi och cirka 10% kommer från biomassa som också avger CO2 vid förbränning.

    Detta kan knappast ändras i större utsträckning (>25%) före år 2050

    Denna fråga är emellertid helt skild från att växthusgaserna i atmosfären påverkar jordens strålningsbalans. De kvantitativa bedömningar ( med felgränser) som gjorts av IPCC anser jag högst rimliga baserat på dagens kunskapsläge. 1.5 ° – målet kan inte uppnås och inte heller 2°-målet. Snarare är en uppvärmning på 1.5 -2°C jämfört med temperaturen NU mer realistisk

  163. Lennart 162

    Uppenbarligen finns ingen exp/obs som stödjer uppvärmning av mänskliga utsläpp av CO2. Likaväl ställer Du Dig bakom IPCC som säger 1 till 5 C (varför inte 0 till 5 eller -5 till + 5?) Hur kan Du som ledande vetenskapsman göra det? Handlar inte vetenskap vad som genom teori + obs kan sägas om världen? Jag tycker Du har en del att förklara.

  164. Munin

    Ta bort klimatkänsligheten (ECS) från det som påstås höja jordens medeltemperatur. Den delen är att likna vid ett vetenskapligt påhitt. Betänk sedan att vid ca 600 ppm upphör koldioxid i stort sett att fungera som s.k. växthusgas enligt strålningsfysiken. Vidare är det mänskliga bidraget till att höja halten koldioxid i atmosfären så litet att det måste röra sig om flera hundratals år innan vi når 600 ppm. Under den tiden gynnas mänskligheten av att det kommer växa bättre och bidra till högre skördar. Jordens växtmassa ökar också och fördröjer ökningen av koldioxiden i atmosfären genom ökad inlagring av kol. Den lilla temperaturökning det handlar om skulle också ge en temperaturutjämning, vilket leder till att extrema väderhändelser blir färre. En liten temperaturökning gör också att färre kommer att dö av kyla, vilket är mycket vanligare än att någon dör av värme.

  165. tty

    #164

    ”Betänk sedan att vid ca 600 ppm upphör koldioxid i stort sett att fungera som s.k. växthusgas enligt strålningsfysiken.”

    Suck. Nej, det gör den inte. Effekten avtar i stort sett omvänt logaritmiskt. Dock blir avtagandet faktiskt något långsammare vid riktigt höga halter.

  166. Munin

    # 165
    Tack, för reaktionen! Jag syftar på hur mycket ”växthuseffekt” tillkommande koldioxid över 600 ppm kan ha. De redan existerande 600 ppm har väl kvar sin inverkan. Det har ju förts fram i vetenskapliga diskussioner hur effekten blir vid en fördubblad halt koldioxid i atmosfären. Jag har förstått det som att detta visats följa en logaritmiskt avtagande funktion. M.a.o. inverkan är mycket större av de första 100 ppm än de första 100 efter 600.

  167. Grunden för CO2hysterin är en av IPCC påstådd Radiative Forcing från dubblad CO2 av 3.3 W/m2 enligt en påhittad formel, som saknar både teori och observation, vilket beskrivs i denna blog post:

    https://claesjohnson.blogspot.com/2022/08/logarithmic-effect-of-more-co2-without.html

    Läs och begrunda, särskilt Lennart och KU.

  168. Göran 136

    Ditt inlägg är viktigt och visar att mer vattenånga o koldioxid har en avkylande effekt eftersom detta kan ge minskad lapse rate, därför att värmeöverföring via strålning (liksom värmeledning) har en utjämnade effekt på temperaturen. Vi bör alltså ÖKA CO2utsläppen om vi vill hejda befarad uppvärmning. Något att tänka på, särskilt för Lennart och KU.

  169. Adepten

    #166 Munin

    Innan vi kommer upp till 600 ppm har vi gjort slut på både kol och olja🥴. Om vi inte hittar något annat🧐?

  170. Munin

    # 169
    Ja, då har vi ännu mindre orsak att oroa oss för att klimatet skulle gå överstyr! Då ska vi också kunna införa var koldioxidhalten maxar i atmosfären och också vid vilken framtida tidpunkt det kan beräknas ske. Kunniga i strålningsfysiken ska också kunna räkna ut, vilken global medeltemperatur vi då kan förvänta oss. Det bör följa av den funktion, som visar på successivt avtagande ”växthuseffekt” från varje tillkommande ppm koldioxid i atmosfären. Exkludera klimatkänsligheten (ECS) från dessa beräkningar.

  171. Gösta Pettersson

    #161 Björn

    Slutmeningen i min svarskommentar #156 blev inte vettigt formulerad. Korrektionsförsöket i #157 blev inte mycket bättre. Det insåg jag, och då gick jag och la mig. Tankegången jag hade i huvudet var som följer:

    Claes tycktes mig vilja framhålla att vi saknar empiriska data som befäster att det föreligger ett orsakssamband (växthuseffekten) mellan ökningar av luftens CO2-halt och global uppvärmning. Jag ville framhålla vi har en empirisk observation (jordens globala yttemperatur) som IPCC åberopat som generellt förstahandsbelägg för existensen av en stark växthuseffekt, men som i själva verket ger generelllt belägg för att växthuseffekten är av insignifikant storlek.
    Varför jag tycker att växthuseffekten borde vara en av de mindre intressanta faktorerna att beakta vid försök att förklara jordens temperaturutveckling.

    Så visst är alla de av dig nämnda faktorerna intressanta och värda att beakta. Men vad sådana ”naturliga” faktorer beträffar är väl kunskapsläget fortfarande sådant att man kunnat påvisa korrelationer, men i flertalet fall inte kunnat befästa ett orsakssamband. Eller presenterat mekanistiska hypoteser som beskriver de förmodade orsakssambanden.

    Så jag förstår Lennart Bengtssons suck i #158. Om han som proponent av GHE-hypotesen ska kunna göra vetenskapliga framsteg, så måste han ha tillgång till alternativa hypoteser som han kan inrikta sig på att systematiskt försöka falsifiera.

    Min bild av det nuvarande kunskapsläget är att vi fortfarande vet alldeles för lite för att kunna hävda att vi förstår vad som styr Jordens globala temperaturutveckling. Förhoppningsvis kommer vi allt närmre en förståelse. Faktorerna bakom istidernas uppkomst har diskuterats sedan 1800-talet. Vi har skäl tro att Milankovich-cylerna är en faktor av synnerlig betydelse. Men jag har ännu inte sett någon trovärdig spådom om när nästa istid kommer att inträffa.

    Om den inte redan är på gång. Nuvarande interglacials topptemperaturer nåddes som bekant för cirka 6 000 år sedan.

  172. Gösta Pettersson

    #167 Claes Johnson

    Tack för påpekandet. Jag funderade själv på att påpeka detta, men avstod. Stöder man inte temperaturberäkningarna på någon mekanistisk beskrivning av växthuseffekten, så finns det ingen GHE-hypotes att söka invalidera.

    Vad klimatmodeller beträffar är därför min tyngst vägande invändning den som jag reste i ett tidigare KU-inlägg (https://klimatupplysningen.se/var-kunskap-om-framtidens-temperaturer/):

    För att en klimatmodell ska kunna avge vetenskapligt trovärdiga förusägelser måste den ha prognostiskt validerats. Ingen hittills presenterad klimatmodell (av vad slag den än må vara) har ännu kunnat valideras. Vetenskapligt sett har vi inte en aning om hur höga elller låga framtidens temperaturer kommer att bli.

  173. Munin

    # 172
    Situationen är den att de inte har något att validera mot. Södra halvklotet hade så dålig täckning med mätstationer att det inte förrän på 1950-talet var meningsfullt att beräkna en medeltemperatur för det halvklotet. Antarktis fick väl inga användbara mätningar förrän i slutet av 1950-talet. Afrika och Sydamerika hade mycket låg täckning. Serier före 1950 på södra halvklotet är för mycket hittepåuppgifter för kunna användas för seriös validering. Sedan har vi den massering av uppgifter, som sänkt 1800-talet och höjt senare års uppgifter. Sedan har vi också detta med att temperaturtoppen på 1930-talet har tryckts ner och att äldre diagram skiljer sig från senare års. Hur mycket kontamineringen med värmeöeffekter (om de inte korrigeras bort) ger i falsk temperaturökning är heller aldrig klarlagt. Det finns studier som funnit betydande skillnader mellan serier från landsbygd respektive tätorter. Flera gånger har på denna blogg diskuterats Uppsala serien och varför den inte korrigeras då mätstationen så uppenbart under 2000-talet byggts in bland nya hus, asfaltsytor. Ständigt finns denna fråga om det är så att det är koldioxidkurvan det görs anpassningar till? och att det som talar emot det sambandet undertrycks.
    Hoppet står till att satellitmätningarna kommer ge ett stabilt och tillförlitligt underlag mot vilket validering kan göras, men det är väl kort tidsperiod att luta sig mot.

  174. Adepten

    #170 Munin

    Här kan du studera hur det blir fram till 2100 och därefter
    https://www.therightclimatestuff.com/wp-content/uploads/2021/12/Reality-Based-Warming-Potential-of.pdf

  175. Munin

    # 174
    Tack, den bör bli en bra grund för fortsatt analys. Jag läser snabbt som att en höjning från 280 till 560 ppm beräknas höja globala temperaturen med 1,1 grader och då har inte ECS blandats in. Nu är dagens nivå en bit över 400 ppm (och vi har ingen klimatkris om man inte är klimathysteriker) och då motsvarar det en återstående höjning med bara 0,3 grader. Om vändpunkten för den högsta nivån på koldioxidhalten blir 560 ppm är 0,3 grader högre temperatur inget att oroa sig för och kommer drunkna i alla naturliga variationer, som El Niño m.fl.

    Intressant om tungviktare, som tty, Lennart Bengtsson m.fl. vill lämna synpunkter på om studien är rätt ute?

  176. tty

    #173

    ”Antarktis fick väl inga användbara mätningar förrän i slutet av 1950-talet. ”

    En aning bättre är det kanske. Den första stationen på själva Antarktis (Port Lockroy) startade 1944, men lades ned 1962. De äldsta som fortfarande är i bruk är McMurdo och Amundsen vid Sydpolen som startade 1956 respektive 1 januari 1957. Det finns ytterligare några stationer som startades 1957, men alla har inte kontinuerliga data (flera ryska stationer var tidvis övergivna på 1990-talet).

    Orcadas på Sydorkneyöarna startade 1903, men det är diskutabelt om det kan räknas som Antarktis även om den ligger söder om den antarktiska konvergensen.

    Sedan har ju i princip alla expeditioner till Antarktis samlat meteorologiska data, men det gäller då bara ett eller möjligen ett par år på varje plats. I McMurdoområdet finns sådana ”strödata” tillbaka till den första övervintringen i Antarktis 1899. Man måste komma ihåg att veterligen hade ingen människa någonsin satt sin fot på Antarktis före 1895.

    Så, ja, data från 1850 är det inte direkt gott om.

  177. #124 Gösta Pettersson

    Du skriver: ”För säkerhets skull vill jag förtydliga att jag med växthuseffekt fortfarande avser begreppet så som det definierats av IPCC, dvs. som den radiativt betingade förhöjning av jordytans temperatur som förmodas åstadkommas av atmosfärens växthusgaser på grund av deras specifika förmåga att absorbera och återutsända infraröd värmestrålning.”

    Här befarar jag att ett semantiskt problem kan föreligga: ”den radiativt betingade förhöjning av jordytans temperatur”

    Håller du med om att detta är korrekt: ”den radiativt betingade förhöjning av atmosfärens temperatur och den därav följande förhöjningen av jordytans temperatur”

  178. #127 Gösta Pettersson

    ”Bidraget GRAV är lätt att beräkna, alldenstund den balansskapande temperaturen -18 °C återfinns i de troposfäriska skikt där det råder konvektiv jämvikt enligt allmän mening sedan tidigt 1900-tal.”

    Varför tror du att GRAV är oberoende av CO2-halten?

    Och hur kommer det sig att du ställer upp denna formel: VÄXT + GRAV = 32 Jag antar att du menar att båda är konstanter som kan adderas. Varför?

    En korrekt behandling i din terminologi skulle vara

    GRAV(VÄXT)=dT Här är dT skillnaden mellan marktemperatur och -18 grader.
    Sätter du in VÄXT=300, vi antar 300 ppm CO2 så får vi
    GRAV(300)=32
    Nu vet vi från Manabe med flera att
    GRAV(600)=33,4

    (Med laps rate kan du räkna ut hur utstrålningsskiktets höjd ändrats.)

    VÄXT, bidraget från växthusgaser, hänger direkt ihop med utstrålningsskiktets höjd. Du verkar tro att det är konstant vilket betyder att du inte läst min definition av utstrålningsskiktets höjd eller inte tror att det är korrekt. I det senare fallet är den saken som behöver diskuteras.

    Vad bestämmer utstrålningsskiktets höjd? Vi vet ju båda att en del strålning kommer från marken och en del från moln och infrarödaktiva molekyler på olika höjd så något skikt i bokstavlig mening kan det inte vara fråga om.

  179. #81 Ingemar Nordin

    ”En av nobelpristagarna var Aspect som genomförde ett berömt experiment som skulle visa att s.k. sammanflätade tillstånd (en superposition) omöjliggjorde en ren partikelteori med s.k. dolda variabler. Men det gjorde experimentet inte eftersom filtren inte uteslöt att det kunde ske en selektion av mätresultat”

    Har du någon referens till detta? Om du har det så tror jag det är någon gammal och numera falsifierad invändning. Det borde gå att kolla upp.

  180. Gösta Pettersson

    ”177 Leif Åsbrink
    ”semantiskt problem”

    Vad jag ville undvika var att det uppstår missförstånd på grund av att många GHE-proponenter använder ordet växthuseffekt för att beteckna den tankeexperimellt beräknade 32-gradiga atmosfärseffekten eller den empiriskt uppmätta temperaturskillnaden mellan mellan jordytan och utstrålningsskiktet.

    En sådan alternativ användning av ordet växthuseffekt förkommer i Adeptens kommentar #105 och #109. Vilket föranledde mig att i #124 påpeka att Adepten använt ordet i annan bemärkelse än jag. Vad jag menar med växthuseffekt är den ökning av jordytans temperatur som en ökning av halten växthusgaser kan åstadkomma på grund av deras specifika förmåga att absorbera och återutsända värmestrålning. Manabes modell ger exempel på en sådan radiativt betingad växthuseffekt.

    ”Håller du med om att detta är korrekt: ”den radiativt betingade förhöjning av atmosfärens temperatur och den därav följande förhöjningen av jordytans temperatur””

    Nej. I den mån en radiativt betingad förhöjning av atmosfärens temperatur uppstår (t. ex. transient), så leder den inte nödvändigtvis till en förhöjning av jordytan temperatur

  181. Munin

    # 175
    Tillägg: Situationen är faktiskt sådan att vi vid nuvarande nivå med lite över 400 ppm koldioxid i atmosfären får väldigt liten tillkommande temperaturhöjning för varje höjning av koldioxidhalten. Koldioxiden som ett hot mot klimatet finns knappast längre och det beror på att det följer av den logaritmiskt avtagande funktionen för koldioxiden enligt strålningsfysiken! Det är dags att avskriva koldioxiden som ett hot mot klimatet!

  182. Lennart Bengtsson

    Att diskutera vetenskap med de i tron övertygade är omöjligt. det gäller såväl de som tro på jordens undergång inom cirka 10 år liksom de som förnekar naturlagarna. Det senare händer understundom även på denna blogg.

    Att växthusgaser påverkar temperaturen i atmosfären är ett sådant naturvetenskapligt faktum. Det är förvisso svårt att förklarar för människor utan naturvetenskaplig utbildning med det ändrar inte situationen.

    Däremot finns det inga trovärdiga belägg för en förestående kris om man nu definierar kris på det sätt som normalt görs. Man kan förstås kalla det kris och morgonkaffet inte är tillräckligt varmt eller om mobilen har laddats ur eller om man fått en punka på sin cykel. Eller om barnet inte omedelbart hittar sin favoritleksak.
    Det som man däremot med full rätt skulle kunna kalla en kris är landets under senare år vanskötta energiförsörjning

  183. Claes Johnson

    Lennart 182

    Du ses som en auktoritet vad gäller klimatvetenskap. Vad Du säger har stor betydelse. Det västerländska samhället står inför en kris av oerhörda dimensioner i sin marsch mot det fossilfria samhället. Marschen drivs av starka krafter och vetenskapen spelar med när Klimatmålen nu skall uppnås. Grunden är en påstådd Växthuseffekt från mänskliga utsläpp av CO2 som påstås leda till global uppvärmning och som påstås måste stoppas genom radikal reduktion av utsläppen, kosta vad det kosta vill. Marschen måste genomföras även om den medför ofantligt lidande och död. Ingenting är viktigare. Du säger att en sådan Växthuseffekt är ett naturvetenskapligt Faktum. Inser Du att Du därmed ger stöd till Marschen? När jag ber Dig redovisa teori + experiment/observation som stödjer detta av Dig som auktoritet påstådda Faktum, blir Du tyst. Jag tycker att Du har en del att förklara i Din roll som auktoritet, eller har/är Du inte det? Vänligen, Claes

  184. Munin

    # 183
    Om min analys och slutsatser i # 175 och # 181 stämmer ska vi i stort sett kunna avföra koldioxiden, som ett hot mot klimatet. Dess temperaturhöjande inverkan har till största delen redan uppkommit vid nuvarande 416 ppm och vi har nu ingen klimatkris i världen. Kunniga i strålningsfysiken bör kunna räkna ut hur mycket tillförande av mer koldioxid kan påverka och då beakta att det är en logaritmiskt avtagande funktion de ska använda och som de säkert väl känner.

  185. tty

    #184

    ”Kunniga i strålningsfysiken bör kunna räkna ut hur mycket tillförande av mer koldioxid kan påverka och då beakta att det är en logaritmiskt avtagande funktion de ska använda och som de säkert väl känner.”

    Det är tyvärr inte så enkelt. Det är möjligt (men inte enkelt) att räkna ut effekten ”allt annat lika”. Men allt annat kommer inte att vara lika, och att från grundläggande fysik räkna ut den slutliga effekten av en koldioxidökning är i praktiken omöjligt, även om modellbyggarna låtsas att det är möjligt.

  186. Munin

    # 185
    Detta kan vara helt avgörande för om koldioxiden fortsatt ska pekas ut eller inte, som den viktigaste faktorn att styra jordens medeltemperatur. Koldioxiden är klimathotets och klimathysterikernas käpphäst. I studien, se # 174, finns uppgiften att vid en ökning från 280 till 560 ppmkoldioxid i atmosfären beräknas temperaturen öka 1,1 grader. Finns det någon risk att det är stor felmarginal i den uppgiften? Nu ligger atmosfären på 416 ppm och i linje med logaritmiskt avtagande funktion har en större del av de 1,1 graderna redan intecknats, som givande en högre temperatur. Det som skulle tillkomma om vi når 560 ppm har jag skönmässigt skattat till 0,3 grader. 560 ppm ligger så högt att det inte är långt till 600 ppm, som i andra sammanhang pekats att över den nivån blir det så små ändringar att det inte finns skäl att fortsätta finräkna över den gränsen. Till det kommer att, som också förts in frågan, att vändpunkten för koldioxiden ligger lägre än 600 ppm om man räknar på hur mycket fossil energi som finns och takten den förbrukas. En annan källa som varit inne på frågan är William Harper och då arbetat med 400 och 800 ppm. De första 400 minskar strålningsflödet med ca 30 Wm2 och de över 400 med bara ca 3 Wm2 dvs. bara 1/10 av de första 400 ppm. Vi ligger nu på 416 ppm och det måste bli en blygsam höjning, om någon av temperaturen om dessa relationer gäller.

    Om det skulle det gå att få ostridiga beräkningar utifrån strålningsfysiken om hur den logaritmiskt avtagande funktionen bör vara pekar uppgifterna ovan mot att koldioxiden kan avföras, som ett klimathot. Det går inte objektivt hävda att vi idag är i klimatkris och om det blir allmän kunskap att fortsatt ökning av koldioxiden i atmosfären ger en obetydlig inverkan faller hela klimathotet. Det kommer leda till att många kommer distansera från klimathotet och inte minst politiker, som inser att de fortast möjligt bör lämna det sjunkande skeppet, som tagit ombord Parisöverenskommelsen och i Sverige klimatlagen.

  187. Munin

    # 185
    Tillägg: Jag har ingen överblick om det i vetenskapliga alster redan finns de som skapat en sådan funktion. Du skriver att modellbyggarna åtminstone låtsas, som att de har gjort det. Är det deras hemligheter? Borde väl vara offentligt åtminstone i de fall statliga medel givits till modellbyggarna.

  188. Gösta Pettersson

    #178 Leif Åsbrink

    ”Varför tror du att GRAV är oberoende av CO2-halten?”

    Därför att GRAV hänför sig till den gravito-termalt uppkomna temperturskillnaden i luftskikt där temperaturavtagandets lapse rate empiriskt styrkts vara styrda av Loschmidts ekvationer och därmed av luftens värmekapacitet. För torr luft är den senare cirka 1,00 J/(kg K) och återspeglar till 99,975% värmekapaciteterna av de tre dominerande molekylslagen kväve (1,038), syre (0,913) och argon (0,519). CO2 (0,846) bidrar vid 300 ppm obetydligt (0,025%) till torr lufts värmekapacitet.

    Kommer därtill att en ökning av CO2-halten under nutida förhållande är associerad med en motsvarande minskning av syrehalten. Eftersom skillnaden i värmekapacitet mellan CO2 och O2 är ringa (ca 8%), så kommer en ökning av CO2 till 600 ppm endast att leda en förändring av värmekapaciteten (och därmed av GRAV) med cirka 0,003%. Det är skälet till att jag betraktar GRAV som oberoende av CO2 och inte komplicerar diskussionen i onödan genom att ta hänsyn till effekter som man åtminstone i meteorologiska sammanhang kan betrakta som försumbart små.

    ”Och hur kommer det sig att du ställer upp denna formel: VÄXT + GRAV = 32 Jag antar att du menar att båda är konstanter som kan adderas. Varför?”

    Därför att jag betraktar VÄXT och GRAV som oberoende storheter, så att deras effekt kan framställas som additiv. De antar konstanta värden under fixa förhållanden.

    ”En korrekt behandling i din terminologi skulle vara … etc.”

    Du har inte ens korrekt förmått använda dig av min terminologi. Mitt VÄXT är ett temperaturbidrag. Det kan inte anta värdet 600, eftersom dess maximala storlek är 32. Mitt GRAV är inte någon funktion av mitt VÄXT (ej heller av CO2, för den delen enligt vad jag sagt ovan).
    Ditt VÄXT förfaller var en slags indexering av GRAV för att markera vilken CO2-halt GRAV hänför sig. Du kommer fram till att GRAV(300) = 32. Bra! Vi är överens om det som jag och Maxwellskolan påpekat. Varav följer att mitt VÄXT = 0.

    ”Nu vet vi från Manabe med flera att GRAV(600)=33,4”

    Nej, enligt Manabe är GRAV (som du definierar det) fortfarande 32, vilket framgår av hans Fig 16. Vad Manabe påvisar som en möjlighet är att det kan uppkomma en växthuseffekt svarande mot 1,4 grader på grund av förhållanden som inte har ett skvatt med utstrålningsskiktets belägenhet att göra.

    ”VÄXT, bidraget från växthusgaser, hänger direkt ihop med utstrålningsskiktets höjd. Du verkar tro att det är konstant”

    Jag har ingen anledning att uttala mig om utstrålningsskiktets höjd, mer än att påpeka att skiktet faller inom de delar av troposfären där konvektionell jämvikt på goda grunder anses råda. Det är bara skiktets temperatur som är av intresse för min och Maxwellskolan argumentering.
    Jag betraktar efter beräkningsutfallet GRAV = 32 VÄXT som en konstant med värdet 0.

  189. Adepten

    Den här tråden är kolossalt intressant😃
    Två åskådningar, växthuseffektens betydelse i relation till den gravito-termala effektens betydelse för atmosfärseffekten på ca 32 grader.
    Men som jag ser det har växthuseffekten gått segrande ur den striden enligt IPCC och majoriteten av klimatforskarna.
    Där striden nu står är vilken effekt CO2 har på den globala temperaturen i föhållande till alla andra klimatpåverkande parametrar🤔

  190. #180 Gösta Pettersson

    På denna fråga: Håller du med om att detta är korrekt: ”den radiativt betingade förhöjning av atmosfärens temperatur och den därav följande förhöjningen av jordytans temperatur” svarar du så här: ”Nej. I den mån en radiativt betingad förhöjning av atmosfärens temperatur uppstår (t. ex. transient), så leder den inte nödvändigtvis till en förhöjning av jordytan temperatur”

    Det är ett mycket märkligt svar tycker jag. Självklart avser jag inte transient som omedelbart efter en kärnvapenexplosion, vi talar ju om effekten av koldioxid och om ifall en stigande halt efter att ha uppnått fördubbling skulle påverka marktemperaturen.

    Jag formulerar om: Håller du med om att detta är korrekt: ”Växthuseffekten är den radiativt betingade förhöjning av atmosfärens temperatur efter att CO2-halten fördubblats och den därav följande förhöjningen av jordytans temperatur”

    Det är ju en fråga i två led. Var tydlig med var du tycker att felet ligger ifall du inte håller med.

  191. Munin

    # 190
    Måste du inte lägga till vilka fördubblingar det handlar om? 400 till 800 ppm ger ingen större inverkan. Det följer av att en logaritmiskt avtagande funktion gäller för koldioxid.

  192. Daniel Wiklund

    När det är klimatmöte med den globala uppvärmningen på dagordningen brukar det bli kyla och snö. I varje fall i Sverige. Som att vädergudarna vill djäklas lite och påminna dom sk klimatexperterna att klimatet kan vi inte göra så mycket åt. Det är bara att hänga med och anpassa sig.

  193. tty

    #190

    ”Måste du inte lägga till vilka fördubblingar det handlar om?”

    Nej. Omvänt logaritmiskt avtagande innebär att alla fördubblingar har samma effekt. Om det är 100-200, 200-400, 400-800 eller 800-1600 spelar ingen roll, de ger lika stor temperaturhöjning.

    Brasklapp: detta är inte strikt sant vid mycket höga halter, typ tusentals ppm, där går avtagandet litet långsammare än logaritmiskt.

  194. tty

    #186

    ”Om det skulle det gå att få ostridiga beräkningar utifrån strålningsfysiken om hur den logaritmiskt avtagande funktionen bör vara pekar uppgifterna ovan mot att koldioxiden kan avföras, som ett klimathot. ”

    Jodå, att koldioxidens effekt i det närmaste är logaritmiskt avtagande med ökad halt går att räkna ut med strålningsfysik.

    Vad som INTE går att räkna ut från grundläggande fysik är den totala klimateffekten av en viss halt koldioxid, detta beroende på att vattenångans/vattnets/isens effekt på klimatet är så extremt komplex att man måste ”parametrisera” (=gissa) ett stort antal av de relevanta variablerna.

  195. Munin

    # 193
    Hur stämmer det med William Happers uppgifter? Se Klimatupplysningen 2021-07-09. ”En fördubbling från nuvarande 400 ppm till 800 ppm för CO2 ger exempelvis en mycket blygsam ökning av temperaturen”

  196. tty

    #188

    ”Jag har ingen anledning att uttala mig om utstrålningsskiktets höjd, mer än att påpeka att skiktet faller inom de delar av troposfären där konvektionell jämvikt på goda grunder anses råda. ”

    Faktiskt inte helt sant. I den centrala delen av koldioxidens starkaste absorptionsband sker utstrålningen ovanför tropopausen, alltså stratosfären där konvektionell jämvikt inte råder.

    Om du tittar på strålningskurvan här:

    http://climatemodels.uchicago.edu/modtran/

    ser du en ”pligg” mitt i CO2:s absorptionsband. Den beror på utstrålning från stratosfären.

  197. tty

    #195

    En fördubbling ger ”allt annat lika” en blygsam temperaturhöjning, ungefär 1,3 grader.

  198. Munin

    # 194
    Om hur vatten m.m. påverkas kan vara av olika stor dignitet beroende på hur stor del av energin, som går rakt ut i rymden vid olika ppm-nivåer. Vid högre ppm-nivåer försvinner mycket mer ut i rymden än vid låga ppm-nivåer. Kan man se det som att vid låga ppm-nivåer rör det om mycket mer i atmosfären (vattnets olika faser) än vid de höga och gissandet blir då mindre?

  199. Svend+Ferdinandsen

    #2
    Vi har været der før Claes om tilbagestråling, som selvfølgelig skal betragtes sammen med fremadstrålingen. Varmeoverføringen bliver differensen.

    At det findes og giver mening kan ses af at Jordytan opvarmes af indfaldende kortbølget stråling, og ytan slipper af med varmen gennem infrarød stråling den anden vej. Det samme sker selvom bølgelængderne er tættere på hinanden eller næsten ens.

    Der er mange argumenter mod den overdrevne drivhuseffekt, men at afskrive ”tilbagestråling” er ikke et godt argument.

  200. Mikael Andersson

    Claes Johnsson

    Är detta vedertagna sanningar?

    ”Carbon dioxide, for example, absorbs energy at a variety of wavelengths between 2,000 and 15,000 nanometers — a range that overlaps with that of infrared energy. As CO2 soaks up this infrared energy, it vibrates and re-emits the infrared energy back in all directions. About half of that energy goes out into space, and about half of it returns to Earth as heat, contributing to the ‘greenhouse effect.’”

    Källa: https://news.climate.columbia.edu/2021/02/25/carbon-dioxide-cause-global-warming/

  201. Mikael Andersson

    Och här lite mer specifikt:

    https://earthobservatory.nasa.gov/features/EnergyBalance

  202. Gösta Pettersson

    #190 Leif Åsbink

    Tjöta inte! Du ställde en fråga och fick mitt svar. Om du tycker svaret är märkligt, så är det väl bäst att du själv talar om vad jag borde ha svarat. För jag förmodar att du har nåt på hjärtat som du gärna vill ha sagt.

  203. Gösta Pettersson

    #129 Mikael Andersson

    ”about half of it returns to Earth as heat, contributing to the ‘greenhouse effect.”

    Det är den senare slutsatsen i argumentationen som GHE-opponenter (främst fysiker) de senaste två decennierna riktat mycket stark kritik mot. Det som återvänder till Jorden är inte värme, utan värmestrålning (vilket är en helt annan sak än värme). Värmeenergi kan enligt termodynamikens andra huvudsats endast överföras från varmare materia till kallare. Så länge atmosfären är kallare än jordytan kan det inte ske någon radiativ överföring av värme från luften till Jorden. Det enda som kan uppvärma Jorden är solen. Den värmeenergi som transporterats upp till troposfären kan endast transporteras vidare mot kallare luftlager eller mot kallare delar av universum.

    Ytterst en entropilag som säger att alla spontana processer i naturen leder till en utjämning av föreliggande energiskillnader.

  204. Gösta Pettersson

    #189 Adepten

    ”Men som jag ser det har växthuseffekten gått segrande ur den striden enligt IPCC och majoriteten av klimatforskarna.”

    Det återspeglar förmodligen din i #105 och #109 framförda syn att den 32-gradiga atmosfärseffekten utgör en växthuseffekt. Du har rätt i att IPCC och en majoritet av klimatforskare anser så vara fallet. Men utgår man axiomatiskt från att så är fallet, så lär man inte kunna komma fram till något annat att så är fallet (cirkelbevisning).

  205. Mikael Andersson

    #203

    Tack för svar Gösta!

  206. Mikael Andersson

    Gösta Pettersson

    Och experimentet att belysa med IR ljus 2 pet-flaskor, den ena med luft och den andra med koldioxid, och därmed erhålla mer värme från den med koldioxid stämmer inte alltså?

  207. Claes Johnson

    Gösta 204

    Om KU har någon upplysande roll så bör den vara att upplysa svenska folket och dess makthavare om att det inte finns någon enighet alls bland vetenskapsmän och experter om vad som utgör ”växthuseffekten” och än mindre om hur stor den skulle vara i den mån den finns.

    Speciellt borde upplysas om att det inte finns några vetenskapliga belägg i form av teori + observation som visar att mänskliga utsläpp av CO2 mätbart/märkbart kan ändra jordens medeltemperatur. Men det gör inte KU. Istället underhålls av ledande aktörer inkl LB uppfattningen att visst måste det finnas en ”växthuseffekt” så att visst måste utsläpp av CO2 medföra global uppvärmning så att visst måste utsläppen begränsas. Men det är inte Upplysning utan vilseledande information eftersom vetenskapen saknas.

    KU har en viktig upplysande roll att fylla men gör inte. Västvärlden går mot sin undergång i sin marsch mot ett fossilfritt samhälle påtvingad folket och då motiverad med en av makten påstådd påhittad farlig ”växthuseffekt”.

    Alla krafter på KU måste nu samlas för att upplysa om att det inte finns någon vetenskap alls som visar att denna marsch måste göras, med allt det ofantliga lidande och död den kommer att medföra. Har KU fattat detta? Speciellt bör KU upplysa KVA som spelar med i CO2hysterin.

    Den vetenskapliga dispyten om ”växthuseffekten” går tillbaka mer än 100 år då Knut Ångström visade att Svante Arrhenius beräkning av ”växthuseffekten” var felaktig eftersom A tillskrev koldioxid även vattenångas inverkan. Med Å dog långt före A som tog hem spelet, och samma förvillning råder idag även på KU.

  208. foliehatt

    Hur förklarar de olika skolorna (GRAV repsektive VÄXT) temperaturerna på de tre grannplaneterna Venus, Jorden, Mars – givet de olika avstånden från solen, olika egenrotation och olika atmosfärer?

  209. Mikael Andersson

    #207

    Jag är enig med dig Claes, men att komma till samma slutsats har för mig varit ganska enkelt eftersom jag av min natur ifrågasätter allt.
    Jag är snickare till yrket men tycker att folk i allmänhet är sinnesslöa, vi är ”curlade” i Sverige (gemene man) alltså. Jag tycker att det är ett personligt ansvar att hålla sig informerad i dagens samhälle och söka efter fakta, inte att gå efter vad som ensidigt filtrerats via public service, D.N och aftonbladet.
    Jag hittade tidigt till KU och tycker att här framkommer exceptionellt välformulerade artiklar, teorier och kommentarer. Ett synnerligen värdefullt och entusiasmerande forum enligt mig.

  210. forts 207

    Kungliga VetenskapsAkademien KVA ger sitt stöd till CO2Hysterin på sin hemsida: Vetenskapen säger – om klimatet

    https://www.kva.se/vetenskap-i-samhallet/miljo-klimat-och-energi/

    Jag tar upp denna desinformation, som KVA i vetenskapens namn sprider, i bloggposten KVA Talar till Barnen:

    https://claesjohnson.blogspot.com/2022/01/kva-talar-till-barnen.html

    CO2Hysterin är grundad på historiens största vetenskapliga humbug, som om den inte avslöjas kommer att förpassa västvärlden tillbaka till förindustriell tid kapabel att försörja endast en tiondel av nuvarande befolkning.

    KUs historiska uppgift är att avslöja denna humbug och speciellt Upplysa KVA om Vad Vetenskapen säger och inte säger! Låt oss skrida till verket!

  211. Munin

    # 193, 197, 194, 198
    Enligt den logaritmiskt avtagande funktionen för koldioxid är vi nu nära att tillkommande inverkan av ökad koldioxid i atmosfären blir försumbar. Tillkommande inverkan kan uttryckas, som att den går mot 0. Då den går mot 0 bör också dess inverkan på totala klimateffekten gå mot 0. Om det finns en korrelation mellan de båda och den faktor som initierar klimatändringar står för värdet 0 bör även den totala klimateffekten leda fram till 0. 0 gånger något annat blir alltid 0.

    Min slutsats är att med stöd av strålningsfysiken kan vi nu avföra att ökningar av halten koldioxid i atmosfären skulle vara ett hot mot jordens klimat.

  212. Adepten

    #204 Gösta Petterson

    Jag menar inte att den gravito-termiska effekten är helt fel, men det betyder inte att växthuseffekten är helt fel heller.
    Det kan ju vara så att växthuseffekten och gravitationseffekten gäller samtidigt, men att termiska förändringar i atmosfären på grund av gravitationseffekten har en mindre inverkan på det övergripande termiska klimatförändringarna på planeten. Ställer du upp på den kompromissen 🙂

  213. Håkan

    #197 tty

    Och vi har nått 80% ökning av radiative forcing om alla växthusgaser räknas samman (ej vattenånga) enligt AGGI, så vi kan väl ganska väl skönja vad klimatkänsligheten landar på vid en fördubbling (dock ej den s.k. jämnviktsklimatkänsligheten, vilken vi aldrig lär få något uppmätt svar på).

  214. Gösta Pettersson

    #206 Mikael Andersson

    Jodå, experimentet med petflaskor stämmer säkert; det är en sentida variant av ett experiment som Tyndall utförde redan 1864.

    Men det finns en helt avgörande skillnad mellan atmosfären och växthusgaser som inneslutits i flaskor. Petflaskan håller kvar värmen där den bildades och avsvalkas endast långsamt genom värmeutbyte med omgivningen.

    I troposfären ger lokalt uppkomna värmeöverskott på grund av gravitationen upphov till en konvektion i vertikal riktning som snabbt förflyttar luftpaket och värme mot det jämviktsläge som beskrivs av Loscmnidts ekvationer (linjärt avtagande temperaturer med en lapse rate som är proportionell mot gravitationskonstanten).

  215. Gösta Pettersson

    #207 Claes Johnson

    ”Men det gör inte KU.”

    Enligt min uppfattning gör KU så gott dom kan, bl. a. genom att i år ha publicerat detta och två föregående av mig författade inlägg som lyfter fram den gravito-termala effektens betydelse för jordytans och troposfärens temperaturer.

  216. Gösta Pettersson

    #208 foliehatt

    Betr planetatmosfärer, se mitt svar #125 till #115 Bubo.

  217. tty

    #200

    ”Är detta vedertagna sanningar?”

    Det är, tyvärr, ett vedertaget sätt att förklara växthuseffekten men det är inte korrekt.

    Vad som händer när en koldioxidmolekyl absorberat en foton är att den hamnar på en högre energinivå. Om den finge vara helt ostörd skulle den i genomsnitt bli kvar på den högre nivån relativt länge (med molekylära mått mätt). Men bara en liten del av molekylerna hinner avge energin igen innan de kolliderar med en annan molekyl.
    Vid en sådan kollision sker (statistiskt sett) en utjämning av energinivåerna mellan molekylerna. Den molekyl som får sin energi ökad kan få det antingen genom att molekylen exciteras till en högre energinivå, eller att molekylens rörelseenergi ökar. Men nästan alla (9996 av 10000) kollisioner mellan en exciterad koldioxidmolekyl och en annan molekyl kommer att gälla en kväve- eller en syremolekyl, och dessa har inga tomma energinivåer inom infrarödområdet, så resultatet blir att molekylens rörelseenergi ökar, alltså att den blir varmare.

    Det är denna process (”termaliseringen”) som värmer upp atmosfären. Utan den skulle växthuseffekten inte existera.

    Men det händer ju också att kväve- och syremolekyler med högre energi kolliderar med koldioxidmolekyler med lägre energi, och då kan det även hända att koldioxidmolekylen exciteras till en högre energinivå och alltså kan avge en foton. När termisk jämvikt råder avger koldioxidmolekylerna därför IR-strålning med en intensitet som motsvarar den temperatur som den lokala atmosfären har.

    Denna strålning avges i alla riktningar. Mycket nära jorden kommer en del att nå markytan och en liten del kanske kommer ut i världsrymden, men det mesta kommer att absorberas av andra molekyler. Men ju tunnare luften är, ju större andel försvinner ut i världsrymden.

    Det är därför man genom att mäta intensiteten i IR-strålningen vid olika våglängder kan avgöra på vilken höjd strålningen i genomsnitt har avgetts, förutsatt att man känner till atmosfärens temperaturprofil.

  218. Claes Johnson

    tty 217

    Det är inte strålning som mäts med Pyrgeometer eller Bolometer, utan temperatur (och det är de instrument som finns). Den strålning som dessa instrument påstås uppvisa bygger alltså på att uppmätt temperatur översätts ad hoc till strålning, som då kan vara vad som helst och användas för att skrämmas. Du kan läsa om detaljer under Pyrgeometer på min blogg.

  219. Gösta Pettersson

    #212 Adepten

    Jag har otroligt svårt att kompromissa med min övertygelse. Men jag accepterar och finner det värdefullt för den vetenskapliga processen att alla inte tycker som jag. Och för att stryka dig riktigt medhårs, jag är som avdankad forskare fullt medveten om att skattade värden på storheter inom empirisk vetenskap normalt är försedda med en osäkerhetsmarginal:

    VÄXT = 0 ska egentligen vara VÄXT = 0 (± någonting)

    Men det är väl inget att springa och babbla om när man vill få folk att uppmärksamma att den i nuvarande kunskapsläge bästa skattningen av värdet för växthuseffektens bidrag till jordytans temperatur faktiskt är noll.

    Glad gubbe tillbaka, om jag hade vetat hur man får in såna i texten.

  220. Claes Johnson

    Gösta 215

    Att Du nu 2022 lyckats efter mycket motstånd få in ett bidrag på KU som påtalar betydelsen den gravito-termala effekten (som varit väl känd av insiktsfulla sedan Loschmidt), och att motståndet mot densamma från KUs ledning fortgår, visar inte att ”KU gör så gott dom kan”, detta till skillnad från Klimatrealisterne i Norge.

  221. Mikael Andersson

    #219

    🙂
    Glad gubbe fungerar nog endast när man använder mobilen Gösta, det tangentbordet har en knapp för ”gubbar”,annars får du göra en ”smiley”…..:-)

  222. tty

    #218

    ”Det är inte strålning som mäts med Pyrgeometer eller Bolometer, utan temperatur (och det är de instrument som finns). ”

    Jag antar att du är bekant med dessa spektra:

    https://www.researchgate.net/profile/Fred-Ortenberg/publication/291164378/figure/fig2/AS:648594344390664@1531648350772/Spectrum-of-Earth-Thermal-IR-radiance-recorded-from-space-a-Desert-Sahara.png

    De är alltså mätta av en Nimbus-satellit. Jag undrar hur man bär sig åt att mäta värme från satellitbana. Vakuum är ju inte direkt någon bra värmeledare.

  223. #202 Gösta Pettersson

    Det du skrev är otydligt: ”Nej. I den mån en radiativt betingad förhöjning av atmosfärens temperatur uppstår (t. ex. transient), så leder den inte nödvändigtvis till en förhöjning av jordytan temperatur”

    Du inför ett egendomligt och i sammanhanget ofysikaliskt villkor ”t. ex. transient” vilket får mig att undra om du menar att ”radiativt betingad förhöjning av atmosfärens temperatur ” endast kan ske transcient. Är det så du menar och kan du beskriva vad för typ av händelse som skulle kunna förorsaka detta?

    Jag må ha missuppfattat, men av ditt sätt att formulera dig får jag intrycket att du menar att förekomsten av infrarödaktiva gaser genom sin emission och absorption av infraröd strålning inte förorsakar någon nettotransport av energi från ett volymselement i atmosfären. Dvs att GRAV skulle vara 32 grader även om samtliga växthusgaser ersattes av andra gaser så att värmekapaciteten blev oförändrad men atmosfären skulle vara helt transparent inom det infraröda området.

  224. Håkan Bergman

    🙂

  225. Håkan Bergman

    Alltså mellanslag före och efter 🙂 är viktigt.
    🙂

  226. Håkan Bergman

    Nehej det var det inte, det funkar med ’:)’, men utan ’ alltså.

  227. Håkan Bergman

    Hela listan finns här:
    https://wordpress.org/support/article/using-smilies/#what-text-do-i-type-to-make-smileys

  228. Munin

    # 223
    Utan någon ambition att blanda mig i ert meningsutbyte om vetenskapliga grunder, hur ser du på min hypotes att tillkommande ”växthuseffekt” blir mindre och mindre för varje tillkommande ppm koldioxid i atmosfären och att vi nu ligger så högt i ppm (416) att det är mycket liten inverkan kvar även om vi når uppemot 600 ppm. Eftersom det blir så liten ändring finns det inga andra parametrar som moln m.m. som heller ändrar sig.

  229. Adepten

    #228 Munin

    Här kan du se hur klimatet har varit och vilka framtida temperaturer man kan förvänta sig😁
    https://www.sec.gov/comments/s7-10-22/s71022-20132171-302668.pdf
    Min uträkning blir att temperaturen ökar med 0,5 grader från idag om vi lyckas öka CO2 halten till 600ppm🤔

  230. Munin

    # 229
    Tack! Det måste vara Figure I-3, som visar den logaritmiskt avtagande funktionen för koldioxid, som tty pekat på ska finnas vid ”allt annat lika”. Det finns vid figuren en formel. Är det någon som kan räkna ut vad den ger för siffror till varje stapel i figuren? Det blir då mer exakt och tydligt än att höfta från strecken i figuren.

  231. Håkan Bergman

    Munin #230
    För en ökning från 400 ppm till 600 ppm räknar du så här:
    (5,35÷3,2)×ln (600÷400) = 0,68 grader C.
    Bör gå att skriva in i en hyfsat avancerad kalkylator.
    För varje fördubbling kan du räkna 5,35*0,69 / 3,2 =1,15 där 0,69 = ln2.

  232. Munin

    # 231
    Tack! Vad står ln för?
    Finns det någon som kan räkna fram de troliga ökningarna i ppm i atmosfären baserat på de uppgifter Lennart Bengtsson gav mig tidigare i tråden. M.a.o. om vi antar att vi fortsätter med lika stora utsläpp, som vi gör vid dagens nivå på vår användning av fossila bränslen (allt annat lika). Vid vilka årtal når vi då 500, 550, 600 ppm i atmosfären?

  233. Håkan Bergman

    Munin #232
    ln står för den naturliga logaritmen, kollade i min androidlur och i avanceratläge har den en knapp för ln. Man switchar tii avancerat läge med ikonen i mitten ovanför siffertangenterna.

  234. #228 Munin

    Tillkommande forcing, minskning i utgående infraröd strålning under antagande av ”allt annat lika” är cirka 3 W/m2 för varje fördubbling av atmosfärens CO2-halt. Ett tillskott på 1 ppm ger alltså hälften så stor effekt vid 600 ppm som vid 300 ppm.

    När vi går från forcing till temperatur så kan vi fortsätta att låta allt annat vara lika och då blir temperatureffekten för fördubbling cirka 1,4 grader enligt Happer, Manabe m.fl.

    Nu är givetvis inte allt annat lika. När tillräckligt lång tid gått för att jorden skall ha återgått till att vara i jämvikt med inkommande solstrålning har marktemperaturen ökat lite och hela troposfären blivit lite varmare. Varm luft kan hålla högre absolut fuktighet och det är en självklarhet att totala innehållet av vatten i gasform måste ha ökat. Med hur mycket tror jag inte man vet. Rimligen har molnbildningen blivit lite annorlunda och därigenom jordens albedo. Luftcirkulationen och därmed värmetransporten från ekvatorn mot högre breddgrader har ändrats lite. Den ökade CO2-halten har också gjort jorden lite grönare och därmed sänkt jordens albedo litegrann. Glaciärer har dragit sig uppåt som i svenska fjällvärlden och ytterligare minskat albedot. Det finns massor med små effekter som kan läggas till. Högre marktemperatur påskyndar nedbrytningen av organiskt matrial och ger därmed ett större flöde av metan och koldioxid från våtmarker och jord. Man kan hitta på allt möjligt.

    ECS equilibrium climate sensitivity är definierat som förändringen i temperatur beräknad i en klimatmodell efter att CO2-halten fördubblats och det fördubblade värdet bibehållits under lång tid, hundratals år. Det finns ingen anledning att tro att en fördubbling från 300 till 600 skulle ge en lika stor förändring som den från 600 till 1200 på temperaturen enligt modellerna.

    I verkligheten kommer sannolikt inte CO2-halten att visa sig bli konstant i hundratals år så länge mänskligheten finns kvar.

    En annan sak av betydelse, betrakta figur 2 här: https://co2coalition.org/wp-content/uploads/2021/11/2015-Cohen-Happer-Fundamentals-of-Ocean-pH.pdf Du kan se att mängden CO2 som lösts i havet inte fullt fördubblats när halten i atmosfären fördubblats. En större andel stannar alltså i atmosfären bara på grund av det sänkta PH-värdet. Till det kommer att ett varmare hav löser mindre CO2. Om det krävs X GtC för att gå från 300 till 600 ppm så krävs det betydligt mindre än dubbelt så mycket för att gå från 600 till 1200 ppm i atmosfären. (Det går säkert att hitta siffror på detta – men dom kommer i så fall från teoretiska modeller.)

    600 ppm i atmosfären tror IPCC kommer att bli ungefär maximum och det tror dom kommer att nås ungefär år 2100. Med att IPCC ”tror” menar jag det av IPCC som mest sannolikt bedömda utfallet av det av IPCC som troligast bedömda scenariet när det gäller vår civilisations utveckling. Naturligtvis mycket osäkert – men min personliga uppfattning är att 1200 ppm inte kommer att bli aktuellt de närmaste tusen åren. Möjligen långt in i framtiden ifall man börjar se en ny istid som ett hot.

    Att gå från 416 till 600 är en faktor 1,44. Det är lika mycket som att gå från 290 till 416. Det är denna faktor, ungefär roten ur två som fått IPCC att tro att koldioxiden höjt jordens medeltemperatur med 1,2 grader. Följaktligen tror IPCC att ytterligare en faktor 1,44 – till 600 ppm också kommer att höja jordens medeltemperatur med 1,2 grader. Dvs fördubbling av CO2 ger 2,4 grader. Nu är inte IPCC:s värde 416 ppm för deras data är publicerade för ett år sedan och gäller 2020 då CO2-halten var lite lägre.

    Klimatkänslighet 2,6 grader med rätt värden, men det är inte ECS, haven är inte i termisk balans med atmosfären, glaciäravsmältning går sakta och så vidare. Enligt IPCC kommer temperaturen att fortsätta att stiga trots att inga fler utsläpp sker. Maximum anser dom sannorlikt bli 3,0 grader ungefär år 2250.

    Din förmodan ”Eftersom det blir så liten ändring finns det inga andra parametrar som moln m.m. som heller ändrar sig.” är inte riktig. Ifall molnen redan ändrats så kommer dom rimligen att ha ändrats lika mycket till när vi nått 600 ppm. Att molnen påverkats av att den globala temperaturen stigit verkar de flesta tro – men på vilket sätt det påverkat värmebalansen verkar det finnas många olika åsikter om.

    Det kan vara bra att komma ihåg att IPCC anger trolig temperatur år 2100 till mellan 2,0 och 3,6 grader över förindustriell, alltså mellan 0,8 och 2,4 grader över dagens temperatur. Osäkerheten om hur mycket temperaturen väntas stiga är således en faktor tre !

  235. Gösta Pettersson

    #223 Leif Åsbrink

    Som jag nämnde i mitt KU-inlägg 2022-05-26 hänförde sig Loschmidts karakterisering av den gravito-termala effekten till ett energetiskt slutet system där ingen hänsyn togs till existensen av en värmetransport från jordyta via atmosfären till rymden. Men redan vid slutet av 1800-talet hade man tillräckligt god kännedom om den reellt föreliggande troposfäriska temperaturgradienten för att inse att den beskrevs väl av Loschmidts ekvationer. Varför Maxwell tog sig en funderare och föreslog att av gravitationen framkallad konvektion i vertikal led ser till att värmeenergin i troposfären ständigt omfördelas i riktning mot det av Loschmidt beskrivna gravito-termala jämviktsläget. Av Maxwell kallat ”konvektiv termisk jämvikt”.

    Betr. din fråga, så vet jag att av växthusgaserna absorberad värmestrålning bl. a. kan omvandlas till kinetisk energi och leda till en temperaturföjning. Men i den mån en sådan temperaturhöjning uppstår (det föreligger ju också samtidiga processer som tenderar att minska temperaturen) och avviker från det konvektiva jämviktsläget, så kommer den att vara temporär och övergående (transient). Därför att avvikelser från det konvektiva jämviktsläget genast kommer framkalla en konvektion som omfördelar atmosfärens värmeenergi och temperaturer i riktning mot det konvektiva jämviktsläget.

    Jag är helt övertygad om att växthusgaser, genom absorption och emission av IR-strålning, bidrar till energitransporten från jordyta till rymd. Men en följdsslutsats som Maxwells tankar leder fram till är att det inte spelar någon roll enligt vilka mekanismer värmetransporten genom atmosfären ombesörjs. Slutresultatet vad troposfärens temperaturer beträffar kommer ändå att bli att de avtar linärt med ökande höjd. Och med en lapse rate som återspeglar den gravito-termala effekten i enligt med Loschmidts ekvationer och Maxwells konvektiva jämvikt. Jämför med vad Peter Stilbs säger i #6.

    Vad Maxwell-Loschmidt säger är högst beaktansvärt, alldenstund deras teoretiska beskrivning av den troposfäriska temperaturgradientens uppkomst och karakteristika stått sig i 130 år och bekräftats av såväl statistisk-mekaniska studier som ett synnerligen omfattande meteorologiskt observationsmaterial.

    Och det är Maxwell-Loschmidts teorier Maxwellskolan stöder sig på när de drar slutsatsen att den 32-gradiga atmosfärseffekten återspeglar en gravito-termal effekt.

  236. Munin

    # 234
    Tack! Det är den mest koncisa och samlade beskrivning jag sett om IPCCs tankevärld. Jag återkommer när jag fått fatt i en kalkylator med ln funktion.

  237. Adepten

    #236 Munin

    Om du googlar på realcalc kan du installera den på din mobil😊

  238. Claes Johnson

    Mikael 201

    Titta på bilderna under Surface Energy Budget där de första två saknar back radiation och ger mycket liten roll åt radiation medan den tredje visar massiv back radiation. Fråga Dig om inte detta är förvillande. Om Du svarar ja, så har Du förstått taktiken bakom CO2Hysterin, nämligen att skapa så stor förvillning hos folket att marschen mot det fossilfria samhället inte längre kan ifrågasättas. KUs mission är att avslöja förvillningen genom att visa att det inte finns någon vetenskap som ens antyder att dubblad CO2 skulle kunna medföra märkbar global uppvärming. Men det verkar inte KUs ledande skribenter vilja framhålla. Och då kan förvillningen fortsätta, med sina förödande konsekvenser.

  239. Till KU

    Gösta får efter visst motstånd in en artikel på KU som påvisar att den av IPCC påstådda ”växthuseffekten” är oklar till sin natur och att speciellt effekten av dubblad CO2 inte på vetenskapliga grunder kan förmodas vara annat än så liten att den inte är märkbar/mätbar. Om Göran har rätt, vilket jag anser, så faller hela CO2Hysterin, och det finns en framtid för västvärlden. Men KUs ledning vill inte ge stöd åt Göstas resonemang och vill heller inte delta i någon sakdiskussion. Tystnad råder. Och tystnad är svår att tolka. Jag har flera gånger ställt frågan till LB och KU om vilka vetenskapliga belägg det finns för att dubblad CO2 kan medföra märkbar/mätbar global uppvärmning. Några svar ges inte. Tystnad råder. Diskussionen av Göstas inlägg är avslutad. CO2Hysterin får fortsätta. Men är inte KUs mission att upplysa om att CO2Hysterin saknar vetenskaplig grund? Svar ? /Claes

  240. Munin

    # 234
    Med utgångspunkt i den logaritmiskt avtagande funktionen för koldioxid enligt strålningsfysiken, oförändrad användning av fossila bränslen och därtill beräknade tidpunkter då vi når 450, 500, 550, 600 ppm i atmosfären får jag följande påverkan på globala medeltemperaturen. Ökningstakten för koldioxid ser ut att ligga på ca 2,5 ppm/år. Om det skulle vara så att tillgången på fossila bränslen ebbar ut tidigare får vi en vändpunkt och då får vi från den räkna med lägre årlig ökningstakt.

    ppm år ökad temperatur, grad
    416 2022 0
    450 2035 + 0,13
    500 2055 + 0,31
    550 2075 + 0,47
    600 2095 + 0,61

    Det rör sig om så låga tal att det kan inte vara försvarligt att fortsätta driva linjen att koldioxiden hotar jordens klimat! Att utgå från nuläget, som det är 2022, är också det mest naturliga när man ska bedöma vad som kan komma i framtiden. Hur det är precis i detta nu har varje människa en uppfattning om och kan relatera till. Att påstå att vi 2022 är i klimatkris, klimatnödläge, klimathelvete etc. måste vara något av den största villfarelse, som man vill påföra mänskligheten. Det är därför inte konstigt att det börjar liknas vid en religion. Religioner har alltid hållit på med domedagsföreställningar och att helvetet väntar om inte bot och bättring görs. Prästerskapet ser också till att de alltid får sin försörjning säkrad.

    Det finns inga trender i att extrema väderhändelser ökat eller att de skulle ha ökat i styrka. Vi har ännu inte haft någon så extrem väderhändelse att det inte går att hitta de som varit ännu värre bakåt i tiden. Att sedan befolkningsökningen lett till att det blir fler, som märker av en storm eller översvämning, är en helt annan sak och har inget med klimat att göra.

    Att hävda att någon extrem klimatändring inträffat i Sverige är verkligen oseriöst. Om det är något som skulle känna av klimatändringar är det jordbruket och det producerar, som aldrig förr. Samma gäller för hela världen med stadigt stigande totala skördar.

    Eftersom koldioxid är en grundval för fotosyntesen och högre halter ger ökade skördar kan det också finnas skäl att fundera på hur den funktion ser ut, som visar hur jordens totala växtmassa ökar vid ökning av koldioxiden i atmosfären. Kan det vara så att vi i den delen har en logaritmiskt ökande funktion vad gäller inlagring av kol. Är de minskande öknarna en indikator på att så är fallet.

    Vidare måste det ifrågasättas att IPCC börjar med 1850. Det finns ju inga globala mätserier, som går att validera mot varken vad gäller temperatur och halten koldioxid i atmosfären, Vi får ju gå till 1950-talet för att kunna börja hävda att det finns något globalt täckande underlag för de båda.

  241. Egentligen kan man håll på att maxwella, stråla och plancka så bäst man vill och klia sig i skallen.

    Historien är facit!
    Det finns inget påvisbart samband mellan CO2 och temp mer än att temp orsakar avgasning från havet.
    Snällt slukar havet CO2 igen när det blir kallare.

    Nu får ni teoretiker anpassa er efter det.

  242. Munin 240

    Beräkningen enligt ”strålningsfysiken” som Du anger utgör (hela) den vetenskapliga grunden för tesen att mer utsläpp av koldioxid orsakar global uppvärmning. Det är den enda grund som finns, men som LB inte vill yppa, av någon anledning.

    Man kan se på beräkningen på två sätt: Antingen anser man att beräkningen säger något som är de facto riktigt, och då att dubblad CO2 inte orsakar någon mätbar/märkbar uppvärmning. Eller så tillmäter man beräkningen litet värde eftersom den endast fokuserar på strålningseffekter vilka är mycket svåra att både mäta och beräkna precist.

    I båda fallen är beräkningen en liten Fjäder som inte kan ge stöd åt någon som helst tro på global uppvärmning av mänskliga utsläpp av CO2.

    Av denna lilla Fjäder gör CO2Hysterin en mycket Stor Höna som skriker ut att ”Himlen Ramlar Ner”.

    KUs uppgift är att visa att Hönan är en Fjäder. Men det gör inte KU, till skillnad från KlimatRealisterne i Norge. Varför?

    Makten har inlett resan mot ett FossilFritt Sverige, medan Folket står kvar på perrongen: https://claesjohnson.blogspot.com/2022/11/vill-svenska-folket-leva-i-fossilfritt.html

    KU behövs! KU behöver en levande debatt!

    Bra Johannes 241!

  243. Claes J #242,

    Nu får du sluta att trolla. Du som vet bäst kan väl starta en egen blogg istället för att slå in öppna dörrar här med fullkomligt stolliga påståenden om KU och om strålningsfysiken.

  244. Ingemar 243

    Vad menar Du med ”trolla”? Jag är ingen magiker. Vilka mina påståenden om KU är stolliga? Vilka mina påståenden om strålningsfysiken är stolliga?

    Jag har min egen blogg. Jag är säker på att det finns flera läsare på KU som läser min blogg och gärna ser att jag bidrar till KU. Jag är tillsammans med Dig ledamot av Klimatrealisternes Vetenskapliga Råd varför vi har en vetenskaplig gemenskap att ta tillvara. Låt debatten fortsätta. KU har en viktig roll att fylla. Vänligen Claes

  245. tty

    #242

    ”Eller så tillmäter man beräkningen litet värde eftersom den endast fokuserar på strålningseffekter vilka är mycket svåra att både mäta och beräkna precist.”

    Nu är det ju faktiskt precis tvärtom – strålningseffekterna är DEN ENDA del av växthuseffekten som är väl förstådd och beräkningsbar.

  246. Claes J 244,

    1. ”Troll” kallas sådana som hela tiden slänger in medvetet provocerande saker och vill få debattråden att spåra ur genom att byta ämne. Ergo. Du är ett troll som vill skifta bevisbördan till att vi skall visa att växthusgaser har en effekt.

    2. Du slår in öppna dörrar när du låtsas som om KU inte gör precis det du efterfrågar, nämligen att föra en argumentation mot klimatalarmismen.

    3, Du har stolliga föreställningar om att strålningsfysiken inte finns, eller är fel. Men strålningsfysiken är som den är. Den fråga som vi ofta diskuterar är istället hur stor uppvärmningseffekt detta har.

    Därför blir dina kommentarer här ganska ointressanta.

    Ämnet för denna tråd är, som jag uppfattar Göstas inlägg, hur Maxwellspåret förklarar den omfördelning av värme (gradienten i troposfären) som solen tillför klotet. För en sådan grov modell så behövs inte antaganden om växthusgasers strålningsegenskaper. Som jag förklarat flera gånger så ser jag dock inte detta som ett svar på frågan hur förändringar av klimatet uppstår eller utvecklas.

  247. Bubo

    #239
    Om Göran och du har rätt, vilket du anser, så tror jag ni bör skicka era beräkningar och resultat till någon ansedd vetenskaplig tidskrift som tusentals forskare inom området läser. Det kan ju knappast vara KUs roll att bedöma och granska ett så extremt revolutionerande resultat som skulle ikullkasta tusentals forskares tidigare resultat, revolutionera all strålningsfysik för att inte tala om spara tusentals miljarder kronor. Har ni gjort det?

    KU kan ju ta upp olika perspektiv vilket här gjorts men knappast ha rollen eller vara forumet för en fördjupad sakdiskussion i en sådan enorm fråga.

  248. Munin

    # 240
    Tillägg: Med den obetydlighet fortsatta ökningar av koldioxiden har för jordens medeltemperatur blir det viktigt att få hejd på de värsta avarterna av s.k. klimatåtgärder och det som hävdas vara nödvändig omställning. Ofantliga resurser riskeras att tas i anspråk helt i onödan. Professor Hassler har insett det vad gäller reduktionsplikten. Det finns dock mycket mer som är gökungar, en av de värsta är land- och havsbaserad vindkraft. Det finns ett antal ord, som gör att man bör tänka några varv extra när förslag om insatser föreslås och beskrivs. De är ”grön, klimat, hållbar och omställning”. Se dem i stället som varningssignaler när åtgärder och projekt ska bedömas.

  249. 235 Gösta Pettersson

    Det finns ingen (vad jag vet) som ifrågasätter det av Loschmidt beskrivna gravito-termala jämviktsläget. Av Maxwell kallat ”konvektiv termisk jämvikt”.

    Vi är alltså helt överens om att CO2 inte (nämvärt) har någon direkt effekt på laps rate. Däremot kan en ökad temperatur, oavsett orsak, förmodas förändra den absoluta luftfuktigheten och därigenom ändra laps rate.

    Du skriver: ”Betr. din fråga, så vet jag att av växthusgaserna absorberad värmestrålning bl. a. kan omvandlas till kinetisk energi och leda till en temperaturföjning.” Detta är ett anmärkningsvärt svar. Betrakta en enskild molekyl. Efter excitation de-exciteras den praktiskt taget alltid genom energiomvandling till värme. När du skriver bl.a. tolkar jag det som att du menar att någon annan process skulle kunna inträffa i stället. Vad menar du med det?

    Det måste vara så att du uppfattar något av min kommentar #67 som felaktigt – eller så har du inte läst den?

    Du skriver: ”Men i den mån en sådan temperaturhöjning uppstår (det föreligger ju också samtidiga processer som tenderar att minska temperaturen) och avviker från det konvektiva jämviktsläget, så kommer den att vara temporär och övergående (transient). Därför att avvikelser från det konvektiva jämviktsläget genast kommer framkalla en konvektion som omfördelar atmosfärens värmeenergi och temperaturer i riktning mot det konvektiva jämviktsläget.”

    Vad skulle det vara för processer? Konvektionen ser till att laps rate upprätthålls. Antalet CO2-molekyler per kubikmeter sjunker sakta med höjden, så varje volymselement är omgivet av andra volymselement som uppvärms ungefär lika mycket. Genom konvektion, latent värme och strålning upprätthålls laps rate och den nödvändiga konsekvensen blir att hela temperaturkurvan lyfts. Det blir lika mycket varmare vid marken som det blir varmare på den ursprungliga ”utstrålningshöjden”.

    I förenklad modell:
    Marktemperatur = 14 °C
    Laps rate = 6,5 °C/km
    Utstrålningsskiktets temperatur = -18 °C
    Växthuseffekt (IPCC,300ppm) = 32 °C
    Utstrålningsskiktets höjd(300ppm) = 32/6,5 = 4,92 km
    Marktemperatur(415ppm) = 15.2 °C
    Växthuseffekt (IPCC,415ppm) = 33,2 °C
    Utstrålningsskiktets höjd(300ppm) = 33,2/6,5 = 5,11 km

    Jag antar att du håller med om denna simpla räkneövning men att du anser att den höjning av utstrålningsskiktets höjd som vi observerar genom att mäta marktemperatur och förutsätta oförändrat laps rate inte beror på koldioxidens strålningsegenskaper.

    Utstrålningsskiktets höjd är en kvantitet som finns endast i en förenklad modell. I verkligheten står det för ett energiflöde som kan mätas från satellit. Man kan mäta total strålning från jorden och medelvärdesbilda över tid, plats, vinkel mot lodlinjen och frekvens. Då får man, när jorden är i jämvikt, samma energiflöde per m2 som man skulle få ur en ideal svartkropp vid -18 °C.

    Experimentellt finner man alldeles tydligt att strålningen från CO2 avtagit relativt resten av spektrum, exakt som man kan förvänta sig (se länk #67). När ökningen av CO2 gör att dess strålning avtar så måste strålningen från övriga gaser (och jordytan) ha ökat för att bibehålla den ekvivalenta svartkroppstemperaturen. Både atmosfären och jordytan måste ha blivit varmare eftersom strålningstemperaturen från CO2 sjunkit och den totala strålningstemperaturen i jämvikt måste vara -18 °C

  250. Gabriel Oxenstierna

    Stort tack till Gösta P. för att du i tråden mycket tydligt förklarat gravito-termala teorin.

    På fredag kommer en artikel här på KU om atmosfärseffekten utifrån CERES data, förhoppningsvis kan den belysa empirin runt influenser från GRAV och VÄXT, och ’X-faktorn’.

  251. Adepten

    #201 Mikael och #238 Claes Johnson

    Michael Manns hockeyklubba och Trenberhs-energibudgetdiagram är de mest vilseledande som serverats till sökande/blivande ”forskare” runt om i världen. Kan finnas en del annat också🙄? T.ex. beräkningarna av klimatkänsligheten och klimatmodellerna som alla ger spretande spagetti diagram🤔
    Allt detta kan ge upphov till en hel del intressanta inlägg för nytillkomna som är intresserad av klimatvetenskap😎

  252. Adepten 251

    Ja det finns mycket medvetet vilseledande humbug vad gäller CO2Hysterin. Du kan hitta avslöjad humbug på min blogg, tex Trenberths energibudget:
    https://claesjohnson.blogspot.com/2017/02/unphysical-basis-of-co2-alarmism.html
    Välkommen med frågor. Viktig ämne! Och roligt att förstå vad som är humbug och inte. Vänligen Claes

  253. GoranA

    Först måste jag tacka Gösta för inägget och alla som bidragit med kommentarer, främst Leif men även Claes och tty. Det har varit väldigt intressant att följa.

    #149 Leif Åsbrink Du skriver

    ”Experimentellt finner man alldeles tydligt att strålningen från CO2 avtagit relativt resten av spektrum, exakt som man kan förvänta sig (se länk #67).”

    Jag antar att det är punkt 6) du syftar på i #67

    6) Fördubblar man CO2-halten från 300 till 600 ppm kommer denna höjd att öka tills antalet ovanförliggande molekyler är nästan detsamma. (Spektrallinjens form beror av tryck och temperatur.) Därvid kommer strålningen från jorden att utgå från ett atmosfärsskikt med annan temperatur vilket betyder att utsänd strålning blir annorlunda.

    För mig är detta inte logiskt.
    Ökningen från ungefär 300 ppm CO2 i atmosfären från 1960 till nuvarande drygt 400 ppm gick uppenbart inte över en natt. Istället har denna ökning varit långsam och successiv. Med en långsam höjning av koldioxidens utstrålningsskikt p g a den ökande halten CO2 så borde väl temperaturen i skiktet följa med upp.

    Om det är så som jag beskriver ovan skulle det inte ge någon förändring av den strålning som utsänds från koldioxiden så som du skriver i pkt 6 eftersom temperaturen följer med upp.

    Är det inte istället så att koldioxidens utstrålningsskikt bestäms av den allmänna förtunningen av atmosfären med höjden så förhållandet att hinna avge en foton ökar visavi att avlämna energin till en luftmolekyl via kollision.

    Studerar man modtran modellen, http://climatemodels.uchicago.edu/modtran/ , sker utstrålning från CO2 i till stor del i tropopausen där temperaturen är tämligen konstant, utstrålningstemperaturen minskar inte med ökande höjd, speciellt vintertid.

  254. Munin

    Är det någon som kan strukturera upp ECS så att alla parametrar kommer med i en fullständig uppräkning. Just nu upplever jag det som ett ”hopkok” av allt möjligt! När det är gjort ska det sedan gå att fundera på i vilken storleksordning de ska ordnas utifrån inverkan på jordens medeltemperatur. Ordningsföljden föreslås vara att man börjar med de som bedöms öka jordens medeltemperatur mest och slutar med de som sänker mest. Eftersom det är IPCC, som infört begreppet är väl en lämplig grund att basera det på att det är vid 1,2 grader ökning av jordens medeltemperatur (de har ju det som konstant för fördubblingar av ppm-halten i atmosfären). I de fall det finns delade meningar om en parameter dvs. någon har källa att de höjer medan annan källa säger att de sänker är det bra att det kommer med, men viktigast är att parametern kommer med.

    Koldioxiden hör inte till listan då den redan är definierad av strålningsfysiken och följer funktionen om logaritmiskt avtagande effekt (IPCC har själva skrivit ut den formel som de anser ska användas),

  255. Här är en testfråga att besvara av både experter o lekmän med kunskap om den s k växthuseffekten GHE:

    Vilken huvudsaklig fysik förklarar att temperaturen på Venus yta är 470 C? Alternativ 1 eller 2? Motivera kort.

    1. Atmosfären består av 96.5 % koldioxid. (runaway GHE)

    2. Trycket på ytan är mycket stort, ca 92 ggr trycket på Jorden.

  256. Munin

    # 255
    Skriver man ut 1) som 965 000 ppm? Jorden har 416 ppm.

  257. Claes Johnson #239 & 168
    Tack för vetenskapligt eldunderstöd!
    Läste Din i #252 refererade blogpost kring Trenberths et al humbug. Själv har jag karaktäriserat hans energibudget, anammad av NASA, i ett av mina inlägg på klimatsans.com, som ett ’fysikaliskt falsarium’.
    Själv anser jag växthusgaseffekten vara en återvändsgränd. Det tongivande skälet är att den inte tillfredsställande kan förklara atmosfärens temperaturprofil, eftersom förenklat jord/sol/atmosfär-förhållandet, och klimatet/temperaturförhållanden som enföljd härav, primärt är en ’termodynamisk maskin’ (GRAV, på det sätt Gösta Pettersson ovan beskrivet detsamma) och inte en ’radiativ’ sådan (VÄXT), varför vi måste hålla isär vad som är orsak resp. verkan, enligt min uppfattning.

  258. Munin

    # 255
    Det bästa du kan göra är att ställa frågorna till Syukuro Manabe. Det bör vara fullt möjligt, forskare emellan. Han om någon måste vara rätt person och vi alla får ökad klarhet i striden på det höga vetenskapliga plan som hänger över hela klimatfrågan. Nobelkommittén kan säkert få upp en direktlinje till Manabe.

  259. Gösta Pettersson

    #249 Leif Åsbrink

    ”När du skriver bl.a. tolkar jag det som att du menar att någon annan process skulle kunna inträffa i stället. Vad menar du med det?”

    Att den exciterade molekylen även kan relaxeras genom emission av IR-strålning,

    ”min kommentar #67 som felaktigt – eller så har du inte läst den?”

    Jag har läst den men, men betraktat större delen av det du säger som irrelevant för att förstå Maxwellskolans argumentering.

    ”Vad skulle det vara för processer?”

    Konvektionen är processen. Med dina egna ord: ”Konvektionen ser till att laps rate upprätthålls.” Varvid jag i begreppet konvektion även inbegriper transporten av latent värme. Såväl torrradiatens som våtdiabatens lapse rate är proportionell mot gravitationskonstanten.

    ”Genom konvektion, latent värme OCH STRÅLNING upprätthålls laps rate”

    Där tycker jag mig hitta ett huvudskäl till att du har så svårt att anamma Maxwellskolans argumentering. Den troposfäriska temperaturgradienten med dess lapse rate är en gravito-termalt uppkommen och upprätthållen företeelse. Strålning är inte inblandad. Det finns inga strålningstermer i Loschmidts ekvationer, ej heller i uttycken för torr- och våtadiabaternas lapse rate.

    Därav följer att i luftskikt där det råder konvektiv jämvikt kommer alla temperaturskillnader att återspegla den gravito-termala effekten utan radiativa bidrag. Så även den 32-gradiga atmosfärseffekten. GRAV = 32. VÄXT = 0. Maxwellskolan har presenterat ett urtjusigt generellt belägg för att jordytans temperatur är ”precis” den som man kan förvänta för en planet på Jordens avstånd från solen.

    Beträffande dina tabelldata har jag inget nämnvärt att anmärka på mer än att du kallar den 32-gradiga atmosfärseffekten för växthuseffekt. Utgår man från att atmosfärseffekten är en växthuseffekt, så lär man inte kunna komma fram till något annat än att så är fallet. Förmår man inte skilja mellan atmosfärseffekt och växthuseffekt, så har man inte skuggan av en chans att begripa Maxwellskolans argumentering.

  260. TorbjörnR

    #255 Claes Johnson

    Din fråga är högintressant. Hade själv en tanke att analysera den utifrån GPs teori om gravitationseffekten. Vet dock inte om alla data är kända för Venus?

    Kanske GP kan göra en analys av Venus och se om teorin håller även där?
    Eller vet du redan svaret Claes?

    Igen, tack Gösta för ditt igen högintressanta inlägg. Det är ju så här vetenskap ska debatteras även fast de flesta av läsarna här inte är rätt målgrupp att säga emot.

  261. Gösta P #259,

    ”Maxwellskolan har presenterat ett urtjusigt generellt belägg för att jordytans temperatur är ”precis” den som man kan förvänta för en planet på Jordens avstånd från solen. Så även den 32-gradiga atmosfärseffekten. GRAV = 32. VÄXT = 0. ”

    Hur vet du det? Temperaturdifferensen på 32 grader är väl en empiriskt uppmätt temperatur minus den teoretiska för en svartkropp. I så fall ingår växthusgaserna i de 32 graderna, eller hur?

  262. Munin

    # 259
    Se # 255, samma bör gälla för det du tar upp. Nu har vi turen att ha en nybliven nobelpristagare, som har fått pris för sin växthusteori. Rimligen har han övervägt din teori och har argument varför han inte gick på den linjen. Då klimathotet lyfts upp som mänsklighetens ödesfråga är det inte mer än rätt att vi får klarhet i kunskapsläget. Med hjälp av Nobelkommittén bör det rimligen inte vara svårt få upp en direktlinje med Manabe. För Sverige är det också extra intressant då det verkar vara två svenska vetenskapsmän på 1800-talet, som är roten till de skilda synsättet. Dock framförallt är det mycket viktigt för att få veta om världen har kommit in på fel spår. Det handlar ju om att i ena fallet finns inget klimathot.

  263. tty

    Jag fattar inte detta ändlösa tuggande. Gravitationseffekten förklarar varför och hur mycket temperaturen sjunker med höjden. Den säger ingenting, jag repeterar INGENTING, om vad den absoluta temperaturen är.

  264. forts 255

    Om man söker på ”Venus temperature” på min blogg:

    https://claesjohnson.blogspot.com/search?q=venus+temperature

    hittar man en serie poster, där i den första LB anger Alt 1 som svar, dvs den höga yttemperaturen påstås bero på en runaway GHE. Läs och begrunda. Vad är det rätta svaret?

  265. Munin

    Kanske dum fundering? Vad säger strålningsfysiken händer i en atmosfär med så hög halt koldioxid som på Venus. På jorden träder logaritmiskt avtagande funktion för koldioxid in och gör att temperaturen slutar öka vid 600 ppm om det tillförs mer koldioxid.

  266. Adepten

    #265 Munin

    Jorden har H2O som växthusgas vilket inte Venus har.😁
    tty kan förklara det bättre😎

  267. tty

    #265

    ”På jorden träder logaritmiskt avtagande funktion för koldioxid in och gör att temperaturen slutar öka vid 600 ppm om det tillförs mer koldioxid.”

    Nej, den slutar inte att öka, den ökar bara allt långsammare.

  268. Munin

    # 267
    Ja, det var slarvigt skrivet. Jag hade i åtanke att när det gått över 600 ppm har jag läst i någon källa att det handlar om så lite effekt (temperaturpåverkan) att det inte är någon större anledning att räkna på det som tillkommer (i alla fall på jorden). Hur det blir för Venus vet jag inte.

  269. tty

    #265

    ”Vad säger strålningsfysiken händer i en atmosfär med så hög halt koldioxid som på Venus.”

    Ingenting egentligen. Vi vet för litet om venusatmosfären för att göra några meningsfulla beräkningar. Vi vet att det finns typ 200 000 gånger mera koldioxid än i jordatmosfären, men vi vet mycket litet om vilka andra gaser som ingår i den, vi vet att där finns N2, CO, SO2 och H2O, men det är ytterst osäkert vad som finns ytterligare i låga halter. Kanske fosfin, kanske ammoniak, kanske klorväte och fluorväte, kanske metallkarbonyler….
    Vi vet att molnen består till större delen av svavelsyra men det är också nästan det enda vi vet om dem. Dessutom är det mycket troligt att koldioxiden nära ytan är superkritisk, alltså befinner sig i ett aggregationstillstånd som vare sig är en gas eller en vätska men har drag av båda. Jag tror ingen vågar sig på att säga hur det påverkar klimatet.

    Kort sagt venusatmosfären är så annorlunda och så dåligt känd att det är meningslöst att försöka räkna på t ex dess optiska egenskaper.

    Med Mars är det annorlunda, där är atmosfären tunn, väl känd och innehåller mycket litet vattenånga. Där är koldioxiden den helt dominerande växthusgasen (och det finns faktiskt mer koldioxid än på Jorden, men mycket litet kväve och inget syre). Det är med andra ord en lättanalyserad atmosfär.

    Det finns bara två exotiska saker med den, dels att globala stoftstormar tidvis kraftigt påverkar temperaturen genom att skymma solen och dels att atmosfärtrycket varierar ganska markant under året eftersom frusen koldioxid på den södra polarkalotten delvis smälter under sydsommaren.

    Att sydsommaren dominerar på detta sätt beror på att mars bana är mera elliptisk än Jordens så sydsommaren är varmare. Detta kommmer dock att ändras så småningom p g a Milankovicheffekter, som troligen är ännu viktigare på Mars än på Jorden. Den mycket säregna lagerbildningen vid polerna tros bero på sådana Milankovichcykler:

    https://upload.wikimedia.org/wikipedia/commons/thumb/6/62/Martian_north_polar_cap.jpg/1024px-Martian_north_polar_cap.jpg

    https://en.wikipedia.org/wiki/Martian_polar_ice_caps#/media/File:South_Polar_Cap_of_Mars_during_Martian_South_summer_2000.jpg

  270. Gösta Pettersson

    #261 Ingemar Nordin

    ”I så fall ingår växthusgaserna i de 32 graderna, eller hur?”

    Jo, det är en möjlighet man inte bör utesluta. . Vilket fick mig att sätta upp ekvationen VÄXT + GRAV = 32. Men när man då enligt Maxwellskolans argument kommer fram till att GRAV=32, så har man samtidigt visat att VÄXT=0 på ett i mitt tycke urtjusigt generellt sätt.

    Värt att begrunda. Det tog mig fem år att inse styrkan av Maxwellskolans argument. Och ytterligare fyra år att komma fram till hur man bäst försöker förmedla den insikten till KU:s läsare. Nu är det gjort, så tack för det.

  271. #240 Munin

    Du ansätter att klimatkänsligheten för fördubbling av CO2 är 1,2 grader och gissar att koldioxidhalten kommer att öka linjärt. Det är en väldigt extrem ansats, du menar med detta att det inte finns någon återkoppling alls och att andra antropogena utsläpp saknar betydelse. Det funkar nog dåligt om du skulle ta 1979 till startår (då börjar UAH och då har också troposfäriska aerosoler stabiliserats.) Sätt in observerade CO2-värden så kommer du att se att temperaturen du beräknar stiger alldeles för sakta. Något fattas – och varför skulle detta något ha upphört just 2022 ? Har du någon förslag på vad som inte stämmer?

    Din slutsats instämmer jag till fullo med: ”Att påstå att vi 2022 är i klimatkris, klimatnödläge, klimathelvete etc. måste vara något av den största villfarelse, som man vill påföra mänskligheten. Det är därför inte konstigt att det börjar liknas vid en religion.” Jag är dock övertygad om att din argumentation är felaktig och därmed skadlig. Du gynnar bara alarmisterna som kan avfärda påståendet om att vi inte är i en klimatkris med att det är baserat på felaktiga teorier av okunniga vetenskapsförnekare. Bjud inte på det! Koldioxidutsläpp är ett allvarligt problem på lång sikt och det behöver hanteras genom att utsläppen går ned till en mycket låg nivå. MEN att låta boten vara värre än soten är alltid en dålig idé ! Långsiktigt effektiva åtgärder men ingen kortsiktig symbolpolitik är vad världen behöver.

    Du skriver vidare: ”Det finns inga trender i att extrema väderhändelser ökat eller att de skulle ha ökat i styrka. Vi har ännu inte haft någon så extrem väderhändelse att det inte går att hitta de som varit ännu värre bakåt i tiden.” Att uttrycka sig på detta sätt är att provocera och det är sakligt fel. Det du vill säga (förmodar jag) kan formuleras så här: ”Extrema väderhändelser är per definition sällsynta. På den korta tid som vi nu kan överblicka är antalet inte stort nog för att vi med statistisk signifikans skall kunna avgöra om antropogena utsläpp har haft någon inverkan.” IPCC har i AR6 studerat förändringar sedan 1950-talet. Se SPM sidan 10:

    Antalet ”hot extremes” (värmeböljor) har ökat i 41 av de 45 regioner dom delat in världen i och dom har funnit orsaken vara antropogena utsläpp i 39 av regionerna. 23 med ”high confidence” och 16 med ”medium confidence”

    Antalet ”observed change in heavy precipitation” (skyfall) har ökat i 19 av de 45 regionerna och dom har funnit orsaken vara antropogena utsläpp i 2 av regionerna. En med ”high confidence” och en med ”medium confidence”

    Antalet ”observed change in agricultural and ecological drought” (torka) har ökat i 12 och minskat i en av de 45 regionerna och dom har funnit orsaken vara antropogena utsläpp i 2 av regionerna med ”medium confidence” och inte i någon med ”high confidence”

    Man har alltså observerat ökad frekvens av både torka och nederbörd men ”Confidence in human contribution to the observed change” är ”Low due to limited agreement.” Kanske kommer IPCC med bättre klimatmodeller att i framtiden upptäcka att ökningen skett på ett sätt som stämmer bättre med modellerna. Det kan vi inte veta.

    Rubriken på sidan 10 är: ”Climate change is already affecting every inhabited region across the globe, with human influence contributing to many observed changes in weather and climate extremes” Detta är inte vad statistiken visar tycker jag.

    En annan ingång till problemet finns här: https://www.preventionweb.net/files/74124_humancostofdisasters20002019reportu.pdf Katastrofer inträffar hela tiden. Världen behöver vidta åtgärder för att lindra konsekvenserna alldeles oavsett om ökningen av antalet extremväder någon gång i framtiden kommer att kunna visas bero på CO2-utsläpp. Man listar 7348 katastrofer i perioden 2000-2019. Det är ungefär en varje dag! Intressant att notera att antalet jordbävningar i perioden 1980-1999 var 445 och att i perioden 2000-2019 har de ökat till552. Ökningen synes mig för stor för att bero på slumpen. På ökad CO2 beror den inte. Jag skulle tro att skillnaden beror på bättre rapportering av jordbävningar. Jordbävningar registreras av seismografer världen över så statistiken är rimligen ganska bra sedan åtminstone sextiotalet. Annat är det med torka och skyfall. Där har rimligen rapporteringen och statistiken förbättrats avsevärt mellan jämförelseperioderna.

    Världen har alltså klimatrelaterade katastrofer dagligen – men det går inte ännu att skylla detta på koldioxiden. Statistik och modeller är inte bra nog ännu.

    Sahelområdet är katastrofdrabbat. Situationen beskrivs så här: ”Sammanfattningsvis har bakomliggande orsaker som osäkerhet kring matförsörjning till följd av klimatförändringar länge bidragit till flera mångfacetterade konflikter i området. Sahel är därför ett varnande exempel på hur ett område med växande oroligheter och försämrad matproduktion, förvärrad av klimatförändringar, snabbt kan drabbas av humanitära katastrofer som präglar hela regioner.” och så här ”Nederbörden i regionen är oberäknelig, och temperaturen i regionen stiger ca 1,5 gånger snabbare än det globala genomsnittet, vilket vidare undergräver livsmedelsproduktionen. Bristen på mat förvärras ytterligare av en kontinuerlig befolkningstillväxt.” Det där med att befolkningen fördubblats på 25 år är rimligen en viktigare faktor än att medeltemperaturen sedan 1997 ökat med 0,7 grader. Mera här: https://manskligsakerhet.se/2020/03/12/humanitara-katastrofer-i-sahel-trots-upprepade-varningssignaler/

    Bästa långsiktiga klimatåtgärd: Se till att levnadsstandarden i Sahel snabbt förbättras! Då kommer befolkningen att stabiliseras snabbare. (ref. Hans Rosling) Dom behöver riktiga elnät med stabil kraftproduktion. ”Afrikanska regeringar sneglar mot sina kolfyndigheter för att förse sina växande befolkningar med energi (Schwikowski 2021, se fotnot). Kina, som nu är Afrikas största handels- och investeringspartner och som har Afrika som en källa för råmaterial, planerar koleldade anläggningar på kontinenten för att driva sina utvecklingsprojekt där (Goldstone 2021). På så sätt kan den svällande skaran av fattiga i Afrika söder om Sahara och andra fattiga, snabbt växande regioner bli potentiellt betydande producenter av växthusgasutsläpp under kommande årtionden.” https://natverketpopulation.com/nyligen-publicerade-tidningsartiklar/

    Är det någon som tror att det blir kinesiska kolkraftverk med hög verkningsgrad? Min förmodan är att det blir billigast möjliga. Det skulle vara klimatsmart att som Elsa Widding föreslagit låta Världsbanken och/eller olika länders klimatsatsningar gå till högeffektiva kolkraftverk i t.ex. Sahel. Hälften så mycket CO2 per producerad megawattimme eller mindre skulle jag gissa. Mycket billigare att subventionera räntan till lån för detta än att reducera CO2 i samma utsträckning på annat sätt. Snabbare demografisk transition blir den största effekten till år 2100 tror jag.

    När det väl finns ett elnät och ett par kraftverk med stor svängmassa blir det enkelt att komplettera med vind och sol. Kolkraften bör på sikt ersättas med kärnkraft, men att bygga kärnkraftverk i de fattigaste länderna är kanske inte någon bra idé redan nu. Kol som hävstång mot snabbare ekonomisk utveckling i de fattigaste länderna tror jag är vad världen bäst behöver. Speciellt de berörda länderna! Naturligtvis är olja och ännu hellre gas att föredra för länder som har.

  272. Gösta Pettersson

    #262 Munin

    Skäll inte på Manabe. Han angav klart att hans växthuseffekt uppstod på grund av att den konvektiva jämvikten ersattes av en förmodad radiativ jämvikt i den översta delen av troposfären, med påföljd att temperaturavtagandet där övergick från att vara linjärt till att vara krökt.

    I #1 påminner Staffan Granström om sitt fjolårsinlägg, i vilket han gav länkar till arbeten av Collony. Arbetena har redan diskuterats på KU, men i ett av dom visades diagram framtagna med datorprogram från NASA. Diagrammen beskrev temperaturavtagandet (under diverse förhållanden) som strikt linjärt från jordytan ända upp till tropopausen. Det tycks alltså föreligga observationmaterial som vederlägger Manabemodellens beräkningsresultat att temperaturavtagandet kröks i övre troposfären. Hans modell tycks stå i strid med gjorda observationer.Detta sagt som en kommentar till vad mitt inlägg formellt sett delvis handlade om.

    Men jag gläder mig åt att era kommentarer centrerats runt en diskussion av den gravito-termala effekten, vilken jag velat dra uppmärsamhet till.

  273. #259 Gösta Pettersson

    Du skriver ”Att den exciterade molekylen även kan relaxeras genom emission av IR-strålning,” och om min kommentar #67 ”Jag har läst den men, men betraktat större delen av det du säger som irrelevant för att förstå Maxwellskolans argumentering.”

    IR strålning är så extremt osannolikt jämfört med omvandling till värme att det kan helt försummas. Det handlar om faktorer i storleksordningen 1000-1000000 Jag minns inte detaljerna, men någon annan här på KU skrev om det för inte särskilt länge sedan.

    ”Där tycker jag mig hitta ett huvudskäl till att du har så svårt att anamma Maxwellskolans argumentering. Den troposfäriska temperaturgradienten med dess lapse rate är en gravito-termalt uppkommen och upprätthållen företeelse. Strålning är inte inblandad. Det finns inga strålningstermer i Loschmidts ekvationer, ej heller i uttycken för torr- och våtadiabaternas lapse rate.”

    Nej, det är inte här problemet ligger. Du skriver: ”Därav följer att i luftskikt där det råder konvektiv jämvikt kommer alla temperaturskillnader att återspegla den gravito-termala effekten utan radiativa bidrag.” Javisst. Men det jag ville framhäva, måhända klumpigt, var att molekylerna befinner sig i termodynamisk jämvikt med omgivande luft inom frekvensområden där absorptionen är stor. Det innebär bland annat att strålning bidrar till energitransport från varmare till kallare områden. Därav följer inte att jag tror att laps rate förändras. Det skulle vara detsamma (nästan) om atmosfären bara innehöll kväve, syre och argon.

    Det borde du sett av min lilla tabell:
    Marktemperatur = 14 °C
    Laps rate = 6,5 °C/km
    Utstrålningsskiktets temperatur = -18 °C
    Atmosfärseffekt (IPCC,300ppm) = 32 °C
    Utstrålningsskiktets höjd(300ppm) = 32/6,5 = 4,92 km
    Marktemperatur(415ppm) = 15.2 °C
    Atmosfärseffekt (IPCC,415ppm) = 33,2 °C
    Utstrålningsskiktets höjd(300ppm) = 33,2/6,5 = 5,11 km
    Samma laps rate 6,5 °C/km i båda fallen.

    Den centrala frågan är hur Maxwellskolan förklarar att utstrålningsskiktets höjd har ändrats. Vad är det enligt Maxwellskolan som bestämmer utstrålningsskiktets höjd?

    Vad skulle utstrålningsskiktets höjd vara enligt Maxwellskolan om atmosfären bara innehöll kväve, syre och argon?

  274. #8 Claes Johnson

    Svaret finns i min kommentar #67. Både teori och experimentell evidens.

    Strålningen sett utifrån rymden har förändrats över tid. Strålningen från CO2 och metan har försvagats jämfört med resten av spektrum. Man vet att strålningen totalt sett är ungefär oförändrad, jorden är skapligt nära jämvikt så summan av all strålning motsvarar en svartkropp vid -18 °C.

    CO2 och metan har blivit svagare, då måste övrig strålning blivit starkare och därav följer att jorden och övriga gaser har blivit varmare.

  275. Gösta Pettersson

    #263 tty

    ”Jag fattar inte detta ändlösa tuggande. Gravitationseffekten förklarar varför och hur mycket temperaturen sjunker med höjden. Den säger ingenting, jag repeterar INGENTING, om vad den absoluta temperaturen är.”

    Jag håller helt med dig. Men din brist på förståelse för tuggandet har nog sin förklaring i att du ännu inte förstått Maxwellskolans argumentering. Den stöder sig inte på vad de absoluta temperaturerna är, utan på vilka temperaturskillnader som gravitationseffekten ger upphov till när den får temperaturen att sjunka med höjden.

    Atmosfärseffekten (32 grader) är en TEMPERATURSKILLNAD (mellan jordytan och den balansskapande temperaturen -18 °C). Vad Maxwellskolan säger är att vid konvektiv jämvikt bestäms alla temperturskillnader i troposfären av temperaturavtagandets lapse rate (en gravitationellt bestämd storhet) och representerar därmed en gravitationseffekt. Det gäller som specialfall även för den 32-gradiga temperaturskillnaden mellan jordytan och utstrålningsskiktets -18 °C.

    Med andra ord, den 32-gradiga atmosfärseffekten kan tillfredsställande förklaras som en gravito-termal effekt. Det finns ingenting som tyder på att några andra (t. ex. radiativa) effekter signifikant bidrar till göra jordytan 32 grader varmare än den balansskapande temperaturen -18 °C.

  276. Gösta Pettersson

    #273 Leif Åsbrink

    ”Den centrala frågan är hur Maxwellskolan förklarar att utstrålningsskiktets höjd har ändrats.”

    Frågan må vara central för dig, men Maxwellskolan har ingen anledning att diskutera utstrålningskiktets höjd när de drar slutsatsen att den 32-gradiga atmosfärseffekten är en gravitationseffekt,

  277. #24 Claes Johnson

    Vad gäller Schwarzschild’s ekv. tycker jag du skall läsa vad Peter Stilbs skrev om saken här: https://klimatupplysningen.se/stralning-atmosfaren-ett-ofta-missforstatt-koncept/ läs och begrunda vad Hartwig Volz skrev.

    Du skriver ”Radiativ värmeöverföring är ett elektromagnetiskt resonansfenomen med cut-off av höga frekvenser som ökar med temperaturen och som därmed ger en riktning från varm till kall, enligt min mening i alla fall.”

    Radiativ värmeöverföring är en sak och strålning en annan. Någon resonans behöver inte vara inblandad och cut-off är lite väl starkt för det branta avtagandet mot höga frekvenser. Det är inte ”cut-off” som är orsaken till att värme genom infraröd strålning alltid går från varmare till kallare.

    Jag har förenklat lite nedan genom att inte ta hänsyn till rotationsenergin.

    Om du har en gas, t.ex. luft med lite CO2 och har lite olika temperatur på olika sidor om en värmeisolerande men IR-transparent vägg så kommer temperaturen att utjämnas trots att strålningen endast förekommer runt cirka 667 cm-1 (vid temperaturer lite eller ganska mycket under rumstemperatur.) Strålningens intensitet beror av temperaturen. En CO2-molekyl har en sannolikhet att vara exciterad som beskrivs av Boltzmannfördelningen https://sv.wikipedia.org/wiki/Boltzmannf%C3%B6rdelning. Den enskilda exciterade molekylen har en viss liten sannolikhet att de-exciteras genom strålning. Strålningen utsänds i slumpmässig riktning. Varje CO2-molekyl som kan nås av strålningen och befinner sig i grundtillståndet har en sannolikhet att absorbera strålningen och därmed värma upp gasen omkring sig. De-excitation genom strålning har mycket låg sannolikhet jämfört med de-excitation genom omvandling till värme.

    I utrymmet med högre temperatur där en större andel av CO2-molekylerna är exciterade kommer alltså strålning att sändas ut med högre intensitet än i det andra utrymmet. Det ger en energiöverföring från varm till mindre varm gas.

    Notera att jag inte behöver nämna fotoner. Dom hör hemma i en mycket användbar modell av verkligheten men om man är filosofiskt lagd och vill veta vad en foton egentligen är skall man helst undvika begreppet.

  278. Munin

    # 271
    Du har på ett föredömligt sätt i # 234 redogjort för nyckeluppgifter, som IPCC använder sig av eller kommit fram till i sin tankevärld. Jag väljer att kalla det för tankevärld för det som täcks av ECS verkar ha en bit kvar för att vara stabil vetenskap. Det jag funnit vara grundelementet i hela klimatvetenskapen, som vilar på strålningsfysiken, är att det finns en logaritmiskt avtagande funktion för koldioxid vad gäller inverkan på jordens medeltemperatur (och som i linje med det gör det troligt att klimateffekterna samvarierar med detta i en nedåtgående trend och att om det skulle existera något som kan kallas klimathot försvinner det också med detta). Hur man ska se på grundelementet tycker jag tty gör så tydligt i # 245.

    Jag lutar mig inte mot någon linjär funktion, snarare verkar det vara IPCC, som gör det, enligt dina uppgifter i # 234.

    I # 240 har jag gjort en tabell, som enligt funktionen för koldioxid visar dess påverkan på jordens medeltemperatur från idag och till vi når 600 ppm. Ökningstakten följer en linjär funktion då jag inte har något djupare information om den delen. Det verkar dock inte avvika från IPCCs då de ”tror det nås ungefär år 2100.” Det som jag tycker i det sammanhanget är viktigt att korrigera för är om det kommer en vändpunkt före dess. För några år sedan var det väldigt inne att tala om ”Peak oil”.

    Det här med att det är svårt att hävda att vi idag skulle ha extremare väderhändelser än någonsin tidigare kan Tyskland och deras floder vara ett sätt att visa det på. Vi har alla fått då och då höra att nu är det katastrofala översvämningar i någon av floderna. Granskar man det noggrannare finner man att de genom historien markerat de högsta nivåer på någon stabil byggnad och hittar streck på ännu högre nivåer. Samma med att när det larmas om att någon flod håller på att torka ut kommer det fram stenar på flodbotten med inmejslade årtal, som ligger flera hundra år tillbaka i tiden. Mer närliggande för oss och dessutom klimatrelaterat är att trädgränsen i vår fjällvärld legat flera hundra meter högre än idag. Ett annat faktum är när odlingsgränser har flyttats, som i fallet med vikingarnas odling av korn på Grönland på 1000-talet och som inte går till mognad idag. Det går hela tiden att finna stöd i den fysiska verkligheten att vi inte är i närheten av något slag av mycket extremare väderhändelser.

    När det gäller ökad ökenutbredningen har jag haft bilden att det larmet nu är avfört då satellitmätningarna med tydlighet visat på motsatsen dvs. en ökad grönska och minskande ytor öken.

    Jag delar helt bilden att befolkningsökningen och mänsklig omdaning av jordens yta och mark måste vara med för att bedöma vad som gör att det som förr i tiden var en väderhändelse idag utmålas, som en katastrof. Fanns där tidigare en skog eller vegetation hände inget vid kraftigt regn medan samma regn idag när det faller på bar mark och spolar bort jorden upplevs som förstörande.

  279. Munin

    # 272
    Jag har inte alls skällt på Manabe! Min poäng är att han är auktoriteten på området och att det därför är rimligt att han kan säga sin uppfattning om det du hävdar ska vara rätt vetenskap. En jämförelse: Jag ser det som att om någon skulle finna skäl att ifrågasätta relativitetsteorin och Einstein fortfarande levde skulle väl det första vara att höra med Einstein direkt.

  280. Gösta P #270,

    Jomen det går ju inte ihop! Grav =32 gäller bara temporärt emedan svartkroppsstrålningen, beräknad till – 18 grader och den empiriskt uppmätta GT = +14 måste bero på fler saker än bara den gravitationella effekten.

    Man säger att det det var c:a 20 grader kallare här i Sverige under förra glacialen. Låt oss anta att det globalt var 10 grader kallare. Skulle Maxwells beräkningar då visa en exakt differens på 22 grader? Eller är det så att andra faktorer, X, också spelar roll? I ”andra faktorer” ingår förstås växthusgasernas påverkan.

    Återigen: Du kan inte från den maxwellska förklaringen till temperaturgradienten dra några slutsatser om vilken storlek det är på X-termen, och därmed inte heller storleken på VÄXT-termen.

  281. Adepten

    Detta intressanta inlägg av Gösta Pettersson måste väl ha gett årets flesta antal kommentarer😲

    Men jag förstår fortfarande inte hur det kommer sig att gravitationen kan ge upphov till 32-33 graders ytuppvärmning.

    Tycker att nedanstående inlägg och kommentarer från tty var upplysande när det gäller lapse rate🤩 https://klimatupplysningen.se/klimat-knep-o-knap/

  282. Lars Cornell

    #281 Adepten. Jag skall försöka ge dig ett litet pedagogiskt tips.
    Uppe på 10000 m höjd är det ca 50 grader kallt. Sätt nu igång och räkna, för det vet jag att du kan. Antag att en liten molekyl på den höjden plötsligt skulle få för sig att ramla ända ned till marken. Då omvandlas den lilla rackarens lägesenergi i slutskedet till värmeenergi. Vilken temperatur skull molekylen få när den nått marken?

    Det handlar i grunden om något så trivialt som energiomvandling. Utan gravitation finns det ingen lägesenergi och du kan gå och sova i stället för att räkna.

  283. GoranA

    #149 Leif Åsbrink Du skriver

    ”Experimentellt finner man alldeles tydligt att strålningen från CO2 avtagit relativt resten av spektrum, exakt som man kan förvänta sig (se länk #67).”

    Jag antar att det är punkt 6) du syftar på i #67

    6) Fördubblar man CO2-halten från 300 till 600 ppm kommer denna höjd att öka tills antalet ovanförliggande molekyler är nästan detsamma. (Spektrallinjens form beror av tryck och temperatur.) Därvid kommer strålningen från jorden att utgå från ett atmosfärsskikt med annan temperatur vilket betyder att utsänd strålning blir annorlunda.

    För mig är detta inte logiskt.
    Ökningen från ungefär 300 ppm CO2 i atmosfären från 1960 till nuvarande drygt 400 ppm gick uppenbart inte över en natt. Istället har denna ökning varit långsam och successiv. Med en långsam höjning av koldioxidens utstrålningsskikt p g a den ökande halten CO2 så borde väl temperaturen i skiktet följa med upp.

    Om det är så som jag beskriver ovan skulle det inte ge någon förändring av den strålning som utsänds från koldioxiden så som du skriver i pkt 6 eftersom temperaturen följer med upp.

    Jag önskar en kommentar. Jag ställde samma fråga i #253 men fick ingen respons så jag försöker på nytt 🙂

  284. Munin

    # 282
    Har du en bild av vilka molekyler det handlar om? Är det i huvudsak fråga om vattnets kretslopp? Hur är koldioxiden inblandad i dessa rörelser?

  285. Här är en ny post som beskriver osäkerheten i den av IPCC angivna radiative forcing (värmande effekt) av dubblad CO2 av storlek 3-4 W/m2, vilken är grundbulten för CO2Hysterin:

    https://claesjohnson.blogspot.com/2022/11/global-warming-measuring-temperature-vs.html

    Posten visar att osäkerheten är så stor att grundbulten inte håller. Och då håller inte heller överbyggnaden som CO2Hysteri.

  286. Adepten

    #282 Lars Cornell

    Utan att tänka skulle jag säga ett molekylen bli 98 grader varmare om de omgivande molekylerna var lika torr och kall🙄
    Den kan ju bli 60 grader varmare om den faller i en fuktig atmosfär också🤓
    Om den är helt ensam skulle den kunna utveckla 223×molekylmassan (w) i medeleffekt🤔
    Men det är säkert något lurt med frågan som jag förbisett🥴

  287. Munin

    # 284
    Tillägg: Var placerar gravitationen i genomsnitt atmosfärens molekyler? Kan det vara så att om man går till lapse rate kurvan (minskning av temperaturen med stigande höjd) finns det egentligen en parallell lapse kurva, men den ska visas som atmosfärens genomsnittliga sammansättning vid varje gradtal. Då blir det så att gravitationen är den överordnande faktorn och den bestämmer hur energiutbytet blir med rymden, eftersom den därmed också avgör hur strålningsfysiken kommer att verka. Det är denna gravitationens uppstyrning av atmosfärens molekyler, som gör att det blir ett visst grundflöde i jordens energiutbyte med rymden och där dyker de 32 graderna upp. Tillförs sedan mer koldioxid till atmosfären ser gravitationen till att de i genomsnitt hamnar så att de ger en viss tillkommande inverkan på marginalen, men denna är avtagande enligt funktionen om logaritmiskt avtagande inverkan.

  288. JonasW

    Loschmidts ekvation är den ”adiabatiska approximationen”. Någon som tror att det är en adiabat när luft stiger uppåt (=ingen värmeväxling med omgivningen) ?

    Jag tycker att virial teoremet är en bättre modell. Ett mångpartikelsystem i ett gravitationsfält. Virialteoremet ger dessutom en lapse rate strax under 7C/km, vilket stämmer bättre med observationer än Loschmidts ekvation.

    Virialteoremet är härlett under antagandet att bara konservativa krafter verkar på systemet. Om man inför en absorption, d.v.s. att partiklarna växelverkar med ett elektromagnetiskt fält så blir det (betydligt) mer komplicerat.

    Utan att ha gjort egna beräkningar så är min bild att en ökad växelverkan med ett elektromagnetiskt fält mycket väl kan påverka lapse rate (utifrån virial teoremets modell). Kan vara så att det är fel, men jag har inte läst någon analys av hur ett mångpartikelsystem i växelverkan med ett elektromagnetiskt fält påverkas av ökad absorption.

    Kort – jag håller det inte alls för osannolikt att ökad IR absorption i atmosfären påverkar lapse rate. Frågan är helt central. Om lapse rate minskar något, så blir CO2 effekten väsentligt mindre.

  289. #276 Gösta Pettersson

    Detta du skriver är absurt. ”Frågan må vara central för dig, men Maxwellskolan har ingen anledning att diskutera utstrålningskiktets höjd när de drar slutsatsen att den 32-gradiga atmosfärseffekten är en gravitationseffekt,”

    Den 32-gradiga atmosfärseffekten finns inte längre. Just nu finns i stället en 33,2-gradig atmosfärseffekt som naturligtvis är en gravitationseffekt.

    För att Maxwellskolan skall ha någon relevans måste den förklara varför gravitationseffekten ökat. Du anser att Maxwellskolan inte har någon anledning att diskutera utstrålningskiktets höjd, det är inte OK. Hela diskussionen handlar ju om förändringen av atmosfärseffekten. Har den med ökade CO2-halter att göra helt eller delvis eller är det något annat fenomen som styr?

  290. Bubo

    Vågar man upprepa sin fråga från #247? Med all respekt för KU och cleas blogg så tror jag inte det är forum som läses av huvuddelen av världens forskare. Jag tror inte heller det är detta forums roll, men rimligen borde man om man kommit med spännande tankar och resultat vilja dela dessa med andra.

    Frågan var om dessa resultat har skickats in till ansedda tidsskrifter för bedömning och vetenskaplig diskussion? Om inte, varför? Om det gjorts, vilken respons blev det?

  291. Munin

    # 281 Adepten
    Du har bidragit med mycket viktigt material, efterhand som diskussionen i tråden fortlöpt!

    Tror du på min hypotes hur gravitationen är den överordnande faktorn, som bestämmer hur jordens temperatur blir genom att styra hur atmosfärens massa struktureras över tid och att till detta måste sedan strålningsfysiken tillämpas för att få fram hur effekten blir som temperatur på jorden och att häri ligger också hur de stabila 32-33 graderna kan förklaras dvs. den berömda växthuseffekten?

    Tacksam också för synpunkter, som helt sänker hypotesen, från de som anser så.

  292. #253 och #283 GoranA

    ”Experimentellt finner man alldeles tydligt att strålningen från CO2 avtagit relativt resten av spektrum, exakt som man kan förvänta sig (se länk #67).”

    Detta syftar på ”Figure 1 här: https://skepticalscience.com/print.php?r=35 ”Med en långsam höjning av koldioxidens utstrålningsskikt p g a den ökande halten CO2 så borde väl temperaturen i skiktet följa med upp.”

    Utstrålningsskikt är inte någon fysisk realitet utan ett begrepp från en enkel modell. Strålningen från CO2 kommer från hela troposfären. Temperaturen i hela troposfären följer med upp. https://clivebest.com/blog/?p=4597 Figur 7 kan jämföras med experiment, skepticalscience.com ovan.

    Enligt Clive Best ger en fördubbling av CO2 än reduktion av radiansen från 90 till 70 mW/m2-sr-cm-1 vid 700 cm-1 Det blir som strålningstemperaturer från 262,5 till 246,6 dvs 16 grader.

    Från 1970 till 1996 har strålningstemperaturen sjunkit med 1,5 grader vid 700 cm-1 CO2 har ändrat sig från 325 till 355 ppm. ln(325)=5.78 ln(355)=5.87 skillnad 0,09 ln(2)=0.69 så experimentellt värde motsvarar 1,5*0,69/0,09 = 11.5 grader. Nu har ju hela atmosfären blivit 1,2 grader varmare så vi skall jämföra 12.7 med 16 grader. Stämmer utmärkt givet alla osäkerheter.

  293. tty

    #282, 286

    Faller en molekyl fritt från 10 000 meters höjd kommer den att nå en hastighet av ca 440 m/s. Hade molekylen någon hastighet när fallet började måste denna vektoradderas.

    Vilken temperatur detta motsvarar får man ur formeln

    v = (3RT/m)^0,5

    där v är molekylhastigheten i m/s, R är den ideala gaskonstanten (8,314), T temperaturen i K och m massan av en mol av den aktuella gasen i kg.

    Påpekas bör dock att sannolikheten att en molekyl skall kunna falla fritt från 10000 meters höjd utan att kollidera med en annan molekyl är så liten att det troligen inte har hänt under Jordens hela historia. I genomsnitt är den fria vägen innan en molekyl kolliderar med en annan vid havsytan mindre än en tiotusendels millimeter. På 10000 meters höjd är den dock drygt en tiotusendels millimeter.

  294. tty

    #292

    Resonemanget kompliceras dock av att i mitten av CO2:s absorptionsband ligger utstrålningshöjden ovanför tropopausen där temperaturen stiger med höjden och utstrålningen alltså istället ökar vid högre koldioxidhalt.

  295. Matz Hedman

    #292,#293 Leif och tty
    Det här är nog den längsta tråden hittills. Om jag räknat rätt 16.9 meter i 400 DPI.😄

    Det visar sig att den ökande solinstrålningen vida överglänser någon CO2 effekt över Europa. Vad kan det bero på? Kan man utreda orsak? Är ni med att diskutera på den fria tråden på söndag?

  296. #41 Claes Johnson

    ”Man kan mäta temperaturen av ett objekt med ett instrument som är varmare, men inte återstrålning eftersom den inte finns. Temperatur är lätt att mäta, strålning svårare, återstrålning omöjligt.”

    Du är uppenbarligen inte experimentalfysiker. Vilket som är svårast, temperatur eller strålning beror på omständigheterna. Det finns i allmänhet förträffliga metoder för båda storheterna.

    Jag anser som du att återstrålning är ett dumt begrepp. Varje gång du träffar på ordet stryk förledet ”åter”.

    Växthuseffekten traditionell beskrivning: Jorden sänder ut infraröd strålning enligt Stefan Boltzmanns lag. Strålningen absorberas till viss del av atmosfären på grund av dess innehåll av infrarödaktiva gaser. Detta innebär en energitransport från jordytan till atmosfären. Dock liten jämfört med latent värme och konvektion. Den varma atmosfären sänder ut strålning i alla riktningar på grund av sitt innehåll av infrarödaktiva gaser. En del av strålningen från atmosfären träffar jordytan och absorberas där. Energin som lämnar jordytan är därför skillnaden mellan utstrålad energi enligt Stefan Boltzmanns lag och energin som kommer från atmosfären. Denna energiförlust innebär en transport av energi från jordytan och uppåt.

    Återstrålning är dumt begrepp. Det antyder att strålningen från atmosfären beror på att atmosfären mottagit strålning från jorden. Så är det inte. Det är enbart atmosfärens temperatur som betyder något. En lustighet är att strålningen uppåt från marken är svagare än strålningen uppåt från 50 meters höjd (enligtMODTRAN) På koldioxidens resonanslinjer är absorptionen och därmed emissiviteten 100,0% på en sträcka av 50 meter. Markens emissivitet är storleksordningen 95%.

    Notera att Stefan–Boltzmanns lag är felaktigt beskriven här: https://sv.wikipedia.org/wiki/Stefan%E2%80%93Boltzmanns_lag Där står att lagen beskriver värmestrålningen för en kropp med temperaturen T till en omgivning med temperaturen T0. Detta är INTE Stefan–Boltzmanns lag utan en formel som beskriver värmeöverföringen genom strålning mellan två kroppar. Den är härledd ur Stefan–Boltzmanns lag.

    Engelska Wikipedia ger en korrekt beskrivning: https://en.wikipedia.org/wiki/Stefan%E2%80%93Boltzmann_law Ett objekt som utsänder termisk strålning gör detta beroende av temperatur och emissivitet helt oberoende av omgivningens egenskaper. Omgivningen må absorbera och/eller, reflektera strålningen. Omgivningen kan också sända ut strålning, termisk eller annan form, laser t.ex. Vad slags strålning omgivningen sänder till objektet kan absorberas reflekteras eller förorsaka stimulerad emission och alltså transportera energi till eller från objektet eller vid reflektion inte påverka objektet men den termiska strålningen påverkas inte.

    Jag har slagit upp Nationalencyklopedin och tittat i ett par gamla läroböcker i fysik som jag fått ärva efter Professor Einar Lindholm. Dom har alla den korrekta definitionen. Fysikböckerna ger också formeln för energiöverföringen till omgivningen specifierad som hur en svartkropp beter sig när den är instängd i en låda med ”omgivningens temperatur.”

    Jag har också tittat i 1975-1976 års upplaga av Handbook of Chemistry and Physics och där definieras Stefan–Boltzmanns lag felaktigt, precis som på svenska Wikipedia.

    Formeln som ger energiöverföringen till omgivningen kan absolut inte tolkas som att ingen strålning utsänds. Däremot, om ingen omgivning finns kan ingen energi överföras till omgivningen.

  297. #49 Claes Johnson

    Se #292

  298. #85 Claes Johnson

    ”Däremot saknar en foton/partikel modell tydlig fysik och någon tröskeleffekt vad gäller värmeöverföring finns inte heller. Många verkar tro att varje kropp spottar ur sig fotoner som värme oberoende av omgivningens temperatur, men varför tror man på något sådant?”

    Jag kan inte hålla med. Den konventionella foton/partikel modellen må sakna tydlig fysik, men den beskriver hur kvantmekanikens formler beter sig så länge vi betraktar fenomen som beskrivs av en-foton-vågfunktioner. Vad gäller det som i dag heter sammanflätade fotoner, en vågfunktion som beskriver två fotoner har jag aldrig funderat på ifall foton/partikel modellen leder till några svårigheter.

    Problemet är att du blandar ihop värme med strålning. Man må prata om värmestrålning, men det är slarvigt och oegentligt.

    ”Många verkar tro att varje kropp spottar ur sig fotoner som värme oberoende av omgivningens temperatur, men varför tror man på något sådant?”

    Alla utom du och möjligen några få andra vet att kroppar som inte befinner sig i sitt grundtillstånd har en viss sannolikhet att de-exciteras genom att utsända elektromagnetisk strålning. Det är alltså energi som om den absorberas av något påverkar detta någots temperatur. Finns inte något i närheten så sker de-excitationen i alla fall och strålning utsänds.

    ”fotoner som värme” är det väl ingen som tror på. Man säger visserligen värmestrålning om den infraröda strålning som värmer utomhusserveringar på höstkanten. Men strålningen är inte värme. Den är IR-strålning som kommer att värma människor som utsätts för strålningen.

    Man kan tala om strålningstemperatur. Det är särskilt praktiskt i radiosammanhang (brustemperatur), men om man överför det till infrarött så är det Spectral Radiance (mW/m2-sr-cm-1) alltså den svartkroppstemperatur som skulle givit samma intensitet vid betraktad frekvens. Radiansen beror oftast på betraktningsvinkeln precis som albedot. https://ncc.nesdis.noaa.gov/data/planck.html

    Se vidare #296